80
Prefazione ` E con grande piacere che vedo apparire la nuova edizione, aggiornata e completata, del libro I problemi di Matematica della Scuola Normale . L’idea di fornire agli studenti liceali e ai loro insegnanti una serie di esempi di problemi di matematica, molti dei quali di tipo non proprio convenzionale, nacque all’inizio degli anni ottanta, nel corso di una riunione della Commissione che preparava le prove per il concorso di ammissione alla Scuola Normale. Si trovarono i testi delle prove assegnate a partire dal 1906, e si pens` o che il presentarli con le soluzioni discusse e commentate avrebbe potuto essere un contributo utile alla didattica e alla diffusione della matematica. Il successo che il libro ha avuto suggerisce ora di riproporlo in forma pi` u estesa con gli aggiornamenti fino al 1997. Desidero ringraziare, a nome della Scuola Normale, il professor Franco Conti e il dottor Alessandro Profeti, che hanno svolto con la loro riconosciuta competenza questo arduo compito, ed esprimo l’augurio che questo testo accompagni generazioni di giovani nell’apprendimento dei segreti di una tra le pi` u belle conquiste della mente dell’uomo. Franco Bassani Direttore della Scuola Normale Superiore Introduzione Il presente volume si propone come continuazione dell’opera I problemi di Matematica della Scuola Normale Superiore, pubblicato nel 1985, che raccoglieva in maniera sistematica gli esercizi di matematica assegnati per il concorso di ammissione negli anni 1906 –1984. Entrano in questa raccolta tutte le prove di esame assegnate dal 1985 fino al 1997 e una selezione degli esercizi pi` u interessanti (scelta difficile e un poco arbitraria) degli anni precedenti. I temi presentano una grande variet` a, che riflette tanto le preferenze culturali dei docenti che di anno in anno si avvicendano nella commissione di concorso, quanto l’esigenza, sempre presente, che le prove non discriminino sulla base di nozioni diligentemente apprese e meccanicamente applicate ma evidenzino invece l’inventiva dei candidati. Nei temi assegnati sono egualmente rappresentate l’innovazione e la tradizione. Accanto a problemi di calcolo delle probabilit` a e problemi di matematizzazione di situazioni concrete, che non hanno ancora trovato spazi adeguati nell’insegnamento della matematica, compaiono temi di tipo combinatorio o aritmetico. Uno spazio abbastanza ampio continua ad essere dedicato ai problemi di geometria piana e solida, che a parere dei curatori hanno rappresentato e dovrebbero continuare a rappresentare un momento importante per l’educazione della sensibilit` a e della intuizione in matematica e che al momento vedono molto ristretto il loro ruolo nei programmi di matematica della scuola secondaria. Il libro non ` e inteso come una raccolta di temi svolti da cui attingere ricette per la soluzione di problemi, obbiettivo questo difficile da un lato e di dubbia utilit` a educativa dall’altro; ` e auspicio dei curatori che esso venga inteso come una raccolta di domande a cui lo studente in procinto di partecipare all’esame di ammissione o semplicemente il lettore interessato alla matematica cerca in primo luogo di rispondere personalmente, per poi confrontare le proprie strategie risolutive con quelle esposte nelle soluzioni. Per questo motivo ` e parso utile fornire di alcuni esercizi pi` u soluzioni, basate sulle diverse componenti della educazione matematica di uno studente di scuola media superiore. Questa raccolta si rivolge in primo luogo agli studenti e ai docenti della Scuola Secondaria, tenendo conto che ` e sempre pi` u sentito fra gli insegnanti il problema di stimolare e indirizzare gli alunni pi` u dotati, ma anche a tutti coloro che sono interessati a conoscere i metodi di soluzione di problemi matematici anche difficili e a cimentare la propria inventiva matematica, anche se tale disciplina ` e lontana dalla loro attivit` a quotidiana. Il testo dei problemi ` e premesso, per invogliare il lettore a cercare da solo le soluzioni, confrontandole poi con quelle esposte nella seconda parte, che non sono necessariamente le migliori possibili. Un asterisco contraddistingue gli esercizi che, nell’opinione dei curatori, presentano particolari elementi di originalit` a o di difficolt` a o di fantasia. Conclude il volume un glossario in cui vengono richiamate alcune nozioni utili per affrontare i quesiti; i termini succintamente citati nel glossario compaiono nel testo in carattere evidenziato. Nella stesura del glossario si ` e preferito dare dei lemmi una presentazione pi` u agile e semplice, rinunciando ad una maggiore generalit` a che avrebbe appesantito l’esposizione. Infine una osservazione: contrariamente all’uso italiano si ` e scelto di non distinguere tipograficamente gli enti geometrici dalle loro grandezze o dalla loro misura, conformemente con una tradizione consolidata nella letteratura anglosassone. I curatori Pisa, aprile 1998

I problemi di Matematica della Scuola Normale Superiore

Embed Size (px)

DESCRIPTION

Collection of normale di Pisa's admission math exams from 1900 to 2000

Citation preview

Prefazione

E con grande piacere che vedo apparire la nuova edizione, aggiornata e completata, del libroI problemidi Matematica della Scuola Normale.

L’idea di fornire agli studenti liceali e ai loro insegnantiuna serie di esempi di problemi di matematica,molti dei quali di tipo non proprio convenzionale, nacque all’inizio degli anni ottanta, nel corso di unariunione della Commissione che preparava le prove per il concorso di ammissione alla Scuola Normale. Sitrovarono i testi delle prove assegnate a partire dal 1906, esi penso che il presentarli con le soluzioni discussee commentate avrebbe potuto essere un contributo utile alladidattica e alla diffusione della matematica.

Il successo che il libro ha avuto suggerisce ora di riproporlo in forma piu estesa con gli aggiornamenti finoal 1997. Desidero ringraziare, a nome della Scuola Normale,il professor Franco Conti e il dottor AlessandroProfeti, che hanno svolto con la loro riconosciuta competenza questo arduo compito, ed esprimo l’augurioche questo testo accompagni generazioni di giovani nell’apprendimento dei segreti di una tra le piu belleconquiste della mente dell’uomo.

Franco BassaniDirettore della Scuola Normale Superiore

Introduzione

Il presente volume si propone come continuazione dell’opera I problemi di Matematica della ScuolaNormale Superiore, pubblicato nel 1985, che raccoglieva in maniera sistematica gli esercizi di matematicaassegnati per il concorso di ammissione negli anni 1906–1984.

Entrano in questa raccolta tutte le prove di esame assegnatedal 1985 fino al 1997 e una selezione degliesercizi piu interessanti (scelta difficile e un poco arbitraria) deglianni precedenti.

I temi presentano una grande varieta, che riflette tanto le preferenze culturali dei docenti chedi annoin anno si avvicendano nella commissione di concorso, quanto l’esigenza, sempre presente, che le provenon discriminino sulla base di nozioni diligentemente apprese e meccanicamente applicate ma evidenzinoinvece l’inventiva dei candidati.

Nei temi assegnati sono egualmente rappresentate l’innovazione e la tradizione. Accanto a problemi dicalcolo delle probabilita e problemi di matematizzazione di situazioni concrete, che non hanno ancora trovatospazi adeguati nell’insegnamento della matematica, compaiono temi di tipo combinatorio o aritmetico. Unospazio abbastanza ampio continua ad essere dedicato ai problemi di geometria piana e solida, che a pareredei curatori hanno rappresentato e dovrebbero continuare arappresentare un momento importante perl’educazione della sensibilita e della intuizione in matematica e che al momento vedono molto ristretto illoro ruolo nei programmi di matematica della scuola secondaria.

Il libro non e inteso come una raccolta di temi svolti da cui attingere ricette per la soluzione di problemi,obbiettivo questo difficile da un lato e di dubbia utilita educativa dall’altro;e auspicio dei curatori cheesso venga inteso come una raccolta di domande a cui lo studente in procinto di partecipare all’esamedi ammissione o semplicemente il lettore interessato alla matematica cerca in primo luogo di risponderepersonalmente, per poi confrontare le proprie strategie risolutive con quelle esposte nelle soluzioni. Perquesto motivoe parso utile fornire di alcuni esercizi piu soluzioni, basate sulle diverse componenti dellaeducazione matematica di uno studente di scuola media superiore.

Questa raccolta si rivolge in primo luogo agli studenti e ai docenti della Scuola Secondaria, tenendoconto chee sempre piu sentito fra gli insegnanti il problema di stimolare e indirizzare gli alunni piu dotati,ma anche a tutti coloro che sono interessati a conoscere i metodi di soluzione di problemi matematici anchedifficili e a cimentare la propria inventiva matematica, anche se tale disciplinae lontana dalla loro attivitaquotidiana.

Il testo dei problemie premesso, per invogliare il lettore a cercare da solo le soluzioni, confrontandolepoi con quelle esposte nella seconda parte, che non sono necessariamente le migliori possibili.

Un asterisco contraddistingue gli esercizi che, nell’opinione dei curatori, presentano particolari elementidi originalita o di difficolta o di fantasia.

Conclude il volume un glossario in cui vengono richiamate alcune nozioni utili per affrontare i quesiti;i termini succintamente citati nel glossario compaiono neltesto in carattereevidenziato. Nella stesuradel glossario sie preferito dare dei lemmi una presentazione piu agile e semplice, rinunciando ad unamaggiore generalita che avrebbe appesantito l’esposizione.

Infine una osservazione: contrariamente all’uso italiano si e scelto di non distinguere tipograficamentegli enti geometrici dalle loro grandezze o dalla loro misura, conformemente con una tradizione consolidatanella letteratura anglosassone.

I curatoriPisa, aprile 1998

Testi 1

1906.1* Sono dati, in un piano, un circolo e due punti. Tirare una tangente al circolo in modo che lasomma delle distanze della tangente dai due punti dati sia uguale a una lunghezza data.

1909.1 In un piano sono dati due circoli che si tagliano in due punti.Si domanda di condurre per uno diquesti punti una retta sulla quale le corde staccate dai due circoli stiano fra loro in un rapporto dato.

1909.2* Dimostrare che sea1, a2, . . . , an sonon numeri diversi da zero, fra i quali sussiste la relazione(a 2

1 + a 22 + · · · + a 2

n−1

) (a 2

2 + a 23 + · · · + a 2

n

)=

(a1 a2 + a2 a3 + · · · + an−1 an)2,

i numeri stessi sono necessariamente in progressione geometrica.

1911.1 Dividere un dato arco di cerchio in due archi le cui corde siano tra loro in un rapporto dato.

1912.2* In un dato quadrilatero inscrivere un parallelogrammo il cui centro cada in un punto dato.

1915.2* Dimostrare che se per un punto interno ad una sfera si conducono tre piani, a due a dueperpendicolari, le somme delle aree dei tre cerchi che essi determinano sulla sferae costante.

2 I problemi della Scuola Normale

1923.2* Dati due punti sopra le facce di un diedro, quale la spezzata minima che li unisce e che ha unvertice sullo spigolo?

1926.2* Condurre un piano che seghi un dato angolo tetraedro in un parallelogrammo.

1927.2 Determinare il luogo dei centri delle circonferenze ottenute segando una superficie sferica con ipiani passanti per un punto qualunque dello spazio.

1929.2* Verificare che il luogo dei puntiM tali che la somma dei quadrati delle loro distanze da tre puntidatiA, B, C sia uguale ad una assegnata costantek2 e un cerchio.

1939 E dato un rettangoloABCD. Facendolo ruotare attorno al latoAB si ottiene un primo cilindroe facendolo ruotare attorno al latoBC si ottiene un secondo cilindro. Si conosce la somma 4π S2 dellesuperfici laterali dei due cilindri; e pure si conosce la somma π V 3 dei volumi degli stessi. Si domanda dideterminare i lati del rettangoloABCD.Qual e la condizione di realta? Quante soluzioni ha il problema? FissatoV , qual e il massimo valorepossibile perS?

1943 Determinare un triangolo rettangolo di cui siano noti il perimetro e il raggio del cerchio inscritto.Una volta fissato il raggio del cerchio inscritto, quale il valore minimo del perimetro?

1960.3 E piu facile, gettando una volta un dado, ottenere 6, oppure, gettandolo tre volte, ottenere tutte etre le volte un numero pari?

1961.4* Con una bilancia a piatti e un certo numero di pesi, si vogliono pesare oggetti di peso inferiorea 500 grammi, con un errore non superiore ad un grammo. Non si possono mettere pesi nel piatto su cuipoggia l’oggetto. Dire quale il numero minimo di pesi sufficienti a tale scopo.

1962.5* Si sostiene talvolta che noi usiamo il sistema decimale di numerazione (per cui, ad esempio,362 significa 3· 102 + 6 · 10 + 2) in quanto abbiamo 10 dita.Un marziano, dopo aver visto scritta l’equazione:

x2 − 16x + 41 = 0,

invitato a scrivere la differenza delle radici scrive 10.Quante dita hanno i marziani?(N.B. Per i numeri compresi fra 0 e 6 la scrittura dei marzianicoincide con la nostra).

Testi 3

1963.5 Un podista si trova su un punto della Terra, che supponiamo perfettamente sferica. Percorre unchilometro verso nord, poi uno verso est e infine uno verso sud. Si ritrova al punto di partenza.Quali sono i punti di partenza che obbediscono a questa condizione?

1967.2* Sono assegnate tre rette parallele. Esiste un triangolo equilatero con i vertici rispettivamentesulle tre rette?

1968.3 Sono dati in un piano quattro puntiA, B, C, D, in modo cheA, B, C e A, B, D siano verticidi triangoli equilateri distinti. Determinare tutte le circonferenzeβ che godono della seguente proprieta: iquattro puntiA, B, C, D hanno dalla circonferenzaβ ugual distanza.

1970.4 Fissato un intero positivon, determinare il piu piccolo interom tale che, presi comunquemnumeri interi positivi, una almeno delle seguenti eventualita si verifichi:(a) tra glim numeri considerati ve ne sonon uguali;(b) tra glim numeri considerati ve ne sonon distinti.

1971.4* Una palla si trova su un biliardo in posizioneP . Provare che esiste almeno una direzionesecondo cui si puo lanciare la palla in modo che essa non ripassi mai per la posizioneP .Si consideri il biliardo privo di attrito e si supponga che ilrimbalzo alle sponde obbedisca alla stessa leggedi riflessione della luce.

1973.3* Un treno parte da Pisa. Il macchinista controlla il cronometro e nota che la lancetta dei secondie sullo zero. Dopo aver percorso 8 chilometri, il macchinista controlla di nuovo il cronometro e nota chela lancetta dei minuti copre esattamente quella delle ore. La velocita media del treno per gli 8 chilometripercorsie di 33 chilometri l’ora. A che orae partito il treno da Pisa?

1974.5 Dati tre numeri interia, b, c aventi massimo comun divisore 1, verificare che i numeri della forma

a m2 + b m + c ,

conm intero qualunque, non possono essere tutti divisibili per 14.Generalizzare il risultato.

1976.3 Sian un intero maggiore di 2 e sia∆ un triangolo rettangolo. Dimostrare che la potenzan-esimadella lunghezza dell’ipotenusa di∆ e maggiore della somma delle potenzen-esime dei cateti.

4 I problemi della Scuola Normale

1977.1* Su un tavolo orizzontale vie una pila di 7 dischi metallici perfettamente uguali, ognuno didiametro 40 cm.Dire qualee la distanza massima sul piano orizzontale che puo avere il centro del disco piu alto dal centrodel disco piu basso senza che la pila crolli.

1979.1* Un battello scende lungo un fiume; sia alla partenza sia a ognistazione intermedia salgonosul battello tanti passeggeri, ognuno diretto ad una diversa stazione, quante sono le fermate successive.Sapendo che il numero massimo di passeggeri contemporaneamente presenti sul battelloe 380, si determiniil numero delle stazioni.

1979.3 Per un puntoP passano tre superfici sferiche distinte tra loro. Si considerino le affermazioniseguenti:(a) nessuna retta passante perP e tangente a tutte e tre le sfere;(b) nessuna sferae tangente a un’altra;(c) esiste un altro puntoQ comune alle tre superfici sferiche.

Dire, per ogni coppia di affermazioni, se esse sono incompatibili, se sono equivalenti o se una delle dueimplica l’altra.

1980.6 Un’autostrada han caselli a distanze successive dip chilometri. Sie osservato che ogni macchinaentra con uguale probabilita da ogni casello e esce con uguale probabilita da un altro casello. Trovare lalunghezza del percorso medio di ogni macchina.

1981.6* Dato un tetraedro avente 5 dei 6 spigoli di lunghezza minore ouguale a 2, provare che il suovolumee minore o uguale a 1. In quale caso il volumee uguale a 1?

1982.5* Dato un pianoα e due puntiP , Q nello stesso semispazio si considerino le sfere passanti per ipunti P , Q e tangenti al pianoα.Si richiede di determinare il luogo dei punti di tangenza.

1983.5* Due amici si sono iscritti alla prima classe di un liceo. Taleliceo ha due sezioni, le cui primeclassi hanno rispettivamenten em studenti, conn em compresi tra 20 e 30.Sapendo che la probabilita che i due amici si trovino nella stessa classee 1/2, dire quanti sono gli studentidelle due classi.

Testi 5

1984.6* Siano dati una circonferenzaγ e un puntoP distinto dal centro. SiaPAB un triangolo che, tratutti quelli che hanno un vertice inP e i rimanenti due suγ, abbia perimetro massimo.Dimostrare che le due bisettrici uscenti dai verticiA eB passano per il centro diγ.(Non si richiede la costruzione geometrica, ne la determinazione degli elementi del triangolo.)

1985.1 L’eguaglianza

p! + q! + r! = s!

e soddisfatta perp = q = r = 2 es = 3.Dire se esistono altri numeri interi positivi per cui tale eguaglianzae vera.(Si ricorda chen! indica il fattoriale din, cioe il prodotton · (n−1) · . . . · 3 ·2 · 1 dei primin numeri interi.)

1985.2* Si considerino nel piano due circonferenzeγ e γ′ di eguale raggio. Determinare il luogo deipunti mediP dei segmentiAA′ conA in γ eA′ in γ′.

1985.3 Fra i triangoli equilateri contenuti in un quadrato assegnato, determinare quelli di area massima.

1985.4 Fissati due puntiP e Q su due lati consecutivi di un dato rettangolo, si determinino sugli altridue lati due puntiR eS tali che il quadrilateroPQRS abbia area massima.

1985.5 Per la costruzione di un certo ponte si prevede che il costo diogni arcata sara di 18s2 miliardidi lire, oves e la distanza in chilometri tra i due piloni di sostegno di quell’arcata, mentre il costo di ognipilone sara di mezzo miliardo.Se il ponte deve essere lungo 3 chilometri quale sara il costo minimo dell’opera?

s1 s2 sn

3 km

6 I problemi della Scuola Normale

1985.6 Con una bilancia a piatti e disponendo di infiniti pesi campione

p, p1, q1, p2, q2, . . . , pn, qn, . . .

da

1, λ, 1/λ, λ2, 1/λ2, . . . , λn, 1/λn, . . .

grammi rispettivamente, doveλ e un numero reale maggiore di 1, si vogliono pesare tutti gli oggetti conuna precisione arbitrariamente grande. Per quali valori diλ cio e possibile?( N.B. Si dispone di un solo esemplare di ogni peso campione e non si possono mettere pesi campione sulpiatto che contiene l’oggetto da pesare ).

1986.1 Si determinino gli interi positivik tali che il polinomio

x5 + x4 + x3 + k x2 + x + 1

sia prodotto di polinomi a coefficienti interi di grado minore di 5.

1986.2 Si dimostri che il sistema di equazioni

y ex − e−y = x y ex

x ey − e−x = ey

ha una sola soluzione.

1986.3 Si dimostri che la composizione di due omotetie nello spazio, con poliP e Q distinti, e ancorauna omotetia di poloR, allineato conP eQ, oppure una traslazione parallela aPQ.

1986.4 Sia ABC un triangolo isoscele di baseBC con l’angolo al verticeBAC minore di 60. Sicostruisca un altro triangoloPQR, di baseQR, circoscritto e simile adABC, tale che il puntoA appartengaal segmentoQR e si abbiaQA = 2 · AR.

1986.5(a) Sianoα, β, γ, δ quattro angoli minori di 180. Si dimostri che

sinα + sinβ + sinγ + sinδ ≤ 4 sinα + β + γ + δ

4.

(b) Utilizzando la relazione precedente, si dimostri che lasomma dei seni degli angoli interni di un triangoloe sempre minore o eguale a 3

√3/2.

Testi 7

1986.6 Si consideri un biliardo di forma triangolare, come in figura, conα = 30 e si supponga di lanciareuna bilia dal puntoA.

α

A B

Si provi che la bilia, qualunque sia la direzione iniziale, effettua solo un numero finito di rimbalzi prima dibattere sulla spondaAB.Si determini anche il numero massimo di tali rimbalzi.Si studi infine il caso in cuiα e un angolo generico.(Si supponga che il biliardo sia privo di attrito, che la palla sia puntiforme e che il rimbalzo sulle spondeobbedisca alla legge di riflessione della luce).1

1987.1 Siano assegnati due numeri reali positivi non nullir e p, con r < p. Tra tutti i quadrilatericonvessi di perimetrop, aventi la somma delle lunghezze di una coppia di lati consecutivi uguale adr, sidetermini quello di area massima.

1987.2 Sianop, q, r tre numeri reali tali che il polinomio

A(x) = x3 + p x2 + q x + r

abbia tre radici reali.Determinare tre numeri realia, b, c, espressi in funzione dip, q, r soltanto, in modo che il polinomio

B(x) = x3 + a x2 + b x + c

abbia per radici i quadrati delle radici diA.

1987.3 Sia dato un segmentoAB nel piano. Si consideri il luogoL dei punti del piano che vedono ilsegmentoAB sotto un angolo di 60. Si scelgaP in L e si scelgano due puntiC eD rispettivamente interniai lati BP eAP del triangoloABP , in modo cheAD = BC.Si costruisca il triangolo equilateroCDQ di baseCD, esterno al quadrilateroABCD.Si studi, al variare diP in L e di C, D secondo le condizioni indicate sopra, il luogo dei punti delpianodescritto dal puntoQ.

1 Il testo originale del problemae leggermente diverso (N. d. C.).

8 I problemi della Scuola Normale

1987.4 Un punto (x, y) del piano si dira razionale sex e y sono numeri razionali. Data una qualunquecirconferenza del piano cartesiano avente centro razionale, si provi che se essa contiene un punto razionale,allora contiene infiniti punti razionali.

1987.5 Nella figurae rappresentato lo sviluppo delle facce di un tetraedro regolare nello spazio.

A C

B

M

N O

P Q

R

SianoP , Q, R tre punti distinti del tetraedro corrispondenti, nello sviluppo, rispettivamente ad un puntointerno al segmentoMN , un punto interno al segmentoMO, un punto interno al triangoloMOC.Siaα il piano contenenteP , Q, R. Si determini, nello sviluppo piano, l’intersezione fra ilpianoα e le faccedel tetraedro.

1987.6 Tizio si trova nella sua abitazione e deve prendere un treno che parte dalla stazione esattamentetra mezz’ora. Sotto la sua abitazione c’e la fermata degli autobus della linea A che lo portano alla stazionein 20 minuti. A 5 minuti di cammino vie una fermata delle linee B e C che lo possono portare alla stazionein 18 minuti.Tizio non conosce l’orario di passaggio degli autobus, ma sache su ognuna delle linee gli autobus passanoogni quarto d’ora.Quale strategia conviene a Tizio per aver maggiore probabilita di prendere il treno?

1988.1 SianoA, B, C, D quattro punti distinti assegnati nello spazio. Determinare una condizionenecessaria e sufficiente affinche ogni superficie sferica che passa perA eB intersechi ogni superficie sfericache passa perC eD.

1988.2 Un ragazzo ha concluso la terza media, sa che sei bravo e ti chiede:(a) cosae un poligono piano,(b) cosae l’area di un poligono piano.Esponi le definizioni richieste con rigore, chiarezza e concisione (in un massimo di 12 righe).Volendo dare le stesse definizioni ad un livello scolastico piu avanzato, quali precisazioni occorrerebbefare? (assiomi e definizioni precedenti, enunciati di eventuali teoremi,. . .).

Testi 9

1988.3 Si considerino i numeri naturali 1, 11, 111,. . . , e in generale si indichi conαn il numero che siottiene giustapponendon cifre uguali a 1.(a) Si provi che seαn e un numero primo alloran e primo.(b) Si provi che, assegnato comunque un numero naturaler, non divisibile ne per 2 ne per 5, si puo trovare

unαn multiplo di r.(c) Si scriva un algoritmo o programma per calcolatore (in unqualunque linguaggio di programmazione)

che, a partire dar, calcoli il minimon per cui vale la (b).

1988.4 Sia assegnato su un piano un numeron arbitrario di triangoli con la proprieta che tre qualsiasidi essi abbiano almeno un punto in comune. Si dimostri che tutti i triangoli assegnati contengono unostesso punto. Come occorre modificare l’ipotesi perche la stessa conclusione valga per un numero finito ditriangoli nello spazio?

1988.5 Sia p(x) = a x4+ b x3+ c x2+d x+e un polinomio con coefficienti razionali. Si supponga che, perogni interom maggiore di un certom0, il numerop(m) sia intero. Si dimostri che allora 24a e un numerointero.Si generalizzi questo risultato a polinomip(x) con coefficienti razionali di grado qualsiasi.

1988.6 Un laboratorio deve organizzare il trasferimento di 10 m3 di scorie radioattive liquide. Occorreordinare un numeron di contenitori, identici, che possano contenere tali scorie e garantire un trasportosicuro.Si stima che il costo di ciascuno di tali contenitori sia 16V 2 milioni di lire, ove V e il volume (in m3) discorie che ciascuno di essi puo contenere; il costo di riempimento di ciascun contenitorerisulta essere diun milione di lire, indipendentemente dalla sua capienza.(a) Quanti contenitori e di quale volume dovra ordinare il laboratorio per spendere il meno possibile?(b) Al momento di effettuare l’ordine si viene a sapere che laditta fornitrice pratica sul prezzo dei contenitori

ordinati un piccolo sconto delk per cento, conk intero, se il loro numero uguaglia o supera le 50 unita(il costo del riempimento rimane inalterato). Quale il piu piccolok per il quale risulta convenientemodificare l’ordine, e perche?

1989.1 Per 0≤ x ≤ π/2 en = 0, 1, 2, . . . , si ponga:

Fn(x) = 1− sinx + sin2 x − . . . + (−1)n sinn x.

Provare che:(a) per 0≤ α ≤ 1/2, l’equazioneF2k = α non ha soluzioni qualunque siak naturale;(b) per 1/2 < α ≤ 1, esiste un numero naturalek∗ tale che, per ognik > k∗, l’equazioneF2k = α ha almeno

due soluzioni.

1989.2 SiaS una superficie sferica di centroO. Per ogniP ∈ S, siafP : S → S l’applicazione che adogni Q ∈ S associa il puntofP (Q) simmetrico diQ rispetto all’asseOP . Dimostrare che:(a) per ogniP ∈ S l’applicazionefP e composizione di due simmetrie rispetto a piani;(b) per ogniP , X, Y ∈ S la distanza traX eY e uguale alla distanza trafP (X) e fP (Y ).(c) per ogniP , Q, X ∈ S si ha chefP (fQ(X)) = ffP (Q)(fP (X)).

10 I problemi della Scuola Normale

1989.3 Trovare le soluzioni reali del sistema:

2y + x − x2 − y2 = 0

z − x + y − y (x + z) = 0

− 2y + z − y2 − z2 = 0

1989.4 Consideriamo la legge che ad ogni puntoP = (x, y) del piano cartesiano fa corrispondere il puntof (P ) dello stesso piano, definito da:

f (P ) =

P se OP ≤ 1( x

OP,

y

OP

)se OP ≥ 1

doveO = (0, 0) eOP indica la distanza daO aP .Provare che per ogni coppia di puntiP eQ la distanza fraf (P ) e f (Q) non supera la distanza fraP eQ.

1989.5 SiaS la superficie di un prisma a base ottagonale regolare inscritto in un cilindro circolare rettodi raggioR e altezzah = 3R sin(π/8). SianoA eB due punti come in figura, conOA = R/

√2.

A O

B

P

N M

Determinare la lunghezza del minimo percorso suS traA eB.

1989.6 Siaf (x) una funzione a valori reali definita sulla semiretta realex ≥ 0. Supponiamo che:(a) f (x) sia derivabile con derivataf ′ continua;(b) f (0) = 0;(c) per ognix ≥ 1 risulti

0 < f (x) ≤ xf ′(x).

Provare che l’equazionef (x) = k ha almeno una soluzionex ≥ 0, per ognik ≥ 0 .

1990.1 Considerare nello spazio euclideo nove punti distinti a coordinate intere. Dimostrare che neesistono due tali che il segmento che li congiunge contiene almeno un punto interno (cioe distinto dagliestremi) a coordinate intere.

Testi 11

1990.2 Sia P un poligono semplice (cioe tale che da ogni vertice escono esattamente due lati) nonnecessariamente convesso, con almeno 4 lati. Supponiamo che P abbia al piu un vertice concavo.E’ vero che esistono due vertici non consecutivi con la proprieta che il segmento che li congiungee contenutoin P? In caso affermativo dimostrarlo, altrimenti trovare un controesempio.

1990.3 Dato il sistema

x1 + x2 + . . . + x100 = 5050

x22 − x2

1 = 3

· · · ·x2

k − x2k−1 = 2k − 1

· · · · · ·x2

100− x299 = 199

trovare tutte le soluzionix1, x2, . . . , x100, conxk ≥ 0, k = 1, 2, . . . , 100.

1990.4 Sia dato il polinomioF (x) = xn + an−1xn−1 + . . . + a0 con coefficientiai interi. Supponiamo che

esistano quattro interi distintia, b, c, d tali cheF (a) = F (b) = F (c) = F (d) = 7.Dimostrare che non esiste nessun numero interok tale cheF (k) = 12.

1990.5 Trovare il piu piccolo numeroα > 1 tale che risulti:

α + sinx

α + siny≤ ey−x per ognix, y conx ≤ y

1990.6 Si consideri un rettangoloR di misure 6×4 metri. Nel punto di mezzoO di un latoe incernieratoun braccio articolato, della lunghezza totale di due metri,formato da due segmentiOA eAB (vedi figura).Il braccio puo muoversi solo all’interno diR. Piu precisamente il segmentoOA puo ruotare intorno alpunto fissoO e, per ogni posizione assunta, il segmentoAB puo ruotare intorno al puntoA; naturalmente,durante il movimento, il braccio deve restare inR.

OA

B

R

E’ possibile scegliere le lunghezze dei segmenti in modo cheogni punto diR a distanza minore o uguale adue metri daO sia raggiunto daB?

12 I problemi della Scuola Normale

1991.1 Provare che per ogni numero interon ≥ 2 si ha

n√

n! <n + 1

2

e che (n + 1)/2 none mai un multiplo intero din√

n!

1991.2 Fra tutti i quadrilateri convessi inscritti in un quadrato,in modo che ogni lato del quadratocontenga almeno un vertice del quadrilatero, si determinino quelli aventi minimo e massimo perimetro.

1991.3 Trovare il piu piccolo numero interoN0 ≥ 1 con la proprieta cheN0 + 1 e 2N0 + 1 siano entrambiquadrati perfetti.Mostrare poi che ogni intero N con questa proprieta e multiplo diN0.

1991.4 Su un treno, inizialmente senza passeggeri e formato dan carrozze, salgonok viaggiatoridisponendosi in modo casuale e indipendente l’uno dall’altro. Quale la probabilita che solo tre carrozzesiano occupate da almeno un viaggiatore?

1991.5 Costruire un polinomio (a coefficienti reali)

P (x, y) = a x2 + b x y + c y2

verificante le proprieta:(i) P (x, y) = 0 soltanto perx = y = 0 ;(ii) sex e y sono numeri interi allora ancheP (x, y) e intero.Determinare poi il massimo della quantita

∆ = b2 − 4a c

al variare diP nell’insieme dei polinomi soddisfacenti le proprieta precedenti.

Testi 13

1991.6 Ci si propone di congiungere con una strada due localita A e B che distano 4 km, fra le quali sitrova una zonaZ costituita da terreno pietroso e avente la forma di un cerchio con centro nel punto mediodi AB e raggio di 1 km.(a) Sapendo che, a parita di lunghezza, il costo di costruzione della strada nella zona pietrosae λ volte (λ

numero reale maggiore di 1) quello relativo alla zona circostante, determinare due puntiP , Q sul bordodi Z in modo che il percorsoAPQB (vedi figura) sia il piu economico possibile.

A O B

PQ

Z

(b) Discutere poi il caso piu generale in cui si considerano percorsi formati, oltre cheda tratti rettilinei,anche da eventuali tratti curvilinei contenuti nel bordo diZ (dove il costo unitario di costruzione si puoconsiderare lo stesso che nella zona esterna aZ).

1992.1 E’ assegnata una legge che a ogni coppia di interix, y associa un interox y in modo che

x (y + z) = y x + z x

per tutti gli interix, y, z. Si dimostri che

x y = x y (1 1).

1992.2 Nel piano due quadratiABCD eA′B′C′D′ sono disposti come in figura.

A

BC

D A ′

B ′

C ′

D ′

Si dimostri che la retta passante perA e perpendicolare aDD′ incontra il segmentoBB′ nel punto medio.

1992.3 Verificare che la somma delle quarte potenze di due numeri reali di assegnato prodottop > 0a) decresce se decresce il valore assoluto della differenzadei due numeri;b) raggiunge il valore minimo quando i due numeri sono uguali.

14 I problemi della Scuola Normale

1992.4 Mostrare che, per ogni intero positivo fissatok, esiste almeno un interon tale che

100≤ nk + n ≤ 101 +k nk−1.

1992.5 SiaF (x) = a3 x3 +a2 x2 +a1 x+a0 un polinomio di terzo grado con coefficienti interi. Si dimostrichei) sep/q, conp, q interi primi tra loro eq 6= 0, e una radice del polinomio, per ogni interom il numero

F (m) e divisibile perp − m q;ii) se esistono due interix1 e x2 tali cheF (x1) = 1, F (x2) = −1, e che|x1 − x2| > 2, alloraF (x) non ha

radici razionali.

1992.61) Dimostrare che, presi comunque tre vertici di un cubo, il triangolo da essi individuatoe rettangolo

oppure equilatero.2) Calcolare la probabilita che tre distinti vertici del cubo, scelti a caso, individuino un triangolo rettangolo

(la probabilita e il rapporto tra il numero di casi favorevoli e il numero di casi possibili).3) Si escludano tre vertici del cubo, e si considerino “ammissibili” i restanti cinque. Si indichi conP

la probabilita che tre vertici ammissibili del cubo, scelti a caso, individuino un triangolo rettangolo.Stabilire quanti valori puo assumereP al variare dei tre vertici esclusi all’inizio.

1993.1 Nel piano cartesiano riferito a coordinate ortogonali, un quadrato striscia in guisa che due suoivertici consecutivi appartengano rispettivamente all’asse delle ascisse non negative e a quello delle ordinatenon negative.Descrivere analiticamente la traiettoria di un puntoP interno al quadrato e rigidamente ancorato ad esso.Caratterizzare le posizioni diP nel quadrato per le quali la traiettoria appartiene ad una circonferenza,oppuree un segmento di retta.

AO x

D

y

PB

C

Testi 15

1993.2 In un piano cartesiano un oggetto puntiforme parte dal punto(0, 2n) (con n intero positivo) escende fino all’asse delle ascisse compiendo 2n passi, con la seguente regola: se prima di compiere unpasso si trova nel punto di coordinate intere (k, l), puo recarsi o in (k−1, l−1) o in (k + 1, l−1) con ugualeprobabilita.Le mosse eseguite nei diversi passi sono indipendenti.Si indichi conpn(k) la probabilita che dopo 2n passi l’oggetto si trovi nel punto (k, 0).i) Calcolarepn(k).ii) Mostrare che 2pn(2) ≥ 1/(2n + 1).

1993.3 Dati tre numeri interip > 2, q > 2, r > 2 si consideri un parallelepipedo di legno tale chei tre spigoli uscenti da un vertice abbiano lunghezzap, q, r. Dopo aver dipinto la superficie esterna delparallelepipedo, questo viene tagliato, mediante sezioniparallele alle facce, in cubetti aventi spigoli dilunghezza 1. Ovviamente alcuni dei cubetti sono parzialmente colorati, mentre altri non sono coloratiaffatto.Si dimostri che esiste solo un numero finito di terne (p, q, r) per ciascuna delle quali il numero dei cubettiparzialmente coloratie uguale al numero di quelli che non sono colorati affatto.

1993.4 Sia data una circonferenzaΓ. Un arco circolareγ congiunge due punti distinti diΓ ede internoal cerchioC racchiuso daΓ. Dimostrare che, se le due regioni in cuiγ divide C hanno aree uguali, lalunghezza diγ supera il diametro diΓ.

1993.5 Sia n un intero positivo pari. Mostrare che si possono trovare (inmodo non necessariamenteunico)n numeri realia1 > a2 > . . . > an > 0 tali che

a1 + a2 + . . . + an = 2n

(a1 − a2) + (a3 − a4) + . . . + (an−1 − an) = n .

Mostrare poi che questo none possibile (pern > 2) se si impone la condizione ulteriore

a1 − an ≤ n .

1993.6 Per ogni intero positivon si indichi conθ(n) il numero reale, compreso fra 0 e 2π, tale che

θ(n) = 3n (mod 2π)

cioe tale cheθ(n) − 3n sia un multiplo intero di 2π.i) Mostrare che 0≤ θ(n) ≤ π/2 per infiniti valori din (si puo usare il fatto che 3.14 < π < 3.15).ii) Mostrare cheθ(n) 6= θ(m) sen 6= m.

1994.1 SiaX un insieme din elementi, doven e un numero pari e siak un intero positivo. Diciamo cheuna funzionef daX in X ha molteplicitak se per ognia ∈ X l’insiemex | f (x) = f (a) hak elementi.Sono di piu le funzioni di molteplicita 1 o quelle di molteplicita 2?

16 I problemi della Scuola Normale

1994.2 Vi sono 4 citta collegate a due a due da 6 strade che non si intersecano (cioe ogni coppia di cittae collegata da una sola strada). Tutte le strade sono aperte al traffico con la stessa probabilita p = 1/2.Determinare la probabilita che in un determinato istante partendo da una qualsiasi citta si possa arrivare adogni altra citta.

1994.3 Mostrare che 41 non puo essere espresso come differenza di una potenza di 2 e di una potenza di3, cioe che non puo sussistere nessuna delle due uguaglianze seguenti:

41 = 2n − 3m , 41 = 3n − 2m

conn, m interi positivi.

1994.4 SiaP un punto interno ad un triangolo equilatero. Per ogni retta passante perP sianoX e Y idue punti di intersezione tra la retta e i lati del triangolo.Determinare, per ogni puntoP , la retta o le retteche rendono minimo il prodotto

PX · PY [1]

1994.5 Consideriamo un triangolo e dividiamo i suoi lati inn parti uguali medianten−1 punti su ciascunlato. Congiungiamo ogni vertice con i punti cosı ottenuti sul lato opposto. Si dimostri che sen e primomaggiore di 2 allora non esistono punti appartenenti simultaneamente a tre dei segmenti cosı costruiti.

1994.6 Sianoa1, a2, . . . , an numeri reali e sianob1, b2, . . . , bn definiti da

bi = max1≤j≤n

(i · j − aj) per ogni i = 1, 2, . . . , n.

Allo stesso modo si costruisconoc1, c2, . . . , cn a partire dab1, b2, . . . , bn e poid1, d2, . . . , dn a partire dac1,c2, . . . , cn; si dimostri che

ci ≤ ai per ognii = 1, 2, . . . , n,di = bi per ognii = 1, 2, . . . , n.

1995.1 SianoC1, C2 due circonferenze di centri e raggi rispettiviO1, O2 e r1, r2. Dato un puntoPesterno alle due circonferenze si considerino le tangenti per P alle due circonferenze e sianoM1, N1 eM2,N2 i rispettivi punti di contatto.Si determini il luogo dei punti tali chePM1

2 + PM22 = 1.

Si determinino i puntiP per cuiPM12 + PM2

2 e minima.

Testi 17

1995.2 Dati quattro punti distinti nel piano dimostrare chee sempre possibile sceglierne tre che deter-minino un angolo inferiore o uguale a 45. In generale datin punti dimostrare che se ne possono scegliere3 che determinino un angolo inferiore o uguale a 180/n.

1995.3 Dimostrare che sea, b, c sono tre interi consecutivi alloraa3 + b3 + c3 e multiplo di 9.

1995.4 Siaf : R → R una funzione non negativa, con concavita rivolta verso il basso, derivabile e talechef ′(0) > 0 ef (x) = f (2− x) per ognix ∈ R.Dimostrare che

∫ 2

0f (x) dx ≤ 2f (1)− [f (1)− f (0)]2

f ′(0)

Provare che se la condizionef (x) = f (2− x) none verificata, tale diseguaglianza puo non valere.

1995.5 Dato A =(x, y) ∈ R

2 : |x|2a + |y|2a ≤ 1

ove a e un numero reale positivo, si consideri alvariare dik ≥ 0 la classeC degli insiemi

Bk =(x, y) ∈ R

2 : |x| + |y| ≤ k

contenuti inA. Determinarek in funzione dia in modo cheBk abbia area massima tra gli insiemi diC .

1995.6 Sianoa, b numeri reali non negativi tali cheb2 + b6 ≤ a2 − a6. Dimostrare che allora risulta:1) a ≤ 1;2) b < 2/3.

1996.1 Dato un quadratoABCD di lato unitario, determinare la massima costanteα e la minima costanteβ per cui si ha

α ≤ PA + PB + PC + PD ≤ β

per ogni puntoP contenuto nel quadrato.

1996.2 Il prezzo di mercatoP di una certa merce dipende dalla quantita totaleQ venduta secondo laleggeP = a − b Q, dovea e b sono due assegnati valori positivi.Sul mercato operano solo due produttori, in concorrenza fraloro.A regime, cioe quando nessuno dei due ha interesse a cambiare la quantita di merce da lui venduta, i dueproduttori vendono rispettivamente le quantita X e Y di merce. Supponendo che la produzione avvenga acosto zero, determinareX eY .

18 I problemi della Scuola Normale

1996.3 Sia P un poliedro e sianoF il numero delle facce,S il numero degli spigoli eV il numerodei vertici di P. Sapendo che per il poliedroP, per la nota formula di Eulero, vale la relazioneF − S + V = 2,(a) provare cheP ha qualche faccia con meno di 6 lati;(b) dettok il numero delle facce con meno di 6 lati, determinare il minimo valore possibile perk.

1996.4 Siaf (t) una funzione iniettiva definita sui numeri reali positivi.Dati x > 0 ey > 0, chiamiamof -Media dix e y l’unico numeroz tale che

f (z) =f (x) + f (y)

2.

Mostrare che la media geometrica√

x y e quella armonica 2x y/(x + y) sono dellef -Medie.Fra le funzioni convessef , individuare quelle per le quali laf -Media risulta minore o uguale della mediaaritmetica.

1996.5 Dato un triangolo nel piano euclideo si indichi conI il centro della circonferenza in esso inscrittae conΓ la circonferenza passante perI e per due qualunque dei vertici del triangolo.Provare che il centro diΓ si trova sulla circonferenza circoscritta al triangolo.

1996.6 A partire da un cerchioC1 tracciare successivamente: un triangolo equilateroP1 inscritto inC1, ilcerchioC2 inscritto inP1, un quadratoP2 inscritto inC2, il cerchioC3 inscritto inP2, un pentagono regolareP3 inscritto inC3, e cosı via, ottenendo in tal modo una successione

C1 ⊃ P1 ⊃ C2 ⊃ P2 ⊃ . . . Cn ⊃ Pn ⊃ Cn+1 ⊃ Pn+1 . . .

di cerchi e poligoni regolari concentrici, dovePn han + 2 lati.Mostrare che l’intersezione di tutti i cerchiCn e un cerchio di raggio positivo. Il candidato puo ricorrerealla diseguaglianza, valida per ogni interok ≥ 1:

1(k + 1)2

+1

(k + 2)2+

1(k + 3)2

+ . . . <1k

.

Testi 19

1997.1 Un cuboe appoggiato su un piano. Un bambino lo muoven volte, facendolo rotolare (senzastrisciare) ogni volta su uno dei lati della faccia su cuie appoggiato. Si suppone che la prima mossa siacasuale e che, ad ogni mossa successiva, il bambino scelga casualmente di far rotolare il cubo su uno deidue lati contigui al lato scelto in precedenza (vedi Figura:dopo aver rotolato sul latoa, il cubo rotolera sullato b oppure sul latoc).

a

b

c

a) Dimostrare che se il cuboe tornato nella posizione iniziale (non necessariamente appoggiato sulla stessafaccia) alloran e divisibile per 4.

b) Calcolare la probabilita p(n) che il cubo sia tornato nella posizione iniziale.c) Dimostrare che

516k

− 116k2

≤ p(4k) ≤ 13k

− 112k2

, k ≥ 1 .

1997.2 Si dice che un puntoP esterno ad una circonferenzaC “vede” la circonferenza sotto un angoloα se l’angolo (contenenteC) compreso fra le tangenti aC condotte daP e uguale aα.a) Data una circonferenzaC e un angoloA di ampiezzaα > 0, costruire il luogo dei punti del piano che

vedonoC sotto l’angoloα.b) Date due circonferenzeC e C′ esterne l’una all’altra, di centriO, O′ e raggir, r′ rispettivamente,

costruire il luogoL dei punti del piano che vedono le due circonferenze sotto lo stesso angolo.c) Dire (in termini dei dati) in che intervallo varia l’angolo di visuale al variare diP in L e quali sono i

punti inL dove tale angoloe minimo e massimo.

1997.3 Si determini, al variare dei parametriα e β interi pari e positivi, il numero di soluzioni realidell’equazione

(α + β) x sin(π x) = x2 + α β .

20 I problemi della Scuola Normale

1997.4 Sia dato un insieme finitoΩ di punti distinti del piano tra loro collegati da un certo numero dipercorsi elementaricongiungenti coppie di vertici distinti come esemplificatonel disegno seguente:

Dati due puntiA e B di Ω un cammino che parte daA e termina inB e una successione di verticiv0, v1, v2, . . . , vn di punti di Ω tali chev0 = A, vn = B, e tale chevi e vi+1 sono congiunti da un percorsoelementare; in questo caso si dice chen e la lunghezzadel cammino.I punti, i percorsi elementari e i cammini soddisfano le seguenti proprieta:i) i percorsi elementari non si incontrano fuori dai punti diΩ;ii) dati due qualsiasi puntiA, B ∈ Ω, esiste almeno un cammino che parte daA e termina inB;iii) c’ e un particolare puntoX ∈ Ω per il quale esiste un cammino che parte e termina inX avente lunghezza

dispari.Si dimostri allora che esisteN intero positivo tale che, scelti due qualsiasi puntiA e B di Ω, esiste uncammino di lunghezzaN che parte daA e termina inB.

1997.5 In ognuna delle tre figure seguentie disegnato un poligonoP . Si immagini che, in ognuno deitre casi, il poligonoP sia costituito di materiale elastico e flessibile. Si chiededi disegnare o descriveresinteticamente la figuraS che, in ognuno dei tre casi, si ottiene facendo combaciare lefrecce dei lati chehanno la stessa lettera, e di illustrare come questi lati appaiono inS.

P P P

a

a

b b c a

d ba

d bc a

a

b c

c b

1997.6 Si determinino tutti gli interi positivin che sono divisibili per tutti gli interi positivi minori ouguali a

√n.

(Suggerimento: considerare il minimo comune multiplo dei numeri minori o uguali a√

n).

Anno 1906 21

1906.1* Sono dati, in un piano, un circolo e due punti. Tirare una tangente al circolo in modo che lasomma delle distanze della tangente dai due punti dati sia uguale a una lunghezza data.

RISOLUZIONE Indichiamo conA, B i due punti dati, conγ il cerchio e cond la lunghezza assegnata.Procediamo nel modo seguente: dapprima caratterizziamo leretter per cui risulta

dist(A, r) + dist(B, r) = d ; [1]

successivamente vediamo se fra queste rette vi sono delle tangenti al cerchioγ.E’ utile a questo punto, dettoC il punto medio diAB, costruire il cerchioγ′ di centroC e raggiod/2 (fig. 1).

γ

γ ′A

C

B

r

Figura 1

Ser e una retta per cui vale la [1], si presentano due possibilita: r non intersecaAB oppurer intersecaAB.Caso (a) Ser non intersecaAB, per il teorema di Talete la distanza diC da r e pari alla semisommadelle distanze diA e B da r, cioe dist(C, r) = d/2. Di conseguenza, la rettar risulta tangente al cerchioγ′. Cosı un primo gruppo di soluzioni del problema si otterra considerando tutte le tangenti comuni ai duecerchiγ e γ′ che non intersecanoAB. E noto che, a seconda della posizione reciproca dei due cerchi, vipossono essere 4, 3, 2, 1 oppure nessuna tangente comune.

Caso (b) Ser e una retta verificante la [1] che intersecaAB, tracciamo le due retter′ e r′′, parallele ar,che passano rispettivamente perA e perB (fig. 2). Si ha allora

dist(r′, r′′) = dist(A, r) + dist(r, B) = d .

Dunquer′ e r′′ sono entrambe tangenti aγ′ (in particolareA e B sono necessariamente esterni a talecerchio).Costruito il romboAHBK circoscritto aγ′, un altro gruppo di soluzioni del nostro problema sara alloracostituito da quelle rette, tangenti aγ e parallele a uno dei lati del romboAHBK, che intersecano il rombostesso. Evidentemente tali rette non possono essere piu di quattro.

22 I problemi della Scuola Normale

γ

γ ′A

C

K

H

Br ′′

r

r ′

Figura 2

Dal momento che i due gruppi di rette indicate comprendono tutte le possibili soluzioni del problema,possiamo concludere che vi sono al piu otto soluzioni.Qualora siaAB < d, il problema si semplifica notevolmente, in quanto il caso (b) non si puo verificare, ele uniche soluzioni possibili sono le tangenti comuni aγ eγ′ che non intersecanoAB.

H

K

A BC

γ0γ ′

γ8

Figura 3

OSSERVAZIONE Non e facile assegnare condizioni esplicite suA, B, γ ed che assicurino la risolubilita delproblema o che ne indichino, piu in generale, il numero di soluzioni. Si puo pero costruire, per ogni interoncompreso fra 0 e 8, un esempio in cui il problema ha esattamenten soluzioni. I casi estremi (zero soluzioniper il cerchioγ0, otto soluzioni per il cerchioγ8) sono indicati nella Figura 3.

Anno 1909 23

1909.1 In un piano sono dati due circoli che si tagliano in due punti.Si domanda di condurre per unodi questi punti una retta sulla quale le corde staccate dai due circoli stiano fra loro in un rapporto dato.

RISOLUZIONE Indichiamo conO1 e O2 i centri dei due cerchi, conP e P ′ le loro due intersezioni e conkil rapporto assegnato. Siar una retta passante perP che verifica la proprieta richiesta, tale cioe che, detteA1 eA2 le sue ulteriori intersezioni con i due cerchi (fig. 1), risulti A1P = k · A2P .Detti M1 eM2 i punti medi delle cordeA1P eA2P , si ha allora

M1P = k · M2P .

Osserviamo che i segmentiM1O1 e M2O2 sono fra loro paralleli perche entrambi ortogonali ar. CondottaperP la perpendicolares ar, seA1 eA2 non coincidono conP ′ la retta perO1 eO2 intersecas in un puntoQ, e per il teorema di Talete

O1Q = k · O2Q . [1]

O1 Q O2

A1

A2

M1

M2

P

P ′

r

Figura 1

Viceversa, seQ e un punto della retta perO1 e O2 che verifica la [1],e sufficiente condurre perP la rettaperpendicolare al segmentoPQ; le corde staccate dai due cerchi su tale retta stanno fra loro nel rapportok.Osserviamo che, sek 6= 1, vi sono due punti sulla retta perO1 e O2 che verificano la [1]; pertanto vi sonodue rette perP che risolvono il problema (fig. 1 e fig. 2).

O1 O2 Q

P

P ′

r

Figura 2

24 I problemi della Scuola Normale

Se k = 1, vi sono ancora due soluzioni: unae la retta perP e P ′ (quindi A1 = A2 = P ′), l’altra e laperpendicolare inP al segmento che congiungeP con il punto medio diO1O2 (vedi Figura 3).

O1 O2

P

P ′

r

O1 O2Q

P

r

Figura 3

Anno 1911 25

1909.2* Dimostrare che sea1, a2, . . . , an sonon numeri diversi da zero, fra i quali sussiste la relazione(a 2

1 + a 22 + · · · + a 2

n−1

) (a 2

2 + a 23 + · · · + a 2

n

)=

(a1 a2 + a2 a3 + · · · + an−1 an)2,

i numeri stessi sono necessariamente in progressione geometrica.

RISOLUZIONE L’espressione

∆ = (a1 a2 + a2 a3 + · · · + an−1 an)2 −(a 2

1 + a 22 + · · · + a 2

n−1

) (a 2

2 + a 23 + · · · + a 2

n

)

e il discriminante (ridotto) dell’equazione di secondo grado:(a 2

1 + a 22 + · · · + a 2

n−1

)x 2 −

2 (a1 a2 + a2 a3 + · · · + an−1 an) x +(a 2

2 + a 23 + · · · + a 2

n

)= 0 .

[1]

Riordinando i termini l’equazione [1] si puo porre nella forma(a 2

1 x2 − 2a1 a2 x + a 22

)+

(a 2

2 x2 − 2a2 a3 x + a 23

)+ · · ·+

(a 2

n−1 x2 − 2an−1 an x + any 2) = 0

o anche

(a1 x − a2)2 + (a2 x − a3)2 + · · · + (an−1 x − an)2 = 0 .

Poiche il discriminantee nullo, la [1] ha un’unica soluzionex0; quindi risulta:

(a1 x0 − a2)2 + (a2 x0 − a3)2 + · · · + (an−1 x0 − an)2 = 0 .

Una somma di termini non negativi, pero, si annulla solo se ciascun terminee nullo; pertanto

a2 = x0 a1 , a3 = x0 a2 , · · · an = x0 an−1 ;

cioe i numeria1, a2, . . . , an costituiscono una progressione geometrica di ragionex0.

1911.1 Dividere un dato arco di cerchio in due archi le cui corde siano tra loro in un rapporto dato.

RISOLUZIONE Il problema si puo risolvere utilizzando il teorema della bisettrice: la bisettricecondotta da un vertice di un triangolo divide il lato oppostoin parti proporzionali ai lati adiacenti.Sia

_AB l’arco assegnato e siaP il punto che divide il segmentoAB in due partiAP e PB che stanno fra

loro nel rapporto dato. Indichiamo conM il punto medio di_

AB e conQ il punto opposto aM rispetto alcentro del cerchio (fig. 1). Il puntoC, ottenuto intersecando l’arco

_AB con la retta perP e Q, risolve il

problema: infatti la retta perC e Q e la bisettrice dell’angoloACB, dal momento che i due archi_AQ e

_QB sono uguali per costruzione, e che quindi anche i rispettiviangoli alla circonferenzaACQ eQCB sonouguali.

OSSERVAZIONE Dato un segmentoAB, il luogo dei puntiP tali cheAP ePB stanno in un rapporto datoeuna circonferenza, dettacirconferenza di Apollonio, di diametroP1P2, doveP1 e P2 sono i duepunti della retta perA eB, l’uno interno adAB e l’altro esterno, che hanno distanze daA eB nel rapportodato.Pertanto il puntoC richiesto si puo ottenere anche intersecando l’arco

_AB con la circonferenza di Apollonio

relativa adAB e al rapporto dato (fig. 2).

26 I problemi della Scuola Normale

Q

M

A B

C

P

P2 P1A B

C

Figure 1, 2

1912.2* In un dato quadrilatero inscrivere un parallelogrammo il cui centro cada in un punto dato.

RISOLUZIONE Supporremo che il quadrilatero assegnatoABCD risulti convesso e che il puntoP sia internoad esso. Diremo che un parallelogrammoe inscritto in un quadrilatero se ha un vertice su ogni lato delquadrilatero.SiaHKLM un parallelogrammo che risolve il problema, e dunque ha il centro in P e i vertici H, K, L,M rispettivamente sui latiAB, BC, CD, DA. Poiche le diagonali di un parallelogrammo si bisecano, perindividuare i puntiH eL occorre tracciare perP una retta che intersechiAB eCD in due punti equidistantidaP .Se i latiAB eCD sono paralleli, il problema non ha soluzione, a meno cheP non giaccia sulla retta parallelae equidistante daAB eCD, nel qual caso ogni retta perP che intersechi i latiAB eCD determina puntiHeL accettabili.SeAB e CD non sono paralleli, i loro prolungamenti si incontreranno in un puntoO (fig. 1). DettoO′ ilsimmetrico diO rispetto aP , i punti H e L sono necessariamente l’intersezione diAB con la parallela aCD passante perO′ e l’intersezione diCD con la parallela adAB passante perO′.Allo stesso modo si caratterizzano i puntiK eM .

AB

C

D

H

K

L

MP

O

O ′

Figura 1

DISCUSSIONE La costruzione descritta sopra consente, quandoe possibile, di determinare il parallelo-grammo di centroP inscritto inABCD; occorre ora determinare in quali casi tale parallelogrammo esisteeffettivamente.

Anno 1912 27

Supponiamo dapprima che i latiAB e CD non siano paralleli; indichiamo conr, s le due semirette,concorrenti inO, che contengonoAB e CD (fig. 2). Per ogni puntoP interno adABCD passa una e unasola rettat che intersecar e s in due punti,H e L, simmetrici rispetto aP ; per poter risolvere il problemaassegnato occorre che la rettat tagli r e s all’interno dei segmentiAB e CD. La costruzione della rettat,precedentemente descritta, mostra che cio si verifica se e soltanto se il puntoO′, simmetrico diO rispetto aP , cade all’interno del parallelogrammo delimitato dalle parallele as passanti perA, B e dalle parallele arperC, D (vedi fig. 2, ove tale parallelogrammoe ombreggiato).

A

B

C

D

H

L

O O ′D ′

B ′

Z

WP

t

r

s

Figura 2

Cio equivale, comee facile verificare, a dire che il puntoP e interno al parallelogrammoS1 = D′ZB′W ,oveD′ eB′ sono i punti medi dei latiAD eBC, mentreZ eW sono i punti medi delle diagonaliAC eBD.Nel caso speciale in cui i latiAB e CD sono paralleli,S1 si riduce al segmentoB′D′, e la condizione diesistenza diH eL e ancora cheP appartenga aS1.

In modo analogo si puo vedere che l’esistenza di due puntiK eM suBC eAD, simmetrici rispetto aP , eequivalente all’appartenenza diP al parallelogrammoS2 = A′ZC′W , oveA′ eC′ sono i punti medi diABeCD.

In conclusione, condizione necessaria e sufficiente affinche il problema dato abbia soluzionee che il puntoP appartenga al parallelogrammoS ottenuto intersecandoS1 eS2 (vedi fig. 3, in cui tale parallelogrammoeombreggiato).

A

B

C

D

A ′

B ′

C ′

D ′

Z

W

Figura 3

Si noti cheS contiene sempre almeno il segmentoZW , e degenera in tale segmento quandoABCD e untrapezio (se poiABCD e un parallelogrammo,ZW si riduce ad un punto).All’infuori dei casi degeneri, la soluzionee unica.

28 I problemi della Scuola Normale

1915.2* Dimostrare che se per un punto interno ad una sfera si conducono tre piani, a due a dueperpendicolari, le somme delle aree dei tre cerchi che essi determinano sulla sferae costante.

RISOLUZIONE Indichiamo conO e r il centro e il raggio della sfera; siaγ un cerchio giacente su di essa esiaC il centro diγ. Usando il teorema di Pitagora si vede che il raggio diγ e uguale a

√r2 − OC2,

e quindi l’area diγ e uguale aπ (r2 − OC2).Sia oraP un punto interno alla sfera e sianoα1, α2, α3 tre piani passanti perP e a due a due perpendicolari.Indichiamo conγ1, γ2, γ3 i tre cerchi ottenuti intersecando la sfera conα1, α2, α3 e conC1, C2, C3 i rispettivicentri. Si ha allora:

area(γ1) + area(γ2) + area(γ3) =

π[3r2 −

(OC1

2 + OC22 + OC3

2)] .[1]

Supponiamo dapprima che i tre puntiC1, C2, C3 siano distinti daO. Dato che il piano passante perO, C2,C3 e parallelo al pianoα1 (e analogamente perα2, α3), il parallelepipedo di spigoliOC1, OC2, OC3 e unparallelepipedo rettangolo avente come vertice opposto adO il punto P . Si ha dunque:

OP 2 = OC12 + OC2

2 + OC32 . [2]

α3

α2

α1

γ3

γ2

γ1

P

OC3

C1

C2

Figura 1

E facile verificare che la [2]e ancora valida quando qualcuno dei tre puntiC1, C2, C3 coincide conO.Dalla [1] e dalla [2] segue:

area(γ1) + area(γ2) + area(γ3) = π(3r2 − OP 2) .

Questa uguaglianza mostra che la somma delle aree dei tre cerchi γ1, γ2, γ3 dipende solo daP , e non dallascelta della terna di pianiα1, α2, α3.

Anno 1926 29

1923.2* Dati due punti sopra le facce di un diedro, quale la spezzata minima che li unisce e che ha unvertice sullo spigolo?

RISOLUZIONE Sianoα eβ i due semipiani che costituiscono il diedro,A eB i punti dati, conA appartenenteadα e B a β, r lo spigolo del diedro eM il vertice della spezzata che appartiene allo spigolo (fig. 1). Laspezzata minima richiesta deve consistere necessariamente dei due soli segmentiAM eMB; si deve quindideterminareM sur in modo che la sommaAM + MB sia minima.Siaβ′ il semipiano opposto aβ e si consideri la rotazione che manda il semipianoα suβ′; essa manda ilpuntoA in un puntoA′ di β′ e, per ogni puntoM di r, il segmentoAM nel segmentoA′M . Risulta quindiAM = A′M e dunqueAM + MB = A′M + MB ed e chiaro per la disuguaglianza triangolare

che quest’ultima somma raggiunge il suo minimo valore quando A′, M eB sono allineati.Dunque la spezzata cercatae quella di verticiA, N , B, oveN e il punto di intersezione dir con il segmentoA′B.

β ′

α

βr

A ′

A

B

M

N

Figura 1

1926.2* Condurre un piano che seghi un dato angolo tetraedro in un parallelogrammo.

RISOLUZIONE SianoO il vertice er1, r2, r3, r4 gli spigoli dell’angolo dato. Siaαij il piano passante perri

erj , i, j = 1, 2, 3, 4. Si puo supporre cheα13 lascir2 e r4 in semispazi distinti. Sias l’intersezione dei pianiα13 e α24: proviamo che per ogni puntoQ di s esiste un pianoα che lo contiene e tale che l’intersezionecon l’angolo dato sia un parallelogrammo.

Per questo, basta costruire una rettar di α24 contenenteQ, una rettat di α13 contenenteQ in modo cheQsia il punto medio dei segmenti staccati sur e t rispettivamente dalle retter1 er3 e dalle retter2 er4 (fig. 1):il piano perr e t avra allora la proprieta richiesta (infatti un quadrilatero le cui diagonali si intersecano neiloro punti medie un parallelogrammo).Siamo ricondotti cosı al seguente problema: date due retter′ e r′′ uscenti da un puntoO e un puntoQ fuoridi esse, costruire una rettas passante perQ e tale cheQ sia il punto medio del segmento staccato dar′ e r′′

sus.Per risolvere questo problema (si veda anche1912.2) basta costruire il puntoR simmetrico diO rispetto aQ (fig. 2); le parallele condotte daR alle due rette determinano assieme ad esse un parallelogrammo di cuiuna diagonaleeOR e l’altra giace sulla retta cercata.

30 I problemi della Scuola Normale

O

O

Q

r2

r1

s r3r4

Q

Rt

s

r ′

r ′′

s

Figura 1

1927.2 Determinare il luogo dei centri delle circonferenze ottenute segando una superficie sferica coni piani passanti per un punto qualunque dello spazio.

RISOLUZIONE SianoO il centro della sfera data,P il punto assegnato (che supporremo distinto daO) eP0

il punto medio del segmentoOP (fig. 1).SeQ e un punto del luogo cercato (dunqueQ e centro del cerchio sezione della sfera con un pianoα passanteper P ), il segmentoOQ e perpendicolare al pianoα. Inoltre il triangolo rettangoloPQO e inscritto nelcerchio di centroP0 e raggioOP0 che giace nel piano passante perO eP e perpendicolare adα. Ne seguecheP0Q = P0P , e allora ogni puntoQ del luogo cercatoe interno alla sfera data e appartiene alla superficiesfericaS di centroP0 e raggioP0P .Viceversa, seQ e interno alla sfera data e appartiene aS, il triangolo PQO e rettangolo inQ e quindi ilpuntoQ e il centro del cerchio ottenuto tagliando la sfera data con il piano passante perP eQ e ortogonaleaOQ.

Dunque il luogo cercatoe costituito dalla parte della superfie sfericaS di centroP0 e raggioP0P che einterna alla sfera data. Si noti che se il puntoP e interno alla sfera data, detto luogoe costituito dall’interasuperficie sfericaS.

α

P

P0

O

Q

S

Figura 1

Anno 1929 31

1929.2* Verificare che il luogo dei puntiM tali che la somma dei quadrati delle loro distanze da trepunti datiA, B, C sia uguale ad una assegnata costantek2 e un cerchio.

RISOLUZIONE Facendo uso di un sistema di coordinate cartesiane ortogonali la verifica e immediata.Un altro modo di risolvere il problema parte dall’osservazione che per valori dik troppo piccoli nessunpunto verifica la condizione richiesta ed il luogoe quindi vuoto. Cerchiamo di determinarek0, il piu piccolovalore dik per cui cio non accade, e supponiamo cheO sia un punto del piano per cuiOA2+OB2+OC2 = k2

0;per ogni altro puntoM risulteraMA2 + MB2 + MC2 ≥ OA2 + OB2 + OC2. Se perP eQ generici punti del

piano si indica con−−→PQ il vettore applicato inP con estremo liberoQ abbiamo, per ogni puntoM :

−−→MA =

−−→MO +

−→OA ,

−−→MB =

−−→MO +

−−→OB ,

−−→MC =

−−→MO +

−−→OC ,

da cui seguono le uguaglianze:

MA2 = MO2 + OA2 + 2−−→MO ·

−→OA ,

MB2 = MO2 + OB2 + 2−−→MO ·

−−→OB ,

MC2 = MO2 + OC2 + 2−−→MO ·

−−→OC

( il punto · indica il prodotto scalare di due vettori ), e quindi(MA2+ MB2 + MC2) −

(OA2 + OB2 + OC2) =

3MO2 + 2−−→MO · (

−→OA +

−−→OB +

−−→OC ) .

[1]

Il secondo membro della [1]e sempre maggiore o uguale a 0 per qualunque scelta diM se scegliamo come

O il punto individuato dalla equazione−→OA +

−−→OB +

−−→OC = 0, chee quindi il baricentro del triangolo

ABC. Si ha dunquek20 = OA2 +OB2 +OC2; per ognik maggiore o uguale ak0 il luogo richiesto puo essere

descritto da

MO2 =13

[k2 −

(OA2 + OB2 + OC2)] ,

ede quindi il cerchio di centro inO e raggior dato da

r =

√13

[k2 −

(OA2 + OB2 + OC2)] .

OSSERVAZIONE La proprieta enunciata vale anche nel caso in cui si ha un numero finito di punti A1, A2,. . . , An in luogo dei tre puntiA, B, C. La formula [1] diviene allora

(MA1

2+ · · · + MAn2) −

(OA1

2 + · · · + OAn2) =

n MO2 + 2−−→MO · (

−−→OA1 + · · · +

−−→OAn ) ,

e ancora il puntoO che rende minima la quantitaMA12 + · · ·+ MAn

2 e quello individuato dalla condizione−−→OA1 + · · · +

−−→OAn = 0, cioe il baricentro del sistema di puntiA1, . . . , An.

32 I problemi della Scuola Normale

1939 E dato un rettangoloABCD. Facendolo ruotare attorno al latoAB si ottiene un primo cilindroe facendolo ruotare attorno al latoBC si ottiene un secondo cilindro. Si conosce la somma4π S2 dellesuperfici laterali dei due cilindri; e pure si conosce la somma π V 3 dei volumi degli stessi. Si domanda dideterminare i lati del rettangoloABCD.Qual e la condizione di realta? Quante soluzioni ha il problema? FissatoV , qual e il massimo valorepossibile perS?

RISOLUZIONE Indichiamo conx ey rispettivamente il lato minore e quello maggiore diABCD; le superficilaterali dei due cilindri (fig. 1) sono uguali a 2π x y e 2π y x, mentre i corrispondenti volumi sono dati daπ x2 y eπ y2 x.

B

C D

A

x

y

Figura 1

Anno 1943 33

Si ha allora 4π S2 = 4π x y e π V 3 = π (x2 y + y2 x), cosicche il problema si riduce a risolvere il sistemasimmetrico:

x y = S2

x + y =V 3

S2

[1]

con la condizione 0< x ≤ y.Le soluzioni del sistema [1] sono le radici positive dell’equazione di secondo grado

t2 − V 3

S2t + S2 = 0 [2]

e tale equazione ha radici reali se e soltanto se (V 3/S2)2 − 4S2 ≥ 0, cioeV 3/S2 ≥ 2S, ovvero se risulta

V ≥ 3√

2S . [3]

Se la condizione [3]e soddisfatta, le radici della [2] sono positive, essendo positivi la loro somma e il loroprodotto.

Pertanto si puo concludere che, seV ≥ 3√

2S, il problema ammette la sola soluzione

x =1

2S2

(V 3 −

√V 6 − 4S6

),

y =1

2S2

(V 3 +

√V 6 − 4S6

),

Dalla condizione [3] si vede poi facilmente che, fissatoV , il massimo valore possibile diS per cui ilproblema ha soluzionie

Smax =V3√

2.

In corrispondenza di tale valore diS, il rettangoloABCD e un quadrato.

1943 Determinare un triangolo rettangolo di cui siano noti il perimetro e il raggio del cerchio inscritto.Una volta fissato il raggio del cerchio inscritto, quale il valore minimo del perimetro?

RISOLUZIONE Indichiamo conr il raggio del cerchio inscritto e con 2p il perimetro del triangolo cercato econsideriamo due semirettem en fra loro perpendicolari uscenti da un puntoA. SianoM eP due punti sum, N eQ due punti sun tali che

AN = AM = r , AP = AQ = p ;

conduciamo le circonferenzeC eC′ tangenti am en rispettivamente nei puntiM , N eP , Q; siaγ il minore

degli archi che i puntiP , Q individuano suC′ (fig. 1).Se t e una tangente comune aC e aγ, essa individua sulle semirettem e n due puntiB e C tali che iltriangoloABC risolve il problema. InfattiC e la circonferenza inscritta in tale triangolo e, dall’uguaglianzadei segmentiBR, BP e CR, CQ (oveR e il punto di tangenza frat e γ), si ha che il perimetro diABC e2p.Il problema pertanto avra soluzione se l’arcoγ e esterno alla circonferenzaC oppuree tangente ad essa;poiche i centri diC eC

′ stanno sulla bisettrice dimn, cio avviene se e solamente se

r + r√

2 ≤ p√

2− p ,

che equivale ap ≥ r (3 + 2√

2).

34 I problemi della Scuola Normale

Il valore minimo del perimetro 2p e dunque 2r (3 + 2√

2); in tal casoC e γ risultano tangenti fra di loro eil triangoloABC e isoscele.Si noti che tale costruzione si puo effettuare per ogni scelta dell’angolo inA e quindi consente di costruireun triangolo, dati un angolo, il perimetro e il raggio del cerchio inscritto.

N

C

Q

n

A M B P m

R

γ

C

Figura 1

1960.3 E piu facile, gettando una volta un dado, ottenere 6, oppure, gettandolo tre volte, ottenere tuttee tre le volte un numero pari?

RISOLUZIONE Supporremo che a ogni lancio ciascuna faccia del dado abbia la stessa probabilita di uscire.La probabilita che in un lancio esca un determinato numeroe quindi 1/6, perche vi sono 6 risultatipossibili equiprobabili. Dunque la probabilita che al primo lancio esca il numero 6e 1/6, mentre laprobabilita che esca un numero parie 1/2 in quanto a tale risultato si perviene in 3 modi distinti sui6risultati possibili. Poiche e ragionevole supporre che risultati di lanci successivi siano tra loro indipendenti,si ottiene che la probabilita che in tre lanci esca sempre un numero parie

12· 1

2· 1

2=

18

.

E quindi piu probabile ottenere 6 (o una qualsivoglia determinata faccia del dado) in un solo lancio cheottenere un numero pari per 3 lanci consecutivi.

Anno 1962 35

1961.4* Con una bilancia a piatti e un certo numero di pesi, si vogliono pesare oggetti di peso inferiorea 500grammi, con un errore non superiore ad un grammo. Non si possono mettere pesi nel piatto su cuipoggia l’oggetto. Dire quale il numero minimo di pesi sufficienti a tale scopo.

RISOLUZIONE Poiche ci accontentiamo di un errore non superiore ad un grammo,e sufficiente scegliere ipesi in modo da poter esprimere, con loro combinazioni, tutti i pesi corrispondenti ai numeri pari da 2 a498.E facile convincersi che a tale scopo sono sufficienti 8 pesi;per esempio i pesi da

2, 4, 8, 16, 32, 64, 128, 256 grammi.

Inoltre 8e il numero minimo: infatti, disponendo dik pesi, se ne possono scegliereh (h ≤ k) in(

kh

)modi

diversi; quindi conk pesi si possono ottenere al piu

p(k) =k∑

h=1

(k

h

)

valori distinti. Ricordando la formula delbinomio di Newton, abbiamo

k∑

h=0

(k

h

)= (1 + 1)k = 2k

e quindip(k) = 2k − 1.Sek ≤ 7 si hap(k) ≤ 127; siccome gli oggetti da pesare hanno peso compreso fra 0 e500 grammi e l’erroremassimo consentito (per difetto o per eccesso)e di un grammo, qualora bastassero 7 pesi occorrerebbepoter ricoprire un intervallo di lunghezza 500 con non piu di 127 intervalli di lunghezza al piu 2. Cio eovviamente impossibile, in quanto 2· 127< 500.

1962.5* Si sostiene talvolta che noi usiamo il sistema decimale di numerazione (per cui, ad esempio,362significa3 · 102 + 6 · 10 + 2) in quanto abbiamo10 dita.Un marziano, dopo aver visto scritta l’equazione:

x2 − 16x + 41 = 0,

invitato a scrivere la differenza delle radici scrive10.Quante dita hanno i marziani?(N.B. Per i numeri compresi fra0 e6 la scrittura dei marziani coincide con la nostra).

RISOLUZIONE Supponiamo che i marziani abbianoN dita e quindi usino il sistema di numerazione in baseN . Possiamo subito osservare cheN > 6, in quanto i simboli 0, 1, 2, 3, 4, 5 e 6 hanno lo stesso significatoper noi e per i marziani. Inoltre i simboli 10, 16 e 41 che per noi indicano rispettivamente il numero dellenostre dita, il numero delle nostre dita aumentato di 6, e quattro volte il numero delle nostre dita aumentatodi 1 indicheranno per i marziani rispettivamente i numeri

N , N + 6, 4N + 1 .

D’altra parte l’algoritmo risolutivo delle equazioni di secondo grado (come tutte le regole algebriche)eindipendente dal sistema di numerazione prescelto; pertanto le due radici dell’equazione

x2 + b x + c = 0

hanno in ogni caso differenza uguale a

δ =√

b2 − 4c .

Nel nostro caso dovra dunque essereN =√

(N + 6)2 − 4 (4N + 1), da cui si ricava

36 I problemi della Scuola Normale

N = 8.

I marziani hanno dunque 8 dita (anche se, invitati a scrivereil numero delle loro dita, scriverebbero “10”).

1963.5 Un podista si trova su un punto della Terra, che supponiamo perfettamente sferica. Percorreun chilometro verso nord, poi uno verso est e infine uno verso sud. Si ritrova al punto di partenza.Quali sono i punti di partenza che obbediscono a questa condizione?

RISOLUZIONE Si osserva subito che il polo sud obbedisce alla condizione richiesta.Sia P un altro di tali punti. Per prima cosa, esso deve distare piu di un chilometro dal polo nord e, inparticolare, non puo essere il polo nord.Indichiamo conA il punto un chilometro a nord diP , conB il punto un chilometro ad est diA, cosicche ilpercorso del podista avviene lungo la “spezzata”PABP .I tratti PA e BP giacciono sul meridiano passante perP (fig. 1), hanno uguale lunghezza e sono percorsiin verso opposto; pertantoA coincide conB e quindi il percorsoAB e un cammino chiuso, consistente inun intero parallelo (percorso una o piu volte).Si conclude allora che, oltre al polo sud, i soli punti che verificano la condizione richiesta sono i punti chegiacciono sui paralleliCp tali che il parallelo un chilometro a nord diCp abbia lunghezza di 1/p chilometri,perp = 1, 2, . . . .

PA=B

Figura 1

Anno 1967 37

1967.2* Sono assegnate tre rette parallele. Esiste un triangolo equilatero con i vertici rispettivamentesulle tre rette?

RISOLUZIONE Indichiamo le tre rette cona, b, c, supponendo chea sia interna alla striscia delimitatadalle altre due. Per dare risposta (affermativa) al quesito, utilizzeremo l’osservazione che un triangoloeequilatero se e solo se una rotazione di 60 gradi attorno ad un vertice trasforma uno degli altri due verticinel terzo.Fissiamo dunque, in modo arbitrario, un puntoB sulla rettab ed eseguiamo una rotazione di 60 attornoa B, indicando cona′ la trasformata della rettaa e conC l’intersezione fraa′ e c. Dato cheC appartienealla rettaa′, essoe il trasformato, tramite la rotazione considerata, di un punto A appartenente ada. PerdeterminareA basta intersecarea con il cerchio di centroB e raggioBC, scegliendo delle due intersezioniquella che sta dalla stessa parte diB rispetto ada′ (fig. 1). Il triangoloABC risolve il problema.

Poiche le rotazioni di 60 che si possono eseguire attorno aB sono due, a seconda del verso scelto, i triangoliequilateri con un vertice inB, un vertice sulla rettaa e il terzo vertice sulla rettac sono due. Essi sonosimmetrici fra loro rispetto alla perpendicolare alla retta b in B. Nel caso speciale in cuia e la bisettricedella striscia (b, c), i due triangoli hanno il latoBC in comune e perpendicolare alle tre rette.

a ′

B

C

A

b

a

c

Figura 1

OSSERVAZIONE Un secondo modo di costruire il triangolo cercatoe il seguente. Fissato un puntoB di b, sitracci daB una delle due rette, formanti con le tre parallele un angolo di 30 (fig. 2). Sia poiD l’intersezionefra r e c, e siaA l’intersezione dia con l’asse del segmentoBD. Il cerchioγ avente centro inA e raggiouguale aAB interseca la rettab, oltre che inD, in un secondo puntoC. Poiche AB = AC eBAC = 60 (inquantoBAC eBDC sono rispettivamente l’angolo al centro e l’angolo alla circonferenza relativi al cerchioγ e all’arco

_BC) si puo concludere cheABC e un triangolo equilatero.

38 I problemi della Scuola Normale

B

A

C

D

γ r

c

a

b

Figura 2

Anno 1968 39

1968.3 Sono dati in un piano quattro puntiA, B, C, D, in modo cheA, B, C e A, B, D siano verticidi triangoli equilateri distinti. Determinare tutte le circonferenzeβ che godono della seguente proprieta: iquattro puntiA, B, C, D hanno dalla circonferenzaβ ugual distanza.

RISOLUZIONE Il quadrilateroADBC e costituito dai due triangoli equilateriABC e ABD simmetricirispetto alla retta perA eB.Siaβ una circonferenza equidistante dai puntiA, B, C, D e sianoO e r il suo centro e il suo raggio.Osserviamo intanto cheβ non puo passare per nessuno dei quattro punti, perche in tal caso dovrebbe passareper tutti, ma il quadrilateroACBD none inscrittibile in una circonferenza.I punti A, B, C, D non possono neppure trovarsi tutti da una stessa parte rispetto alla circonferenzaβ (tuttiinterni o tutti esterni ad essa): ne seguirebbe infatti che essi sono equidistanti dal centro diβ.Si danno allora due casi:(1) tre punti sono da una parte diβ e uno dall’altra;(2) due punti sono interni aβ e due sono esterni.

La distanza di un puntoP daβ e r − OP seP e interno aβ, OP − r seP e esterno.

Nel caso (1), supponendo ad esempio cheA, B, C stiano dalla stessa parte, si deve avere:

AO − r = BO − r = CO − r = r − DO ;

quindi O e il circocentro del triangoloABC eβ ha raggio

r =AO + DO

2=

√3

2AB .

Le circonferenze di questo tipo sono dunque quattro, a seconda che il punto isolato siaD, C, B oppureA;oltre alla circonferenza sopra descritta si ha quindi la suasimmetrica rispetto adAB, la circonferenza cheha centro inB (circocentro del triangoloACD) e raggior = AB/2 e infine la simmetrica di quest’ultimarispetto alla retta perC eD.

Nel caso (2), i due punti interni aβ devono essere equidistanti dal suo centroO, e cosı pure i due puntiesterni; dunqueO appartiene all’asse dei due punti interni e all’asse dei duepunti esterni. Poiche gli assidei segmentiAD e BC sono paralleli e distinti, e lo stesso accade per gli assi deisegmentiAC e BD, lasola possibilita e cheO sia il punto di incontro degli assi diAB e CD, cioe il centro del romboACBD(fig. 1). Sara allora:

AO − r = BO − r = r − CO = r − DO ;

da cui

r =AO + CO

2=

√3 + 14

AB .

A B

C

D

O

Figura 1

40 I problemi della Scuola Normale

Abbiamo cosı trovato cinque circonferenze distinte che hanno la proprieta richiesta; dall’analisi fatta seguechiaramente che non ve ne sono altre.

Anno 1970 41

1970.4 Fissato un intero positivon, determinare il piu piccolo interom tale che, presi comunquemnumeri interi positivi, una almeno delle seguenti eventualita si verifichi:(a) tra gli m numeri considerati ve ne sonon uguali;(b) tra gli m numeri considerati ve ne sonon distinti.

RISOLUZIONE Consideriamo un insiemeS di numeri che non verifica nessuna delle due proprieta. InS visono allora non piu di (n − 1) tipi diversi di numeri, altrimenti (b) sarebbe soddisfatta, e di ciascun tipo vene sono non piu di (n − 1), in caso contrario sarebbe verificata la (a). DunqueS non puo contenere piu di(n − 1) · (n− 1) = (n − 1)2 elementi.Osserviamo pero che deve risultare

m > (n − 1)2 ;

e facile infatti costruire un insieme di (n − 1)2 numeri che non contienen numeri uguali ne contienennumeri diversi fra loro: basta prendere (n− 1) classi ciascuna contenente (n− 1) numeri uguali fra loro, inmodo che elementi di classi diverse siano diversi. Ad esempio:

1, 1, . . . , 1︸ ︷︷ ︸(n − 1) volte

, 2, 2, . . . , 2︸ ︷︷ ︸(n − 1) volte

, . . . , (n − 1), (n − 1), . . . , (n − 1)︸ ︷︷ ︸(n− 1) volte

.

Dunque, il piu piccolom per cuie certamente verificata una delle eventualita (a), (b)e dato da (n− 1)2 + 1.

42 I problemi della Scuola Normale

1971.4* Una palla si trova su un biliardo in posizioneP . Provare che esiste almeno una direzionesecondo cui si puo lanciare la palla in modo che essa non ripassi mai per la posizioneP .Si consideri il biliardo privo di attrito e si supponga che ilrimbalzo alle sponde obbedisca alla stessa leggedi riflessione della luce.

RISOLUZIONE Supponiamo il biliardo rettangolare, con lati di lunghezzaa e b; introduciamo un sistema diassi cartesiani ortogonali aventi origineO nel centro del biliardo e assi paralleli ai lati.Indichiamo conP (t) la posizione della palla all’istantet, conP = P (0) la posizione iniziale, conA(t) eB(t) le proiezioni diP (t) rispettivamente sull’assex e sull’assey e infine conνx e νy le componenti dellavelocita iniziale lungo le direzioni degli assi (fig. 1).

y

xO A(t)

B(t)P(t)

νy

νx

Figura 1

Seνx = 0, oppureνy = 0, la palla ripassa certamente perP . A meno di invertire la direzione di uno o dientrambi gli assi, possiamo supporre

νx > 0, νy > 0 .

Osserviamo che, durante il moto, la componente della velocita lungo l’assex si mantiene costante in moduloe cambia segno a ogni rimbalzo lungo i lati verticali del biliardo; allo stesso modo, la componente lungol’assey e costante in modulo e cambia segno a ogni rimbalzo lungo i lati orizzontali.Caso semplice Trattiamo dapprima il caso in cui la posizione inizialeP e l’origine, dunqueA(0) = B(0) =O.Il punto A(t) coincidera nuovamente conO se e soltanto se tale punto ha percorso una distanza cheemultipla dia, il che avviene negli istanti

t =n a

νx, n = 1, 2, . . . ;

analogamenteB(t) coincidera conB(0) = O negli istanti

t =m b

νy, m = 1, 2, . . . .

Pertanto la palla ritorna nella posizione iniziale se e soltanto se esistono degli interim, n tali che

n a νy = m b νx . [1]

La condizione [1] equivale a dire che i numeriνy/νx e b/a sono “razionalmente dipendenti”, cioe vi e unnumero razionalem/n che moltiplicato per il primo da il secondo.Osservando ora che assegnare la direzione iniziale equivale ad assegnare il rapportoνy/νx, da quanto dettosi ricava che esistono infinite direzioni iniziali che verificano la condizione richiesta: basta che sia

Anno 1971 43

νy

νx= α

b

aconα irrazionale;

in tal caso, infatti, la [1] none soddisfatta per nessuna scelta degli interim en, e dunque la palla non ritornamai nella posizione iniziale (ad esempio si puo prendereνx = a, νy = b

√2).

Caso generale Siano orax0 e y0 le coordinate del punto inizialeP . Il punto A(t) ritornera in A(0), diascissax0, dopo aver effettuato o un numero pari o un numero dispari di rimbalzi sulle sponde verticali. Laprima eventualita si verifica negli istanti

t =2n a

νx, n = 1, 2, . . . ;

la seconda negli istanti

t =2n a + 2

(a2 − x0

)

νx=

(2n + 1)a − 2x0

νx, n = 0, 1, 2, . . . ;

Analogamente il puntoB(t) ritorna nella posizione inizialeB(0) negli istanti

t =2m b

νy, m = 1, 2, . . .

e

t =(2m + 1)b − 2y0

νy, m = 0, 1, 2, . . . .

Pertanto la palla ritorna nella posizione inizialeP se e soltanto se esistono due interim edn che verificanouna delle condizioni seguenti

n a νy = m b νx , [1]

2n a νy = [(2m + 1)b − 2y0] νx , [2]

[(2 n + 1)a − 2x0] νy = 2m b νx , [3]

[(2 n + 1)a − 2x0] νy = [(2m + 1)b − 2y0] νx . [4]

Scegliamo la velocita iniziale, come nel caso precedente, in modo cheνy/νx non sia razionalmente dipen-dente dab/a e mostriamo che, con tale scelta, la palla passa al piu tre sole volte daP . Ne seguira la tesi, inquanto la direzione dell’ultimo passaggio inP ovviamente non da luogo a ulteriori passaggi perP .Per la scelta effettuata diνy/νx, la [1] non si puo verificare.Se esistessero due distinte coppie di interin, m e n′, m′ per le qualie soddisfatta la condizione [2], siavrebbe

2n a νy = [(2m + 1)b − 2y0] νx ,

2n′ a νy = [(2m′ + 1)b − 2y0] νx ,

da cui, sottraendo membro a membro,

2 (n − n′) a νy = 2 (m − m′) b νx

e dunqueνy/νx e b/a sarebbero razionalmente dipendenti.Allo stesso modo si prova che non vi possono essere due coppiedistinten, m e n′, m′ che verificano lacondizione [3], ne che verificano la condizione [4].Dunque [2], [3], [4] possono, ciascuna, essere soddisfatteal piu una volta e dunque la palla ritornera nellaposizione iniziale al piu 3 volte.

44 I problemi della Scuola Normale

OSSERVAZIONE 1 Si puo facilmente verificare che se le condizioni [2] e [3] sono entrambe soddisfatte allorae soddisfatta anche la [4]. Inoltre, se si verifica la [4] per una certa coppia di interin em, allora la traiettoriadella palla passa per un vertice del rettangolo. Le traiettorie che non sono “razionalmente dipendenti” dailati del biliardo (cioe conνy/νx eb/a non razionalmente dipendenti) e che non passano per i vertici possonoal piu verificare, e un sola volta, una delle condizioni [2] o [3].

OSSERVAZIONE 2 Un altro metodo per risolvere il problema, piu conciso ma che fa uso di nozioni nonelementari sulla cardinalita degli insiemi,e il seguente.Si osserva anzitutto che ad ogni rimbalzo sulle pareti, poiche l’angolo di incidenzae uguale all’angolo diriflessione, il percorso riflessoe simmetrico del prolungamento del percorso incidente rispetto al lato su cuiavviene il rimbalzo (fig. 2).Dunque se si costruiscono le immagini simmetriche del biliardo rispetto a tutti i suoi lati, e successivamente sicostruiscono i simmetrici di tali immagini, infinite volte,attorno ai loro lati, si ottiene un “tappezzamento” delpiano con rettangoli delle stesse dimensioni del biliardo ein ciascuno di essi risulta individuata l’immaginedel punto di partenza (fig. 3).Per l’osservazione precedente, il percorsoe rappresentato da una semirettat uscente dal punto inizialeP ;pertanto la palla tornera inP se e soltanto se la semirettat passa per una delle immagini diP stesso.Ma l’insieme delle immagini simmetriche diP e in corrispondenza biunivoca con l’insieme dei numerinaturali, cioee uninsieme numerabile; mentre l’insieme delle semirette uscenti daP none numerabile(essoe in corrispondenza biunivoca, ad esempio, con l’insieme dei punti della circonferenza di raggiounitario e centroP , dunque con l’insieme dei numeri reali compresi fra 0 e 2π, il quale none numerabile).Dunque esiste almeno una (anzi, infinite) semiretta uscentedaP che non passa per alcuna delle immaginisimmetriche diP ; il corrispondente percorso della palla sul biliardo non ritorna dunque mai inP .

PP

t

Figure 2, 3

OSSERVAZIONE 3 Di fronte alla proprieta geometrica espressa dall’esercizio in esame, si possono studiareinteressanti proprieta di tipo statistico del moto ideale di una palla di biliardo.Ad esempio si puo dimostrareche ogni traiettoria che partendo daP non vi ritorna passa infinite volte in un intorno di raggio arbitrariamentepiccolo di ogni punto del biliardo, e in particolare diP . Vengono per questo dette traiettorie quasi-periodiche(o ergodiche); si puo dimostrare che le traiettorie quasi-periodiche sono “quasi tutte”, in un senso che, peressere precisato matematicamente, richiede un considerevole apparato, ma chee intuitivamente chiaro intermini statistici, se si tiene presente l’osservazione 2;le traiettorie che partono daP e vi ritornano formanoun insieme numerabile, quelle che non vi tornano formano un insieme infinito piu che numerabile e sonoquindi “molte di piu”.

Anno 1974 45

1973.3* Un treno parte da Pisa. Il macchinista controlla il cronometro e nota che la lancetta deisecondie sullo zero. Dopo aver percorso8 chilometri, il macchinista controlla di nuovo il cronometro enota che la lancetta dei minuti copre esattamente quella delle ore. La velocita media del treno per gli8chilometri percorsie di33 chilometri l’ora. A che orae partito il treno da Pisa?

RISOLUZIONE Osserviamo subito che le informazioni in nostro possesso non ci permetteranno di stabilirese il trenoe partito prima o dopo mezzogiorno. Supponiamo quindi che iltreno sia partito il mattino, eprecisamentep minuti dopo la mezzanotte, conp intero compreso fra 1 e 12· 60 = 720. Per percorrere 8chilometri alla velocita media di 33 km/h, il treno impiega 8· 60/33 minuti; percio, quando il macchinistaguarda nuovamente il cronometro, sono trascorsip + 160/11 minuti dalla mezzanotte.D’altra parte, la lancetta dei minuti si sovrappone a quelladelle ore 11 volte ogni mezza giornata, cioeuna volta ogni 12· 60/11 minuti. Allora, nell’istante in cui il macchinista guarda per la seconda volta ilcronometro, sono trascorsik · 720/11 minuti da mezzanotte, conk intero.Deve essere quindi, perp, k interi e 1≤ p ≤ 720:

72011

k = p +16011

,

da cui si ricava

80 (9k − 2) = 11p . [1]

Da 1≤ p ≤ 720 segue

11≤ 720k − 160≤ 720· 11,

e quindi, dovendo esserek intero, necessariamente

1 ≤ k ≤ 11. [2]

D’altra parte, per la [1], il numero 9k − 2 deve essere divisibile per 11; ma l’unico numerok che verificala [2] e tale che 9k − 2 sia divisibile per 11e k = 10, da cui, per la [1],p = 640. Quindi il trenoe partitoalle 10 : 40.

1974.5 Dati tre numeri interia, b, c aventi massimo comun divisore 1, verificare che i numeri dellaforma

a m2 + b m + c ,

conm intero qualunque, non possono essere tutti divisibili per14.Generalizzare il risultato.

RISOLUZIONE Dimostriamo direttamente che il risultato vale non solo peril numero 14 ma per qualunqueinteroN ≥ 3. Ragioniamo per assurdo, distinguendo due casi.

Caso (a) SeN ha un divisore disparid > 1, allora scegliendom = 0 si ottiene chec e divisibile perde quindi anchea m2 + b m lo e, per qualunquem. In particolare (m = 1 em = −1) i numeria + b e a − brisultano divisibili perd, cosı come la loro somma 2a e la loro differenza 2b. Ma allora, essendod dispari,anchea e b hannod per fattore, il chee impossibile.

Caso (b) SeN e una potenza di 2 non inferiore a 4, cioe N = 2k conk ≥ 2, si puo ragionare come per ilcaso precedente e si ottiene che tutti e tre i numeria, b, c sono divisibili per 2k−1.

Si osservi che perN = 2 il risultatoe falso: ad esempio, sea = b = 1 ec = 2, si ha chem2+m+2 = m (m+1)+2e sempre pari.

46 I problemi della Scuola Normale

1976.3 Sia n un intero maggiore di2 e sia∆ un triangolo rettangolo. Dimostrare che la potenzan-esima della lunghezza dell’ipotenusa di∆ e maggiore della somma delle potenzen-esime dei cateti.

RISOLUZIONE Sianox, y, z rispettivamente le lunghezze dei cateti e dell’ipotenusa;quindix

z< 1 ,

y

z< 1 .

Pertanto, sen > 2, si ha:(x

z

)n

<(x

z

)2,

(y

z

)n

<(y

z

)2

e quindi

xn + yn

zn=

(x

z

)n

+(y

z

)n

<(x

z

)2+

(y

z

)2=

x2 + y2

z2= 1.

1977.1* Su un tavolo orizzontale vie una pila di7 dischi metallici perfettamente uguali, ognuno didiametro40 cm.Dire qualee la distanza massima sul piano orizzontale che puo avere il centro del disco piu alto dal centrodel disco piu basso senza che la pila crolli.

RISOLUZIONE Indichiamo conCk la proiezione, sul piano del tavolo, del centro delk-esimo disco (dall’alto),e conBk la proiezione del baricentro del gruppo dei primik dischi (sempre dall’alto).

Perche la pila stia in equilibrio occorre che il baricentro dei primi k dischi stia sopra il (k + 1)-esimo disco,e cioe, dato che il raggio di ogni discoe 20 cm:

BkCk+1 ≤ 20 conk = 1, 2, . . . , 6 . [1]

Noti i punti Bk eCk+1 il puntoBk+1 si ricava cosı: esso sta sul segmentoBkCk+1 (fig. 1) ede

BkBk+1 =1

k + 1BkCk+1 .

Allora le condizioni per l’equilibrio della pila si traducono in

B1B2 ≤ 12

20

B2B3 ≤ 13

20

. . .

B5B6 ≤ 16

20

B6C7 ≤ 20

[2]

Anno 1977 47

Ck+1 B

k+1 Bk

Figura 1

Osserviamo chee C1 ≡ B1, e che la distanza tra le proiezioni dei centri del primo e delsettimo discoedunqueB1C7. Ma la lunghezza diB1C7 e non superiore a quella della spezzataB1B2B3B4B5B6C7, quindi

C1C7 ≤ 20(

1 +12

+ · · · + 16

)= 49

per ogni pila di dischi in equilibrio (fig. 2).

Mostriamo chee possibile costruire una pila per cuiC1C7 = 49: se i puntiC1, C2, . . . , C7 sono allineati(in quest’ordine), e seC1C2 = 20, C2C3 = 20/2, . . . , C6C7 = 20/6, e facile verificare che le [1] e le [2]diventano tutte delle uguaglianze, e cheC1C7 = 49.

Si puo generalizzare facilmente il risultato ak dischi di raggior: per essi risulta

C1Ck ≤ r

(1 +

12

+ · · · + 1k − 1

).

Figura 2

48 I problemi della Scuola Normale

1979.1* Un battello scende lungo un fiume; sia alla partenza sia a ognistazione intermedia salgonosul battello tanti passeggeri, ognuno diretto ad una diversa stazione, quante sono le fermate successive.Sapendo che il numero massimo di passeggeri contemporaneamente presenti sul battelloe380, si determiniil numero delle stazioni.

RISOLUZIONE Sian il numero delle stazioni, compresa la stazione di partenza.Il battello parte conn − 1passeggeri a bordo: alla prima stazione ne scende uno e ne salgonon− 2; sul battello vi sono ora 2 (n− 2)passeggeri. Alla stazione seguente ne scendono 2 e ne salgono n − 3, e il numero dei passeggeri passa a3 (n − 3). Per ricorrenza si deduce che il battello riparte dallak-esima stazione conk (n − k) passeggeri.Ora osserviamo che il prodottop q di due interi positivi aventi somma assegnatan e massimo quando risulta:

p =n

2, q =

n

2sen e pari;

p =n + 1

2, q =

n − 12

sen e dispari.

Infatti, postop = n/2−δ, q = n/2+δ, l’espressionep q = n2/4−δ2 e tanto piu grande quanto piu δ e piccolo.

Nel nostro caso si hak (n − k) = 380; poiche 380 none della forman2/4 per alcunn, mentre invece si ha380 = 19· 20, si conclude che

k = 20 e n − k = 19

(o viceversa) e quindi si han = 39.

1979.3 Per un puntoP passano tre superfici sferiche distinte tra loro. Si considerino le affermazioniseguenti:(a) nessuna retta passante perP e tangente a tutte e tre le sfere;(b) nessuna sferae tangente a un’altra;(c) esiste un altro puntoQ comune alle tre superfici sferiche.

Dire, per ogni coppia di affermazioni, se esse sono incompatibili, se sono equivalenti o se una delle dueimplica l’altra.

RISOLUZIONE SianoS1, S2, S3 le tre sfere passanti perP e sianoα1, α2, α3 i rispettivi piani tangenti inP .

Proviamo che(a) implica (b).Supponiamo infatti, per assurdo, che valga(a) e non valga(b) e siano, ad esempio,S1 e S2 tangenti inP .Allora α1 = α2 e l’intersezione diα1, α2, α3 contiene almeno una retta perP , che risulta tangente alle tresfere, contro l’ipotesi.Da (b) pero non segue(a): basta per questo considerare tre sfere perP tali che i tre pianiα1, α2, α3 che puressendo distinti (e quindi nessuna sferae tangente ad un’altra) abbiano in comune una rettar, che quindietangente alle tre sfere.Dunque (a) implica (b) (in particolare(a) e (b) sono compatibili), ma le due affermazioni non sonoequivalenti.Da (a) segue(c). Supponiamo infatti che valga(a); per quanto gia dimostrato, vale anche(b), e diconseguenza Siaα il piano di γ; α e distinto daα3, altrimenti la retta tangente aγ in P sarebbe tangentealle tre sfere. Pertanto l’intersezione diα con S3 e un cerchioγ1 passante perP , che none tangente aγ(altrimenti la retta tangente aγ e aγ1 sarebbe tangente alle tre sfere) e quindiγ1 ha conγ, oltre aP , un altropunto in comune,Q, che appartiene alle tre sfere. Cio prova(c).Da(c) non segue pero (a). In un pianoπ prendiamo una circonferenzaγ, e siaP un punto diγ; se scegliamotre sfere distinteS1, S2 eS3 contenentiγ, queste verificano la condizione(c) perche hanno in comune infinitipunti distinti daP , ma non verificano(a) perche la retta tangente aγ in P e tangente a tutte e tre le sfere.

Anno 1980 49

Da (b) non segue(c). Vediamo un controesempio: l’intersezione diS1 eS2 e una circonferenzaγ perP , epossiamo scegliereS3 in modo che sia tangente aγ in P ; allora le tre sfere hanno in comune il solo puntoP(e dunque non verificano l’ipotesi(c)), ma nessuna di essee tangente a nessun’altra (e dunque vale l’ipotesi(b)).Invece, da(c) segue immediatamente(b). Ragionando per assurdo, se due sfere sono tangenti la lorointersezionee costituita da un solo punto, e quindi le tre sfere hanno in comune il solo puntoP , per cuil’ipotesi (c) none verificata.

Abbiamo cosı dimostrato che:dall’affermazione(a) segue la(b) ma dalla(b) non segue la(a);dall’affermazione(a) segue la(c) ma dalla(c) non segue la(a);dall’affermazione(c) segue la(b) ma dalla(b) non segue la(c).

1980.6 Un’autostrada han caselli a distanze successive dip chilometri. Sie osservato che ognimacchina entra con uguale probabilita da ogni casello e esce con uguale probabilita da un altro casello.Trovare la lunghezza del percorso medio di ogni macchina.

RISOLUZIONE Per ragioni di simmetria bastera tener conto delle sole vetture che percorrono l’autostrada inuna direzione; poiche ogni percorso fra glin casellie equiprobabile, il percorso medioe la media aritmeticadelle lunghezze di tutti i percorsi possibili.Dal primo casello sono possibili (n − 1) percorsi, dal secondo (n − 2) e cosı via; dunque il numero deipercorsi possibilie

1 + 2 +· · · (n − 2) + (n− 1) =n (n− 1)

2,

e la media aritmetica delle loro lunghezzee

pn =1 + [1 + 2] + · · · + [1 + 2 + · · · + (n − 1)]

n (n − 1)/2p .

Osserviamo che la precedente relazione si puo porre nella forma

pn =

1 +1 + · · · + [1 + 2 + · · · + (n− 2)]

(n − 1) (n − 2)/2n − 2

n

p

da cui segue che

pn = p + pn−1n − 2

n. [1]

Poiche, ovviamente, si hap2 = p, dalla [1] si ricavap3 = p + p/3 = 4p/3, p4 = p + 2p/3 = 5p/3. Proviamoper induzione chee

pn =n + 1

3p . [2]

Supponiamo dunque che per un certon0 si abbiapn0= (n0 + 1)p/3; dalla [1] si ottiene:

pn0+1 = p + pn0

n0 − 1n0 + 1

= p +n0 − 1

3p =

n0 + 23

p ,

dunque la [2]e soddisfatta pern0 + 1, e quindi per tutti gli interin ≥ 2.Il percorso medio di ogni vetturae dunque di (n + 1)p/3 chilometri.

50 I problemi della Scuola Normale

1981.6* Dato un tetraedro avente5 dei 6 spigoli di lunghezza minore o uguale a2, provare che il suovolumee minore o uguale a1. In quale caso il volumee uguale a1?

RISOLUZIONE SianoA, B, C, D i vertici del tetraedro e supponiamo che gli spigoli diversidaCD abbianolunghezze non superiori a 2.DettaS la superficie del triangoloABC eh l’altezza del tetraedro relativa al verticeD, si ha:

volume(ABCD) =13

S h . [1]

SiaH la proiezione diD sulla retta perA e B e siah′ = DH (fig. 1); ovviamenteh′ e maggiore o ugualealla distanza diD dal piano diABC, dunque si ha:

h ≤ h′ , [2]

e il segno di uguaglianza vale solo quando le facceABD eABC sono ortogonali.Poniamo oraAB = a; poiche nel triangoloABD i lati AD e BD hanno lunghezza minore o uguale a 2, siha:

h′ ≤√

4− a2

4. [3]

Infatti, se ad esempioAH ≤ BH e dunqueBH ≥ a/2,

h′2 = BD2 − BH2 ≤ 4− a2

4.

Nella [3] il segno di uguaglianza si ha solo quandoBD = AD = 2.Allo stesso modo si vede che l’altezza del triangoloABC relativa al latoAB e minore o uguale a

√4− a2/4;

dunque per la sua area si ha

S ≤ a

2

√4− a2

4, [4]

valendo il segno di uguaglianza solo quandoBC = AC = 2.Dalle [1], [2], [3], [4] si ricava allora:

volume(ABCD) ≤ a

6

(4− a2

4

)=

a

24(4− a) (4 +a) =

124

[4 − (2− a)2] (4 + a) .

Ovviamente se 0≤ a ≤ 2, l’espressione [4− (2− a)2] (4 + a) raggiunge il suo massimo pera = 2; dunque

volume(ABCD) ≤ 1 .

A C

B

D

HK

hh ′

A C

BH

D

Figure 1, 2

Anno 1982 51

Dalla discussione precedentee poi chiaro che il volumee uguale a 1 se e solo seABC eABD sono triangoliequilateri di lato 2 situati in piani ortogonali (quindiCD =

√6) (fig. 2).

1982.5* Dato un pianoα e due puntiP , Q nello stesso semispazio si considerino le sfere passanti peri punti P , Q e tangenti al pianoα.Si richiede di determinare il luogo dei punti di tangenza.

RISOLUZIONE SiaS una sfera passante per i puntiP , Q e tangente al pianoα; indichiamo conT il puntodi tangenza. Conduciamo la rettar perP , Q; si possono presentare due casi.

Caso (a) La rettar none parallela al pianoα.

s

r

α

L

C

S

T

P

Q

Figura 1

Indichiamo allora conL il punto di intersezione frar eα; L e distinto daT , in quanto per tre punti in linearetta non passa alcuna sfera, dunqueT e esterno alla rettar.Consideriamo il piano perr e T (fig. 1); detto piano taglia la sfera in un cerchioC e il pianoα lungo unarettas. C passa per i puntiP , Q ede tangente inT alla rettas; dunque, per ilteorema della secante

e della tangente, risulta:

LT 2 = LP · LQ . [1]

Viceversa, ogni puntoT del pianoα che verifica la [1]e punto di tangenza conα di una sfera che passa perT , P , Q e ha centro sulla perpendicolare adα in T . (Per costruire il centro di tale sfera basta intersecare ilpianoβ perpendicolare al segmentoPQ nel suo punto medio con la perpendicolare adα in T .)Si conclude quindi che il luogo cercato dei punti di tangenzae l’insieme dei punti diα che verificano la [1],e quindie un cerchio di centroL e raggio medio proporzionale fraLP eLQ.

Caso (b) La rettar e parallela al pianoα.

52 I problemi della Scuola Normale

α

β

Q

P

S

T

Figura 2

In questo caso il pianoβ perpendicolare al segmentoPQ nel suo punto medioe perpendicolare adα equindi contiene, oltre al centro della sferaS, anche il punto di tangenzaT (fig. 2).Viceversa, ogni punto della retta comune aα e β e punto di tangenza conα di una sfera che passa perT ,P , Q e ha centro sulla perpendicolare adα in T . (Per costruire il centro di tale sfera questa volta occorreintersecare la perpendicolare adα in T con il piano perpendicolare al segmentoPT nel suo punto medio.)Nel caso (b), pertanto, si puo concludere che il luogo cercatoe la retta che si ottiene intersecandoα con ilpianoβ perpendicolare al segmentoPQ nel suo punto medio.

1983.5* Due amici si sono iscritti alla prima classe di un liceo. Taleliceo ha due sezioni, le cui primeclassi hanno rispettivamenten em studenti, conn em compresi tra20 e30.Sapendo che la probabilita che i due amici si trovino nella stessa classee1/2, dire quanti sono gli studentidelle due classi.

RISOLUZIONE Indichiamo conA e B i due amici, e siaM la classe conm studenti eN la classe connstudenti.Vi sonom casi, su un totale dim + n, nei qualiA viene iscritto inM ; sapendo poi cheA sta inM , in m− 1casi sum + n − 1, ancheB viene iscritto inM . Percio la probabilita cheA eB siano entrambi inM e

m (m − 1)(m + n) (m + n − 1)

.

Analogamente, la probabilita che i due amici risultino iscritti entrambi inN e espressa da

n (n− 1)(m + n) (m + n − 1)

.

La probabilita cheA eB siano nella stessa classee espressa dalla somma delle probabilita sopra calcolate.Si ottiene cosı l’equazione

m (m − 1) +n (n− 1)(m + n) (m + n − 1)

=12

, [1]

ossia

Anno 1984 53

2 [m2 + n2 − (m + n)] = (m + n)2 − (m + n) ,

m2 + n2 − 2m n − (m + n) = 0.

L’equazione [1]e dunque equivalente a

(m − n)2 = m + n , [2]

con m e n fra 20 e 30. Dalla [2] si deduce chem + n e un quadrato perfetto; necessariamente, allora, siha chem + n = 49 chee il solo quadrato perfetto compreso fra 40 e 60. Supponendo per esempiom > n,otteniamo

m + n = 49

m − n = 7

e risolvendo si ha

m = 28, n = 21.

In conclusione le due classi hanno 28 e 21 studenti.

1984.6* Siano dati una circonferenzaγ e un puntoP distinto dal centro. SiaPAB un triangolo che,tra tutti quelli che hanno un vertice inP e i rimanenti due suγ, abbia perimetro massimo.Dimostrare che le due bisettrici uscenti dai verticiA eB passano per il centro diγ.(Non si richiede la costruzione geometrica, ne la determinazione degli elementi del triangolo.)

RISOLUZIONE SiaPAB un triangolo che, fra tutti quelli che hanno un vertice inP e gli altri due suγ, haperimetro massimo. Proviamo che la bisettrice diPAB e perpendicolare alla tangente aγ in A e dunquepassa per il centro della circonferenza data.Ragioniamo per assurdo e supponiamo che la perpendicolarep a detta bisettrice non sia tangente allacirconferenza. La rettap ha quindi in comune conγ oltre adA un altro puntoC (fig. 1). Proveremo che iltriangoloPCB ha perimetro maggiore di quello diPAB. Sia infattiP ′ il simmetrico diP rispetto ap; ipunti P ′, A, B sono allineati (in quantop e la perpendicolare alla bisettrice diPAB) e risulta

PA = P ′A , PC = P ′C .

B

A

C

P ′

O

P

γ p

Figura 1

Dalla disuguaglianza triangolare applicata al triangoloP ′CB si ha

54 I problemi della Scuola Normale

P ′B < P ′C + CB

e dunque

PA + AB = P ′A + AB = P ′B < P ′C + CB = PC + CB .

Abbiamo cosı costruito un triangoloPCB che ha perimetro maggiore diPAB; dunquePAB non haperimetro massimo, contro l’ipotesi. Ovviamente lo stessoragionamento si puo ripetere per il verticeB.

OSSERVAZIONE 1 Dalla proprieta dimostrata seguePAB = 2·OAB, PBA = 2·OBA, ma poiche il triangoloOAB e isoscele si ricavaOAB = OBA; pertanto il triangoloPAB di perimetro massimoe isoscele. Seanche il puntoP giace sulla circonferenza, anche la bisettrice diAPB passera per il centro diγ e quindiil triangolo APB, avendo l’incentro coincidente con il circocentro, risulta equilatero. In questo modo siedimostrato che, fra tutti i triangoli inscritti in un cerchio, quello di perimetro massimoe equilatero, risultatoche none facilmente dimostrabile in modo elementare per altra via.

OSSERVAZIONE 2 La proprieta enunciata nel problema si puo ottenere anche con il seguente ragionamentodi tipo meccanico. Consideriamo un elastico teso che passa per il puntoP (fissato), per i puntiA eB (liberidi muoversi sulla circonferenzaγ) e che si chiude tornando inP . Supponiamo che la tensione dell’elastico,in ogni configurazione, sia la stessa in tutti i suoi punti. Laposizione dei puntiA eB per i quali il perimetrodel triangoloPAB e massimo corrisponde al massimo allungamento possibile dell’elastico e rappresentaquindi una posizione di equilibrio (instabile) dello stesso.

P

B

O

FF1

F2A

γ

Figura 2

E chiaro che in tale configurazione la risultante−→F delle due forze

−→F1 e

−→F2 che trasmettono al puntoA la

tensione nelle direzioniPA e PB (fig. 2) deve essere perpendicolare alla tangente al cerchionel puntoA;

in caso contrario, infatti, vi sarebbe una componente non nulla di−→F nella direzione della tangente e dunque

il punto A non sarebbe di equilibrio.

La tesi segue facilmente osservando che|−→F1| = |

−→F2| e di conseguenza il parallelogrammo individuato da

−→F1,−→

F2,−→F e un rombo; dunque gli angoli formati dalle forze

−→F1,

−→F e

−→F2,

−→F sono uguali.

Anno 1985 55

1985.1 L’eguaglianza

p! + q! + r! = s!

e soddisfatta perp = q = r = 2 e s = 3.Dire se esistono altri numeri interi positivi per cui tale eguaglianzae vera.(Si ricorda chen! indica il fattoriale din, cioe il prodotton · (n−1) · . . . ·3 ·2 ·1 dei primin numeri interi.)

RISOLUZIONE Supponiamo di aver trovato quattro numerip, q, r, s che verificano l’eguaglianza. Sia, adesempio,p ≤ q ≤ r; il numeror sara certamente strettamente minore dis, e quindir ≤ s − 1. Avremoallora

s! = p! + q! + r! ≤ 3 · r!

e

s! = s · (s − 1)! ≥ s · r!

per cui risulteras ≤ 3.Ma l’espressionep! + q! + r! vale almeno 3, e quindi deve esseres = 3.I numerip, q, r e di conseguenza i rispettivi fattoriali sono tutti minori ouguali a 2; affinche possa valerel’eguaglianzap! + q! + r! = s! = 6 e necessario che siap = q = r = 2.

1985.2* Si considerino nel piano due circonferenzeγ e γ′ di eguale raggio. Determinare il luogo deipunti mediP dei segmentiAA′ conA in γ eA′ in γ′.

RISOLUZIONE Indichiamo rispettivamente conO, O′ i centri delle due circonferenze e conM il puntomedio diOO′. Tracciamo la circonferenzaγ0 di centroM e raggio uguale a quello diγ eγ′.

γ γ0

γ ′

O

O ′

A B

M P

B ′ A ′

Figura 1

PrendiamoA suγ eA′ suγ′. Costruiamo partendo daA il segmentoAB parallelo, congruente e equiverso alsegmentoOM : ne segue cheABMO e un parallelogramma, e dunqueB e suγ0. Costruiamo poi partendodaA′ il segmentoA′B′ parallelo, congruente e equiverso al segmentoO′M ′: avremo cheA′B′MO′ e unparallelogramma, e dunqueB′ e suγ0. Inoltre, i segmentiAB e A′B′ sono uguali e paralleli, e dunqueABA′B′ e un parallelogramma. Quindi il punto medioP di AA′ e anche punto medio diBB′, chee unacorda della circonferenzaγ0, per cuiP e interno aγ0; se i puntiB e B′ coincidono (il che accade se ilsegmentoAA′ e parallelo aOO′) il punto medioP giace suγ0.D’altra parte seP e un punto interno alla circonferenzaγ0 consideriamo la cordaBB′ staccata daγ0 sullaretta perpendicolare inP alla rettaMP . Il puntoP e punto medio di tale corda e, con la stessa costruzionevista sopra, possiamo determinare un puntoA su γ e un puntoA′ suγ′ in modo cheP sia punto mediodi AA′; seP e sulla circonferenzaγ0, possiamo usare la stessa costruzione prendendoB e B′ coincidenticonP .Questo dimostra che il luogo cercato coincide col cerchio racchiuso daγ0.

56 I problemi della Scuola Normale

1985.3 Fra i triangoli equilateri contenuti in un quadrato assegnato, determinare quelli di area massima.

RISOLUZIONE Osserviamo (fig. 1) che se un triangolo equilateroABC e contenuto in un quadrato,epossibile traslarlo in modo che un suo vertice coincida con un vertice del quadrato. Da tale posizionee poipossibile ruotarlo intorno al vertice in modo che l’altezzarelativa giaccia sulla diagonale del quadrato.

AB

C

B ′

C ′

B ′′

C ′′

B ′′′

C ′′′

A ′

A ′ A ′

Figura 1

Si vede cosı che l’area massimae realizzata dal triangoloA′

B′′′

C′′′

, che si ottiene quando gli altri duevertici cadono sui lati del quadrato; dettaa la lunghezza del lato del quadrato, il lato di tale triangoloequilateroe dato da

A′

B′′′

=a

cos 15=

a√(1 + cos 30)/2

=

a√(2 +

√3)/4

= 2a

√2−

√3

e l’area massima ottenibileea2(2√

3− 3).

Anno 1985 57

1985.4 Fissati due puntiP eQ su due lati consecutivi di un dato rettangolo, si determinino sugli altridue lati due puntiR eS tali che il quadrilateroPQRS abbia area massima.

RISOLUZIONE SianoP , Q, R eS quattro punti rispettivamente sui latiAD, AB, BC eCD del rettangolo. IpuntiP eQ sono fissi, mentreR eS vanno scelti in modo da rendere massima l’area del quadrilateroPQRS,vale a dire in modo da rendere minima la somma delle aree dei triangoliPDS, SCR eRBQ. Poniamo

AD = a , AB = b , PD = c , QB = dDS = x , CR = y ;

la quantita da rendere minimae dunque (a parte un fattore 2)

S = x c + (b − x) y + (a − y) d = x c + (b − d) y − x y + a d

A BQ

D CS

P R

Figura 1

Supponiamox fissato; essendo i terminix c, a d costanti occorre minimizzare (b − d − x) y, con 0≤ y ≤ a.E ovvio che si hanno tre casi, a seconda del segno dib − d − x:(1) sex < b − d il minimo si ha pery = 0;(2) sex > b − d il minimo si ha pery = a;(3) sex = b − d la grandezzae costantemente nulla.Facciamo ora variarex, tenendo conto che deve essere 0≤ x ≤ b: nel caso (1) si haS = x c + a d e quindi ilminimo si ha perx = 0, nel qual casoS = a d. Nel caso (2) si haS = x c+(b−d) a−xa+a d = a b−x (a−c) eil minimo si avra in corrispondenza del massimo valore possibile perx, cioe perx = b, nel qual casoS = b c.Osserviamo infine che il terzo caso si puo escludere, in quanto si avrebbe

S = a d + (b − d) c = b c + (a − c) d

chee certamente maggiore tanto dia d quanto dib c.

Concludiamo che seb c > a d l’area massima diPQRS si ha quandoS coincide conD eR coincide conC;seb c < a d l’area massima si ha quandoS coincide conC eR conB.

SECONDA SOLUZIONESe il quadrilateroPQRS ha area massima, modificando comunqueR o S si otterra unquadrilatero di area non superiore a quella di partenza. Mostriamo che certe scelte diR e diS non possonomassimizzare l’area. Distinguiamo tre casi, a seconda della lunghezza diDS.

SeDS e piu corto diAQ (fig. 2), tenendo fissoS, l’area diPQRS aumenta se aumenta quella del triangoloSQR e questo ha area massima quandoR coincide conC, perche a parita di base l’altezzae maggiore.Quindi, seDS < AQ, il punto R deve coincidere conC; il quadrilateroPQCS non ha certamente areamassima seS 6= D. Dunque, fra i quadrilateri per cuiDS < AQ, l’unico possibile candidatoe quello conR = C eS = D.

A A AQ Q QB B B

P P P

D S C D S

R

C=R D=S C=R

Figura 2

58 I problemi della Scuola Normale

SeDS = AQ, qualunque scelta diR porta ad un’area perPQRS pari alla meta di quella diABCD, ma seBR 6= AP certamente tale area puo essere aumentata.

SeDS > AQ (fig. 3), allora tenendo fissoS conviene prendereR coincidente conB, e perR = B l’areadel quadrilatero non ha un massimo seS 6= C; l’altro candidato per l’area massimae dunquePQRS conR = B, S = C.

A A AQ Q QB

R

P P P

D S D S DC C C=S

B=RB=R

Figura 3

Fra PQCD e PQBC il primo quadrilatero ha area maggiore seCB · QB < DC · DP , equivalente aAD · (AB − AQ) < AB · (AD − AP ), cioeAB · AP < AD · AQ, e finalmenteAP/AD < AQ/AB.

1985.5 Per la costruzione di un certo ponte si prevede che il costo diogni arcata sara di 18s2 miliardidi lire, oves e la distanza in chilometri tra i due piloni di sostegno di quell’arcata, mentre il costo di ognipilone sara di mezzo miliardo.Se il ponte deve essere lungo3 chilometri quale sara il costo minimo dell’opera?

s1 s2 sn

3 km

RISOLUZIONE Sian il numero delle arcate, di lunghezzas1, s2, . . . , sn; il ponte avra quindin − 1 piloni.Si richiede di determinare numero e lunghezza delle arcate,in modo da rendere minimo il costo di costruzione

12

(n − 1) + 18(s2

1 + s22 + · · · + s2

n

)

con la condizione

s1 + s2 + · · · + sn = 3.

Cominciamo col provare che in una configurazione che realizza il costo minimo fra quelle con un numerondi arcate fissato, queste devono avere uguale lunghezza; infatti, se due arcate consecutive hanno lunghezzaa1 = a − h e a2 = a + h, il loro contributo al costoe proporzionale aa2

1 + a22 = 2 a2 + 2h2 e puo essere

ridotto prendendoh = 0, ossia sostituendo le due arcate di lunghezzaa1 e a2 con due arcate uguali lunghea = (a1 + a2)/2.Dunque in una configurazione di costo minimo conn arcate, queste hanno tutte lunghezza 3/n, e il numeron che minimizza il costoe quello per cuie minima l’espressione

Anno 1985 59

12

(n − 1) + 18n

(3n

)2

=12

(n +

182

n

)− 1

2

Tenuto conto che i numerin e 18/n2 hanno prodotto costante, si avra che la loro sommae minima quandosono uguali, cioe quandon = 18.Il costo minimo dell’opera sara allora, in miliardi di lire:

12

(18− 1) + 18· 18(

318

)2

= 8.5 + 9 = 17.5 .

1985.6* Con una bilancia a piatti e disponendo di infiniti pesi campione

p, p1, q1, p2, q2, . . . , pn, qn, . . .

da

1, λ, 1/λ, λ2, 1/λ2, . . . , λn, 1/λn, . . .

grammi rispettivamente, doveλ e un numero reale maggiore di 1, si vogliono pesare tutti gli oggetti conuna precisione arbitrariamente grande. Per quali valori diλ cio e possibile?( N.B. Si dispone di un solo esemplare di ogni peso campione e non si possono mettere pesi campione sulpiatto che contiene l’oggetto da pesare).

RISOLUZIONE Ricordiamo anzitutto che la somma dei primin termini di una progressione geome-

trica a, a2, a3, . . . , an e

a (1− an)/(1− a) .

Cosı, disponendo di un numero arbitrariamente grande di pesi da1/λ, 1/λ2, . . . , 1/λn grammi il pesomassimo che si puo bilanciaree

1− 1/λn

1− 1/λ<

1λ − 1

.

Seλ e maggiore di 2 si ha che 1/(λ − 1) e strettamente minore di 1, e quindi lo scopo non potra essereraggiunto. Si consideri infatti un pesop maggiore di 1/(λ − 1) grammi e minore di 1 grammo, ad esempio

pari a1 + 1/(λ − 1)

2; usando il peso da un grammo otteniamo una approssimazione di p per eccesso, mentre

usando solo i pesi da 1/λk grammi otteniamo stime per difetto inferiori a 1/(λ−1). Si vede quindi che tanto

la stima per eccesso che quelle per difetto differiscono dalpeso vero per almeno1− 1/(λ − 1)

2grammi.

Mostriamo ora che l’obiettivo puo essere conseguito se 1< λ ≤ 2.

0 λ − 11 λ λ2 λn λn+1w

Figura 1

Fissato un pesow, daλ > 1 segue che vie un unico intero relativo, che indichiamo conn0, tale che

λn0 ≤ w < λn0+1 ; [1]

60 I problemi della Scuola Normale

poniamo quindi sulla bilancia il peso daλn0 grammi (che sara il pesop sen0 = 0, il pesopn0sen0 > 0

oppure il pesoqm0sen0 = −m0 < 0).

Sew = λn0 abbiamo determinatow con precisione infinita; altrimenti troviamo l’interon1 per cui

λn1 ≤ w − λn0 < λn1+1 .

Dalla [1] e dalla diseguaglianzaλ ≤ 2 otteniamo

0 ≤ w − λn0 < λn0+1 − λn0 = λn0 (λ − 1) ≤ λn0

per cui deve essereλn1+1 ≤ λn0 , e quindin1 + 1 ≤ n0. In particolaren1 e distinto dan0 e possiamo dunqueaggiungere sulla bilancia il peso daλn1 grammi.Proseguendo in questo modo si trovera una sequenza strettamente decrescente di indici,n0, n1, n2, . . ., e sulpiatto della bilancia saranno messi i corrispondenti pesi daλn0 , λn1 , λn2 , . . ..Questo procedimento puo terminare, nel qual caso il pesow e esattamente determinato, o proseguire fintantoche per il peso residuod da bilanciare non si ha

λnh ≤ d < λnh+1

eλnh+1 e inferiore alla precisione richiesta.

Anno 1986 61

1986.1 Si determinino gli interi positivik tali che il polinomio

x5 + x4 + x3 + k x2 + x + 1

sia prodotto di polinomi a coefficienti interi di grado minore di 5.

RISOLUZIONE Se il polinomioP e prodotto di due polinomi a coefficienti interi, si devono prendere inconsiderazione due possibilita a priori distinte:P (x) = P1(x) P2(x) conP1(x) di grado 1 eP2(x) di grado4 oppureP1(x) di grado 2 eP2(x) di grado 3. Possiamo sempre supporre che il coefficiente deltermine digrado massimo diP1 e diP2 sia 1.Nel primo caso si ha cheP (x) = (x− a) P2(x). Questo significa chea e una radice intera diP (x) e, dovendoessere un divisore del termine noto, puo essere solo 1 o−1.Sea = 1 deve essereP (a) = k +5 = 0, e quindik = −5; sea = −1 deve essereP (a) = −1+1−1+k−1+1 =k − 1 = 0, e quindik = 1.Per il secondo caso osserviamo che i termini noti diP1 e diP2 hanno come prodotto 1; dunque sono entrambi1 o entrambi−1.Se sono entrambi 1 si ha:

P (x) = (x2 + a x + 1) (x3 + b2 x2 + b1 x + 1) =

x5 + (a + b2) x4 + (1 +a b2 + b1) x3+

(b2 + a b1 + 1)x2 + (a + b1) x + 1 ;

uguagliando i coefficienti dei termini di grado 4 e di grado 1 si ottiene a + b2 = a + b1 = 1, e quindib1 = b2 = 1− a; uguagliando i coefficienti degli altri due termini, e sostituendob1 a b2 si ha

1 +a b1 + b1 = 1

b1 + a b1 + 1 = k

e le due relazioni sono compatibili solo sek = 1, nel qual caso si ottiene che (a + 1)b1 = (a + 1) (1− a) = 0e finalmentea = 1, b1 = b2 = 0 oppurea = −1, b1 = b2 = 2.Se invece i termini noti diP1 e diP2 sono entrambi−1 si ha:

P (x) = (x2 + a x − 1) (x3 + b2 x2 + b1 x − 1) =

x5 + (a + b2) x4 + (−1 +a b2 + b1) x3+

(−b2 + a b1 − 1)x2 + (−a− b1) x + 1 ;

uguagliando i coefficienti dei termini di grado 4 e di grado 1 si haa+b2 = 1,−a− b1 = 1, da cuib1 = −1−a,b2 = 1−a; uguagliando i coefficienti degli altri termini e sostituendo le relazioni trovate perb1 eb2 otteniamo:

− 1 +a (1− a) − 1− a = 1

− 1 +a + a (−1− a) − 1 = k

da cui

− 2− a2 = 1

− 2− a2 = k

ma la prima delle due relazioni none soddisfatta per nessun valore dia.

Dunque il polinomio si fattorizza in prodotto di polinomi a coefficienti interi solo perk = 1 e perk = −5.Perk = −5 si ha

P (x) = (x − 1) (x4 + 2x3 + 3x2 − 2x − 1) .

Perk = 1 si ha

P (x) = (x + 1) (x4 + x2 + 1) .

62 I problemi della Scuola Normale

1986.2 Si dimostri che il sistema di equazioni

y ex − e−y = x y ex

x ey − e−x = ey

ha una sola soluzione.

RISOLUZIONE Possiamo riscrivere il sistema nella forma

y (1− x) ex = e−y

(1− x) ey = −e−x[1]

da cui otteniamo chex 6= 1 e anchey 6= 0. Dividendo membro a membro le due equazioni otteniamo

y ex−y = −ex−y

e quindi, dato cheex−y 6= 0,

y = −1 .

Sostituendo nella prima equazione di [1] otteniamo

(x − 1)ex = e ,

ovvero

(x − 1)ex−1 = 1 . [2]

Ne segue chex > 1; la funzione (x−1)ex−1 e, perx ≥ 1, una funzione crescente in quanto prodotto di duefunzioni crescenti non negative; perx = 1 vale 0, e perx = 2 valee = 2.718... > 1, e quindi l’equazione [2]ammette una sola soluzionex0, compresa fra 1 e 2.L’unica soluzione del sistema propostoe dunque la coppiax = x0, y = −1.

1986.3 Si dimostri che la composizione di due omotetie nello spazio, con poliP e Q distinti, e ancorauna omotetia di poloR, allineato conP eQ, oppure una traslazione parallela aPQ.

RISOLUZIONE Si considerino due omotetie di centri rispettivamenteP eQ e costantiα eβ che, per fissarele idee, supporremo per ora positive e minori di 1.Un qualunque puntoX dello spazio (diverso daP eQ) viene mandato dalla prima omotetia in un puntoX ′

allineato conP e conX in modo chePX ′ : PX = α e dalla seconda in un puntoX ′′ allineato conQ e conX in modo cheQX ′′ : QX = β.SiaA un qualunque punto dello spazio e applichiamo le due omotetie: la prima mandera A in un puntoA′

allineato conP e conA tale chePA′ : PA = α, la seconda mandera A′ in un puntoA′′ allineato conQ econA′ tale cheQA′′ : QA′ = β (fig. 1).

XX ′P

X ′′

Q

A

P

Q

A ′

A ′′

R

Figura 1

Anno 1986 63

Il piano perP , Q, A contiene ovviamente siaA′ sia A′′; dato cheα e β sono minori di 1, il puntoA′ einterno al segmentoPA e il puntoA′′ e interno al triangolo di verticiP , Q, A. Pertanto la retta perA eA′′

intersechera il segmentoPQ in un suo punto interno, che indicheremo conR.Proveremo che il puntoR non dipende dalla scelta diA e che il rapportoRA′′ : RA e costante. Ne seguirache la trasformazione che mandaA in A′′ e una omotetia dello spazio di centroR.Sia M l’intersezione della parallela perA′ alla rettaPQ con la rettaAR; dalla similitudine dei triangoliAPR eAA′M eQRA′′ eA′MA′′ (fig. 2) si ha:

A ′A

M

P

RQ

A ′′

Figura 2

MA′

PR=

AA′

PA=

PA − PA′

PA= 1− PA′

PARQ

MA′=

QA′′

A′A′′=

QA′′

QA′ − QA′′=

QA′′

QA′/

(1− QA′′

QA′

).

[1]

Tali rapporti sono dunque delle costanti (rispettivamenteuguali a 1− α e aβ/(1− β)) indipendentementedalla scelta del puntoA.Pertanto il rapportoRQ : PR e indipendente daA e quindi il puntoR e univocamente individuato sulsegmentoPQ, qualunque sia il puntoA iniziale. Inoltre, sempre per similitudine, abbiamo

RM

RA=

PA′

PA= α ,

RA′′

RM=

QA′′

QA′= β ,

e dunqueRA′′ : RA e una costante (pari aα β).Risulta cosı dimostrato, nel caso 0< α, β < 1, che la composizione di due omotetiee una omotetia.

Il caso in cui siaα siaβ sono maggiori di 1e del tutto simile, con il ruolo dei puntiP eQ scambiato (fig. 3).

A ′′

MA A ′

P

R

Q

Figura 3

Nel caso in cui, invece, si ha 0< α < 1 eβ > 1 (o viceversa), occorrera distinguere, in quanto none piugarantito che la retta perA e perA′′ incontri la retta perP e Q. Se la incontra, certamente il punto di

64 I problemi della Scuola Normale

intersezionee esterno al segmentoPQ e l’argomentazione precedente si puo ripetere inalterata (anche secambiera il segno della costante nella seconda delle [1] in quantoA′A′′ = QA′′ − QA′).

A ′′

A ′ M

A

P

QR

A ′′

A

P

Q

A ′

Figura 4

Se invece la retta perA, A′′ e parallela alla retta perP e Q, i triangoli AA′A′′ e PA′Q sono simili conrapporto di similitudineA′A : PA′ = A′A′′ : QA′ , da cui segue

PA − PA′

PA′=

QA′′ − QA′

QA′, cioe

PA

PA′=

QA′′

QA′,

da cui finalmenteα β = 1; in questo caso la composizione delle due omotetiee una traslazione.I casi in cui uno o entrambi i parametriα eβ sono negativi si trattano in maniera analoga.

SECONDA SOLUZIONE

Diamo anche una soluzione basata sulla geometria analitica.Indichiamo conhP l’omotetia di poloP e conα la sua costante, conhQ l’omotetia di poloQ e conβ lasua costante; se fissiamo un sistema di coordinate e indichiamo con (xP , yP , zP ) le coordinate del puntoPe con (xQ, yQ, zQ) le coordinate del puntoQ, le due omotetie si scrivono come

x′ = xP + α (x − xP )

y′ = yP + α (y − yP )

z′ = zP + α (z − zP )

x′′ = xQ + β (x − xQ)

y′′ = yQ + β (y − yQ)

z′′ = zQ + β (z − zQ)

che possiamo indicare compattamente con la notazione

A′ = P + α (A − P ) A′′ = Q + β (A − Q)

E facile vedere che tale notazionee compatibile con le usuali proprieta algebriche (associativita, distributi-vita, . . . ).Indichiamo cont la trasformazione composta, definita da

t(A) = hQ(hP (A)) = Q + β(P + α(A − P ) − Q) =

Q + β(P − Q) + βα(A − P ) .

Seαβ = 1 si ha

t(A) = Q + β(P − Q) + (A − P ) =

Q + β(P − Q) + (A − Q) + (Q − P ) =

A + (1− β)(Q − P )

e dunquet e una traslazione con vettore (1− β)(Q − P ), parallelo alla retta perP eQ.Dimostriamo ora che seαβ 6= 1 la composizionee una omotetia, mostrando che esiste un solo punto unitoR e chet(A) = R + αβ(A − R).

Anno 1986 65

Per un generico puntoA la distanza dit(A) dalla rettal per P e Q e pari alla distanza diA dalla rettal,moltiplicata per|αβ| 6= 1; dunque seR e un punto unito dit, deve giacere sulla rettal, e quindi ha la formaR = P + r(Q − P ). Dalla relazionet(R) = R segue allora che

R − P = r(Q − P ) = (Q + β(P − Q) + rαβ(Q − P )) − P

e quindi

r(1− αβ)(Q − P ) = (1− β)(Q − P ) , r =1− β

1− αβ

Verifichiamo che la trasformazionet e una omotetia di poloR e di costanteαβ: per ogni puntoA si ha cheil punto immaginet(A) = Q + β(P − Q) + αβ(A − P ) si puo esprimere comeR + αβ(A − R). Infatti

Q + β(P − Q) + αβ(A − P ) =

P + (Q − P ) + β(P − Q) + αβ(R − P ) + αβ(A − R) =

P +(

1− β +αβ(1− β)

1− αβ

)(Q − P ) + αβ(A − R) =

P + (1− β)(

1 +αβ

1− αβ

)(Q − P ) + αβ(A − R) =

P +1− β

1− αβ(Q − P ) + αβ(A − R) =

R + αβ(A − R) .

66 I problemi della Scuola Normale

TERZA SOLUZIONE

Diamo una soluzione sintetica piu rapida, anche se lievemente piu sofisticata.Il problemae di geometria tridimensionale, ma presenta una simmetria cilindrica intorno alla retta per i polidelle due omotetie, il che consente di provare la tesi studiandolo su un piano (e dunque su tutti i piani perP eQ).Una omotetia manda, per ilteorema di Talete, ogni retta del piano in una retta ad essa parallela; maallora anche la composizione di due omotetie gode della stessa proprieta. Prendiamo due puntiA eB nongiacenti sulla rettaPQ e indichiamo conA′ e B′ le loro immagini attraverso la trasformazione composta.Per quanto osservato le retteAB eA′B′ sono parallele. Vi sono due possibilita:

(a) le retteAA′ eBB′ sono fra loro parallele; in tal casoAA′B′A′ e un parallelogramma, e in particolaresi haAA′ = BB′ eAB = A′B′.

A

B

C

A ′

B ′

C ′

Figura 4

SeC e un qualunque altro punto del piano eC′ e il suo trasformato si ha che i triangoliABC e A′B′C′

sono simili; essendoAB = A′B′, essi sono uguali e quindiAC = A′C′. Ne segue cheACC′A′ e unparallelogramma e dunque i segmentiCC′ eAA′ sono paralleli ed hanno ugual lunghezza. La trasformazionequindi e una traslazione; per provare che il segmentoAA′ e parallelo aPQ basta osservare che, per quantoappena visto,AA′ e parallelo aPP ′; d’altra parte il puntoP viene lasciato invariato dalla prima omotetiadi poloP e quindi il puntoP ′ giace sulla retta perP eQ.

(b) le retteAA′ e BB′ si intersecano in un puntoR, che puo cadere esternamente o internamente aisegmentiAA′ eBB′.

A

B

C

A ′

B ′

K

R

A

B

C

K

B ′

A ′

R

Figura 5

Dal fatto cheAB e A′B′ sono paralleli segue che i triangoliABR e A′B′R sono simili e quindi cheAB : A′B′ = AR : A′R = BR : B′R.SiaC un qualunque punto non allineato conA eB. Tracciamo la retta perC eR e siaK la sua intersezionecon la parallela condotta perA′ alla rettaAC. Per il teorema di Talete si haCR : KR = AR : A′Re quindi, tenendo conto delle proporzionalita precedentemente ricordate,CR : KR = BR : B′R;quindi K e intersezione della parallela adAC per A′ con la parallela aBC per B′, e dunque coincidecon il trasformato diC; in particolare, per ogni puntoC, la rettaCC′ passa perR e vale la proporzione

Anno 1986 67

CR : C′R = AR : A′R. La trasformazionee quindi una omotetia; che il puntoR giaccia sulla retta perPQ segue dal fatto che esso appartiente alla retta perP eP ′, e cheP ′ giace sulla retta per i poli.

1986.4 Sia ABC un triangolo isoscele di baseBC con l’angolo al verticeBAC minore di60. Sicostruisca un altro triangoloPQR, di baseQR, circoscritto e simile adABC, tale che il puntoA appartengaal segmentoQR e si abbiaQA = 2 · AR.

RISOLUZIONE Si traccino la circonferenza perA e B, tangente inB al segmentoBC, e la circonferenzaperA eC, tangente inC al segmentoBC, e sianoO eO′ i centri delle due circonferenze.

CB

A ′

O O ′

T

A

Figura 1

I lati obliqui del triangolo sono visti, dai punti degli archi di tali circonferenze esterni adABC, sotto unangolo uguale a quelli alla base del triangolo stesso. Si individui il punto T del segmentoOO′ tale cheOT = 2TO′. Si noti che, essendoBAC < 60, l’ulteriore puntoA′ di contatto delle circonferenzee internoal triangolo, cosicche il puntoT giace sotto la parallela perA alla baseBC.Si mandi daT la congiungente conA e daA la perpendicolare a questa; essa tagliera le circonferenze indue ulteriori puntiQ eR che sono quelli cercati.

P

B C

O T O ′

QA

RH

H ′

Figura 2

Infatti seH e H ′ sono i piedi delle perpendicolari condotte daO e daO′ a QR, per il teorema di

Talete si avra HA = 2AH ′, maQH = HA e AH ′ = H ′R e dunqueQA = 2AR. D’altra parte le rette

68 I problemi della Scuola Normale

perQ, B e perR, C individuano un triangoloQRP che ha gli angoli alla baseQR uguali a quelli diABC,e dunquee isoscele e ad esso simile.

1986.5*

(a) Sianoα, β, γ, δ quattro angoli minori di180. Si dimostri che

sinα + sinβ + sinγ + sinδ ≤ 4 sinα + β + γ + δ

4.

(b) Utilizzando la relazione precedente, si dimostri che la somma dei seni degli angoli interni di un triangoloe sempre minore o eguale a3

√3/2.

RISOLUZIONE Per il punto (a) basta provare che seα eβ sono due angoli minori di 180 risulta

sinα + sinβ ≤ 2 sinα + β

2; [1]

la stessa relazione varra perγ e δ, e anche per gli angoli (α + β)/2 e (γ + δ)/2 che sono anch’essi minori di180. Avremo quindi

sinα + sinβ + sinγ + sinδ ≤ ≤ 2 sinα + β

2+ 2 sin

γ + δ

2≤

4 sinα + β + γ + δ

4.

La diseguaglianza [1] segue immediatamente dalla concavita della funzione seno tra 0 e 180; ne diamocomunque una dimostrazione elementare. La [1] vale se e soltanto se

sin 2α + sin 2β ≤ 2 sin (α + β)

perα eβ fra 0 e 90. L’ultima diseguaglianza equivale a

2 sinα cosα + 2 sinβ cosβ ≤ 2 sinα cosβ + 2 cosα sinβ

e ancora a

(sinα − sinβ) (cosα − cosβ) ≤ 0 ;

ma cio e evidente perche, per angoliα eβ fra 0 e 90, sinα ≤ sinβ se e solo se cosα ≥ cosβ.

Punto (b): sianoα, β, γ gli angoli interni di un triangolo. Scriviamo la disuguaglianza del punto (a) per gliangoliα, β, γ e δ = (α + β + γ)/3 = 60. Si ha

sinα + sinβ + sinγ + sinδ ≤ 4 sin(14· 4δ)

da cui

sinα + sinβ + sinγ ≤ 3 sin 60 =3√

32

.

Anno 1986 69

1986.6* Si consideri un biliardo di forma triangolare, come in figura, conα = 30 e si supponga dilanciare una bilia dal puntoA.

α

A B

Si provi che la bilia, qualunque sia la direzione iniziale, effettua solo un numero finito di rimbalzi prima dibattere sulla spondaAB.Si determini anche il numero massimo di tali rimbalzi.Si studi infine il caso in cuiα e un angolo generico.(Si supponga che il biliardo sia privo di attrito, che la pallasia puntiforme e che il rimbalzo sulle spondeobbedisca alla legge di riflessione della luce). 1

RISOLUZIONE Supponiamo dapprima che il biliardo abbia la forma di un angolo (infinito) conO di 30.

Figura 1

Indichiamo conOS0 la sponda su cui giace il punto di partenzaA e conOS1 quella su cui giace il primopunto di rimbalzoA1. Siano poiA2, A3, . . . , i successivi punti di rimbalzo. La traiettoria della biliaecostituita dalla spezzataAA1A2A3 . . .. Consideriamo un angoloS1OS2 di 30 e siaA′

2 il punto di OS2 percui OA′

2 = OA2; proviamo che i puntiA, A1 eA′2 sono allineati.

I triangoli OA2A1 e OA′2A1 sono uguali, pertantoA′

2A1O = A2A1O; per ipotesi l’angolo di incidenzaeuguale a quello di riflessione, quindiA′

2A1O = A2A1O. Ne segue

A′2A1O + A2A1O + A2A1A = AA1S1 + A2A1O + A2A1A = 180 ,

dunque i puntiA, A1 eA′2 giacciono su una retta.

Allo stesso modo si prendano degli angoliS2OS3, S3OS4, . . . , tutti di 30, e sulle semiretteOSm siprendano dei puntiA′

m tali cheOA′m = OAm perm = 2, 3, . . . (i punti A′

1 e A1 coincidono). Con lo stesso

1 Il testo originale del problemae leggermente diverso (N. d. C.).

70 I problemi della Scuola Normale

ragionamento si vede che i puntiA1, A′2 eA′

3 sono allineati, e dunqueA, A1, A′2, A′

3, . . . stanno su una rettar chee lo sviluppo della traiettoria della bilia.

S6 S0O

S5 S1

S4 S2

S3

A

A1

A2

A3

A4

A2′A3

′A4

Figura 2

E chiaro quindi che non vi possono essere piu di 5 rimbalzi.Se ora reintroduciamo la spondaAB (fig. 2) e evidente che la bilia non puo toccare piu di 5 volte le spondelaterali prima di battere suAB.Nel caso di un angoloα generico il numero massimo di rimbalzi sara dato dall’interon tale che

n <π

α≤ n + 1.

O A

B

Figura 3

Figura 4

Anno 1987 71

1987.1 Siano assegnati due numeri reali positivi non nullir e p, conr < p. Tra tutti i quadrilatericonvessi di perimetrop, aventi la somma delle lunghezze di una coppia di lati consecutivi uguale adr, sidetermini quello di area massima.

RISOLUZIONE SiaABCD un quadrilatero convesso di area massima fra quelli che verificano le condizioniassegnate:AB + AD + CB + CD = p e AB + AD = r. In particolare tale quadrilatero ha area maggiore ditutti quelli che verificano le condizioni precedenti e hannofissata la diagonaleBD.

A

C

B D

A ′

C ′

Figura 1

Sea e b indicano le lunghezze dei latiAB eAD, l’area del triangoloABD e data da

S =√

p′ (p′ − a) (p′ − b) (p′ − d) ,

ovep′ e il semiperimetro di tale triangolo. Essendod e a + b (e quindi anchep′ e p′ − d) quantita fissate, ilmassimo diS si avra quandoe massimo (p′ − a) (p′ − b) = p′

2 − p′ (a + b) + ab, ovvero quandoe massimo ilprodottoa b, vale a dire quandoa = b.Ne segue che il triangoloABD, che massimizza l’area,e un triangolo isoscele e quindiAB = AD = r/2.Allo stesso modo si vede cheBCD e isoscele e cheCB = CD = (p− r)/2; si ha inoltre che gli angoliABC

eADC sono uguali perche somme di angoli uguali.Per risolvere il problema basta ora osservare che l’area diABCD e uguale a quella del parallelogramma dilati r/2 e (p − r)/2, e tale parallelogramma ha ovviamente area massima quandoe un rettangolo.

A ′

C ′

B D

Figura 2

In conclusione il quadrilatero di area massima ha le coppie di lati consecutiviAB, AD eCB, CD di eguallunghezza e gli angoli inC eD retti.

72 I problemi della Scuola Normale

1987.2 Sianop, q, r tre numeri reali tali che il polinomio

A(x) = x3 + p x2 + q x + r

abbia tre radici reali.Determinare tre numeri realia, b, c, espressi in funzione dip, q, r soltanto, in modo che il polinomio

B(x) = x3 + a x2 + b x + c

abbia per radici i quadrati delle radici diA.

RISOLUZIONE Per il teorema di Ruffini seα, β, γ sono le radici diA(x) si ha

A(x) = (x − α) (x − β) (x − γ) =

x3 − (α + β + γ) x2 + (α β + α γ + β γ) x − α β γ

e per il principio di identita dei polinomi si ha

p = −(α + β + γ)

q = α β + α γ + β γ

r = −α β γ .

Analogamente seB(x) ha per radiciα2, β2, γ2 sara

B(x) = (x − α2) (x − β2) (x − γ2) =

x3 − (α2 + β2 + γ2) x2+

(α2 β2 + α2 γ2 + β2 γ2) x−α2β2 γ2 .

Dobbiamo quindi esprimereα2 + β2 + γ2, α2 β2 + α2 γ2 + β2 γ2 eα2 β2 γ2 in termini di p, q e r.Avremo

α2 + β2 + γ2 = (α + β + γ)2 − 2 (α β + α γ + β γ) =

p2 − 2q ;

α2 β2 + α2 γ2 + β2 γ2 =

(αβ + α γ + β γ)2 − 2α β γ(α + β + γ) =

q2 − 2p r ;

α2 β2 γ2 = r2 .

Il polinomio B(x) richiesto sara quindi dato da

x3 − (p2 − 2q) x2 + (q2 − 2p r) x − r2 .

Anno 1987 73

1987.3 Sia dato un segmentoAB nel piano. Si consideri il luogoL dei punti del piano che vedono ilsegmentoAB sotto un angolo di60. Si scelgaP in L e si scelgano due puntiC eD rispettivamente interniai lati BP eAP del triangoloABP , in modo cheAD = BC.Si costruisca il triangolo equilateroCDQ di baseCD, esterno al quadrilateroABCD.Si studi, al variare diP in L e di C, D secondo le condizioni indicate sopra, il luogo dei punti delpianodescritto dal puntoQ.

RISOLUZIONE Per prima cosa osserviamo che il luogoL e costituito dall’unione dei due archi dei cerchicircoscritti ai triangoli equilateriABK e ABK ′ costruiti da parti opposte del latoAB, archi contenentirispettivamente i puntiK eK ′.

K ′ K

B

A

L

PD

C

Figura 1

E’ immediato osservare che seP = K, per qualunque scelta diC e D il vertice Q del triangolo equilaterocostruito coincide conK, e simmetricamente seP = K ′ si ha cheQ = K ′.Proviamo quindi a dimostrare che, seP appartiene all’arco contenenteK, il vertice Q coincide conK;otterremo questo mostrando che, per qualunque scelta diP e per qualunque scelta diC eD coerente con lecondizioni, il triangoloCDK risulta equilatero e esterno adABCD.Gli angoliKAD eKBC sono uguali perche angoli alla circonferenza che insistono sullo stesso arcoPK; ilati AK eBK sono uguali perche lati di un triangolo equilatero; i latiAD eBC sono uguali per costruzione.Quindi i triangoliKAD e KBC sono uguali, e in particolare sono uguali i latiKD e KC, e sono ugualigli angoli AKD e BKC; si ha quindi cheDKC = DKB + BKC = DKB + AKD = AKB e dunquel’angolo DKC, opposto alla base del triangolo isosceleCKD e di 60. Di conseguenza il triangoloCKDe equilatero, esterno al quadrilateroABCD, e quindiQ coincide conK seP sta sull’arco contenenteK,oppure conK ′ seP sta sull’arco contenenteK ′. Il luogo da determinare consiste dunque nei due puntiKeK ′.

74 I problemi della Scuola Normale

1987.4* Un punto(x, y) del piano si dira razionalesex ey sono numeri razionali. Data una qualunquecirconferenza del piano cartesiano avente centro razionale, si provi che se essa contiene un punto razionale,allora contiene infiniti punti razionali.

RISOLUZIONE Mostriamo per prima cosa che la tesi vale per una circonferenza centrata nell’origine.Supponiamo che una circonferenzaC di centroO = (0, 0) e raggior contenga un punto razionaleP = (x1, y1).Se indichiamo conα l’angolo formato dalla semirettaOP con la semiretta positiva dell’assex, avremo cher cosα = x1 e r sinα = y1 sono numeri razionali.Seβ e un angolo per cui cosβ e sinβ sono entrambi razionali, allora possiamo esibire un altro punto acoordinate razionali suC, prendendo quello di coordinate

x = r cos(α + β) = (r cosα) cosβ − (r sinα) sinβ ,

y = r sin(α + β) = (r sinα) cosβ + (r cosα) sinβ .

I numeri cosβ e sinβ sono razionali se e solo se esiste una terna di numeri interia, b, c tali che cosβ = a/ce sinβ = b/c; i numeri a, b, c verificano quindi la relazionea2 + b2 = c2 e formano quindi una terna

pitagorica; viceversa, due terne pitagoriche non proporzionali individuano due punti razionali distintidel cerchio unitario.Data una successione di terne pitagoriche(an, bn, cn) a due a due non proporzionali, gli angoliβn

individuati da cosβn = an/cn e sinβn = bn/cn sono tutti diversi, e dunque i punti

Pn = (r cos(α + βn), r sin(α + βn))

al variare din = 1, 2, . . . sono tutti distinti, e sono infiniti punti razionali che appartengono alla circonferenzaC.La successione di terne pitagoriche data da

an = n2 − 1 , bn = 2n , cn = n2 + 1

non contiene terne proporzionali. Da 2n : 2m = (n2+1) : (m2+1) segue infatti che 2n (m2+1) = 2m (n2+1)e infine che 2 (n m− 1) (m − n) = 0, per cuim = n.

Nel caso generale, seC e una circonferenza con centro nel punto a coordinate razionali O′ = (x0, y0) eraggior, e contiene il punto razionaleP = (x1, y1), allora la circonferenzaC′ di centroO e raggior contieneil punto razionale (x1 − x0, y1 − y0) = (r cosα, r sinα), e dunque contiene gli infiniti punti razionali dicoordinate (r cos(α + βn), r sin(α + βn)); ma allora i punti

Pn = (x0 + r cos(α + βn), y0 + r sin(α + βn))

al variare din sono infiniti punti razionali distinti sulla circonferenzaC.

Anno 1987 75

1987.5* Nella figurae rappresentato lo sviluppo delle facce di un tetraedro regolare nello spazio.

A C

B

M

N O

P Q

R

SianoP , Q, R tre punti distinti del tetraedro corrispondenti, nello sviluppo, rispettivamente ad un puntointerno al segmentoMN , un punto interno al segmentoMO, un punto interno al triangoloMOC.Siaα il piano contenenteP , Q, R. Si determini, nello sviluppo piano, l’intersezione fra ilpianoα e le faccedel tetraedro.

RISOLUZIONE Prolunghiamo il segmentoQR fino ad intersecare il perimetro del triangoloMOC. Seil punto di intersezioneS appartiene al segmentoMC (e certamenteS non coincide conM ), e faciledeterminare l’intersezione diα con le facce del tetraedro (fig. 1): dato che il segmentoMC coincide, neltetraedro, con il segmentoMA, il punto di intersezioneS sara rappresentato suMA dal puntoS′, simmetricodi S rispetto adM . Il piano α interseca quindi solo tre facce del tetraedro, lasciando ilverticeM da unaparte, e gli altri tre dall’altra, e l’intersezionee data dai segmentiPQ, QS, S′P .

A CMS ′ S

N OP

Q

R

B

Figura 1

Se il puntoS appartiene invece al segmentoOC (fig. 2), marchiamo suOB il punto S′ conOS′ = OS eindichiamo conπ1 il piano contenente i verticiM , N e O, e conπ2 il piano contenente i verticiB, N e O.L’intersezione del pianoπ1 conπ2 e data dalla rettas su cui giace lo spigoloNO; l’intersezione del pianoα conπ1 e data dalla rettar1 perP eQ.

Distinguiamo due casi, a seconda che le rettes e r1 siano incidenti in un puntoZ, oppure siano parallele.(a) Rettes er1 incidenti inZ: il puntoZ e intersezione dei tre pianiα, π1 eπ2; dunque la rettar2 perZ e il

puntoS′ rappresenta, nello sviluppo piano, l’intersezione diα eπ2.

76 I problemi della Scuola Normale

A

B

CM

N O

P QR S

S ′

T

T ′

Zr1

s

r2

Figura 2

(b) Rettes e r1 parallele: si vede facilmente che la retta lungo cuiα taglia il pianoπ2 non puo intersecarela rettas o, per il ragionamento precedente, anche la rettar1 dovrebbe intersecares; di conseguenzal’intersezione diα con la facciaOBN e data dalla rettar2 perS′ parallela ads.

Indichiamo conT il punto in cui r2 incontra il segmentoBN e conT ′ il suo simmetrico rispetto aN . Ilpianoα incontra tutte e quattro le facce del tetraedro, e l’intersezionee data daPQ, QS, S′T , T ′P .

1987.6 Tizio si trova nella sua abitazione e deve prendere un treno che parte dalla stazione esattamentetra mezz’ora. Sotto la sua abitazione c’e la fermata degli autobus della linea A che lo portano alla stazionein 20 minuti. A 5 minuti di cammino vie una fermata delle linee B e C che lo possono portare alla stazionein 18 minuti.Tizio non conosce l’orario di passaggio degli autobus, ma sache su ognuna delle linee gli autobus passanoogni quarto d’ora.Quale strategia conviene a Tizio per aver maggiore probabilita di prendere il treno?

RISOLUZIONE Dato che non conosce l’orario di passaggio degli autobus, masolo che passano ogni 15minuti, Tizio assume che la probabilita di prendere un autobus aspettandox minuti siax/15.L’autobusA passa certamente entro 15 minuti, ma lo porta alla stazione in tempo solo se passa entro30− 20 = 10 minuti; scegliendo di aspettareA la probabilita di successoe di 10/15.Se invece parte subito per la fermata degli autobusB eC, Tizio avra 25− 18 = 7 minuti utili per attendereil passaggio diB o C. La probabilita cheB passi in tempo utilee di 7/15, e cosı per C; Tizio quindiriuscira a prendere il treno se non capita cheB e C passino entrambi negli 8 minuti successivi, e quindila probabilita di successo sara data da 1− 8/15 · 8/15 = 161/225, chee maggiore di 10/15 = 150/225,probabilita di successo che avrebbe aspettandoA. Fra le due alternative, la miglioree quella di partiresubito per la fermata degli autobusB eC.Potrebbe essere piu conveniente per Tizio una strategia intermedia, consistente nell’aspettare l’arrivo diAperx minuti (x ≤ 7) e poi partire per la fermata diB eC?La probabilita cheA passi nel primix minuti e x/15; giunto alla fermata diB e C, Tizio ha 7− x minutifavorevoli su 15, e quindi la probabilita che non passi nessun autobus in tempo utilee data da ((8 +x)/15)2.La probabilita di successo con la strategia intermediae data da

x

15+

[1−

(8 +x

15

)2]

=15x + 225− 64− 16x − x2

225=

161− x − x2

225che perx = 0 fornisce la probabilita di successo con la strategia “subitoB o C”, e decresce all’aumentaredi x. Dunque Tizio decide di partire subito per la fermata diB eC.

Anno 1988 77

1988.1* SianoA, B, C, D quattro punti distinti assegnati nello spazio. Determinare una condizionenecessaria e sufficiente affinche ogni superficie sferica che passa perA eB intersechi ogni superficie sfericache passa perC eD.

RISOLUZIONE Articoliamo la soluzione in vari punti, in modo da mettere prima in evidenza le condizioninecessarie, e arrivare poi a dimostrarne la sufficienza.

(1) Cominciamo con l’osservare che se esistono due semispazi Π1 e Π2 disgiunti, delimitati dai pianiparalleliσ1 eσ2, tali cheA eB appartengano aΠ1 eC eD appartengano aΠ2, allora esiste una sferaS1

passante perA eB e tangente al pianoσ1 ed una sferaS2 passante perC eD e tangente al pianoσ2; S1

e contenuta inΠ1, S2 e contenuta inΠ2 e sono quindi disgiunte. Per costruire la sferaS1, si tracci nelpiano passante perA eB e perpendicolare aσ1 una circonferenza perA eB e tangente aσ1. Il centro eil raggio di tale circonferenza sono centro e raggio diS1; analoga la costruzione diS2.

S1

A

B

S2

C D

σ1

σ2

Figura 1

I semispaziΠ1 e Π2 esistono certamente nei due seguenti casi.(a) i segmentiAB eCD appartengono a due retter1 e r2 sghembe. SianoH sur1 eK sur2 i due punti

tali cheHK e il segmento di minima distanza frar1 e r2; alloraHK e perpendicolare adr1 in H eadr2 in K. PrendiamoL e M suHK in modo cheHL = LM = MK; il piano perpendicolare adHK in L saraσ1, frontiera diΠ1; il piano perpendicolare adHK in M saraσ2, frontiera diΠ2.

(b) A, B, C, D appartengono ad uno stesso pianoπ e i segmentiAB e CD non si intersecano. In talcaso esistono due rette paralleler1 e r2 che lasciano da parti opposte le coppie di puntiA, B e C,D; i piani σ1 eσ2 saranno i piani perpendicolari aπ e contenenti rispettivamente le retter1 e r2.

A

BH

LM

KC

D

r1

r2

A

B

C

D

r1 r2

Figure 2,3

(2) Quindi, seA, B, C, D sono tali che ogni superficie sferica passante perA eB interseca ogni superficiesferica passante perC e D, necessariamente i quattro punti sono complanari e i segmenti AB e CDsi intersecano. Consideriamo nel pianoπ contenenteA, B, C, D gli assi dei segmentiAB e CD,

78 I problemi della Scuola Normale

che certamente hanno un puntoO in comune dato che i due segmenti non sono paralleli. Le superficisferiche di centroO e raggio rispettivamenteOA eOC non si intersecano seOA 6= OC e coincidono seOA = OC; in tal caso il quadrilateroACBD e inscrivibile in una circonferenza.

A

B

C

D

O

Γ1

Γ2

Figura 4

(3) Mostriamo infine che tale condizionee anche sufficiente. Se osserviamo che l’intersezione conπ diogni superficie sfericaS1 passante perA eB e una circonferenzaΓ1 perA eB e che, analogamente,S2

ha per intersezione una circonferenzaΓ2 perC e D, basta provare che seACBD e inscrivibile in unacirconferenza, allora ogni altra circonferenzaΓ1 perA e B lasciaC e D da parti opposte. Infatti, se ilcentroO1 di Γ1 non coincide con l’intersezioneO degli assi dei segmentiAB e CD, le distanze daO1

dei puntiC eD sono una minore e una minore del raggio diΓ1; un puntoe quindi interno e uno esternoe qualunque circonferenzaΓ2 passante perC eD interseca necessariamente la circonferenzaΓ1.

1988.3 Si considerino i numeri naturali 1, 11, 111,. . . , e in generale si indichi conαn il numero chesi ottiene giustapponendon cifre uguali a 1.(a) Si provi che seαn e un numero primo alloran e primo.(b) Si provi che, assegnato comunque un numero naturaler, non divisibile ne per 2 ne per 5, si puo trovare

unαn multiplo di r.(c) Si scriva un algoritmo o programma per calcolatore (in un qualunque linguaggio di programmazione)

che, a partire dar, calcoli il minimon per cui vale la (b).

RISOLUZIONE Scriviamo gliαn in una forma piu maneggevole:

αn = 1 + 10 + 100 +. . . + 10n−1 =

(1 + 10 + 100 +. . . + 10n−1) (10− 1)/9 = (10n − 1)/9 .

(a) Supponiamo per assurdo che sian = p q, conp, q interi > 1; allora

αn =10pq − 1

9=

10p − 19

(10p(q−1) + . . . + 10p + 1

)

e un numero composto.

(b) Dobbiamo provare che ser e un intero non divisibile ne per 2 ne per 5, allora esistek tale cher divideαk. Consideriamo, al variare din, i numeriαn modr (i resti degliαn nella divisione perr); questi sonoinfiniti numeri, tutti compresi fra 0 er − 1, e quindi dovranno esistere due interin em, conn > m, talicheαn modr = αm modr. Si ha quindi cheαn − αm = r s cons intero:

Anno 1988 79

αn − αm =10n − 1

9− 10m − 1

9=

10n − 10m

9=

10m 10n−m − 19

= 10mαn−m = r s ;

mar e primo con 10 e quindi deve dividereαn−m.

(c) Il programma seguente (in qbasic Microsoft) termina, per quanto dimostrato nel punto (b), stampandoil minimo indicek, il corrispondenteαk e il quoziente fra questo er.

DEFINT a-z ’ le variabili sono intere

INPUT "r: ", r

k = 1 ’ inizializzazioni

a = 1 ’ a(1) = 1

ciclo: IF r * INT( a / r ) = r THEN GOTO fine

k = k + 1 ’ prova l’indice successivo

a = 10 * a + 1 ’ calcola il relativo a(k)

GOTO ciclo

fine: PRINT k, a, a / r

END

1988.4 Sia assegnato su un piano un numeron arbitrario di triangoli con la proprieta che tre qualsiasidi essi abbiano almeno un punto in comune. Si dimostri che tutti i triangoli assegnati contengono unostesso punto. Come occorre modificare l’ipotesi perche la stessa conclusione valga per un numero finito ditriangoli nello spazio?

RISOLUZIONE La tesi e vera pern = 3; supponiamo che sia vera per ogni insieme din − 1 triangoli edimostriamo con un ragionamento per assurdo che deve valereanche per un insieme din triangoli.Siano assegnati i triangoliT1, . . . , Tn−1, Tn con la proprieta che tre qualsiasi di essi abbiano intersezionenon vuota, e supponiamo per assurdo cheC = T1 ∩ · · · ∩ Tn−1 eTn siano disgiunti.Dato cheC e Tn sono dueinsiemi convessi disgiunti nel piano, esiste una rettar che li lascia insemipiani opposti.Dato pero che per ogni scelta dii e j risultaTi ∩ Tj ∩ Tn 6= ∅, l’insiemeTi ∩ Tj contiene punti che stanno daparti opposte rispetto ar, e quindi la famiglia di segmentiSi = Ti ∩ r gode della proprieta cheSi ∩ Sj 6= ∅per ognii e j. Come dimostreremo fra un attimo, gli intervalliS1, . . . , Sn−1 hanno intersezione non vuota,e quindiC ∩ r e diverso dal vuoto, contro l’ipotesi.Il risultato per i segmentie ovvio nel cason = 2; per il caso generale, ancora per assurdo, supponiamo cheS1, . . . , Sn−1 abbiano intersezione non vuota e disgiunta daSn. Allora e possibile trovare un puntoP surche lascia da parti opposte il segmentoSn e l’intersezione degli altri. Ma cio e assurdo, percheSi ∩ Sn enon vuoto e dunque il puntoP appartiene a tutti gliSi.

Una analoga argomentazione si puo ripetere nello spazio tridimensionale se si hannon triangoli tali chequattro qualunque di essi abbiano un punto in comune, “separando” con un piano l’intersezione dei primin − 1 dall’n-mo e utilizzando il risultato precedente nel piano.Nello spazio se si hanno dei triangoli che hanno a tre a tre intersezione non vuota la proprieta e falsa, comemostra l’esempio dato dalle quattro facce di un tetraedro, cosı come nel piano non basta che siano non vuotesolo le intersezioni fatte a due a due (fig. 1).

80 I problemi della Scuola Normale

Figure 1, 2

Si noti che la sola proprieta utilizzata dei triangolie quella di essere una figura convessa. In effetti la tesie vera nel piano per figure convesse qualunque con a tre a tre punti in comune e nello spazio per figureconvesse (anche tridimensionali) che hanno a quattro a quattro un punto in comune. Il requisito dellaconvessita e fondamentale, come mostra la Figura 2.

1988.5* Sia p(x) = a x4+ b x3+ c x2+ d x + e un polinomio con coefficienti razionali. Si supponga che,per ogni interom maggiore di un certom0, il numerop(m) sia intero. Si dimostri che allora24a e unnumero intero.Si generalizzi questo risultato a polinomip(x) con coefficienti razionali di grado qualsiasi.

RISOLUZIONE Affrontiamo subito il caso generale: sepn(x) e un polinomio a coefficienti razionali cheassume valori interi per ogni interom ≥ m0, allora le stesse proprieta valgono anche per il polinomiopn(x + 1), e anche il polinomio differenza

pn−1 = pn(x + 1)− pn(x)

ha coefficienti razionali e assume valori interi per ogni interom ≥ m0. Se il termine di grado massimo dipn(x) ea xn, e immediato verificare che il termine di grado massimo dipn−1 saran a xn−1. Analogamente,se si considera

pn−2 = pn−1(x + 1)− pn−1(x) ,

si otterra un polinomio con le stesse proprieta, di gradon − 2 e coefficiente direttoren (n − 1)a.Cosı proseguendo, dopon passi si otterra un polinomio di grado 0,p0(x), con “coefficiente direttore”n (n − 1) · · ·3 · 2 · 1a = n! a. Dato chep0(x) e costante, ede intero per ogni interom ≥ m0, otteniamo chen! a e intero.Nel cason = 4 si ha che 24a = 4!a e intero.

Anno 1988 81

1988.6* Un laboratorio deve organizzare il trasferimento di10 m3 di scorie radioattive liquide. Occorreordinare un numeron di contenitori, identici, che possano contenere tali scorie e garantire un trasportosicuro.Si stima che il costo di ciascuno di tali contenitori sia16V 2 milioni di lire, oveV e il volume (inm3) discorie che ciascuno di essi puo contenere; il costo di riempimento di ciascun contenitorerisulta essere diun milione di lire, indipendentemente dalla sua capienza.(a) Quanti contenitori e di quale volume dovra ordinare il laboratorio per spendere il meno possibile?(b) Al momento di effettuare l’ordine si viene a sapere che la ditta fornitrice pratica sul prezzo dei contenitori

ordinati un piccolo sconto delk per cento, conk intero, se il loro numero uguaglia o supera le 50 unita(il costo del riempimento rimane inalterato). Quale il piu piccolok per il quale risulta convenientemodificare l’ordine, e perche?

RISOLUZIONE

Per il punto (a), se si indica conn il numero dei contenitori, il costo del trasferimentoe, in milioni di lire,

C = 16V 2 n + n ;

poiche V = 10/n si deve rendere minima la quantita

C = 16n

(10n

)2

+ n =1600

n+ n .

Poiche i due numeri 1600/n e n hanno prodotto costante, il valore minimo della loro somma si ha quandosono uguali, ovvero pern = 40. Il costo complessivoe quindiC = 80 milioni.

Per il punto (b) notiamo che sek e la percentuale di sconto praticata, il nuovo costoe

C1 = 1600 (1− k

100)

1n

+ n =16 (100− k)

n+ n

e affinche tale costo sia inferiore al precedente occorre che per qualchen ≥ 50 sia

16 (100− k)n

+ n < 80 ,

ovvero

n2 − 80n + 16 (100− k) < 0 .

Tale disequazionee soddisfatta nell’intervallo tra le due radici del trinomio, che sono 40± 4√

k. Affincherisulti conveniente utilizzare lo sconto, che si applica solo per n ≥ 50, occorrera dunque che risulti4√

k > 50− 40, ovverok > 100/16. Dato chek deve essere intero, lo sconto minimo per cui vale la penadi modificare l’ordineek = 7 (anche se il risparmio che si ottienee solo di 240.000 lire!).

82 I problemi della Scuola Normale

1989.1* Per 0 ≤ x ≤ π/2 en = 0, 1, 2, . . . , si ponga:

Fn(x) = 1− sinx + sin2 x − . . . + (−1)n sinn x.

Provare che:(a) per 0 ≤ α ≤ 1/2, l’equazioneF2k = α non ha soluzioni qualunque siak naturale;(b) per 1/2 < α ≤ 1, esiste un numero naturalek∗ tale che, per ognik > k∗, l’equazioneF2k = α ha

almeno due soluzioni.

RISOLUZIONE Cominciamo con l’osservare cheFn(x) · (1 + sinx) = 1 + (−1)n sinn+1 x e che dunque

F2k =1 + sin2k+1 x

1 + sinx.

Per il punto (a) notiamo che se 0≤ x < π/2 risulta

F2k = (1 + sin2k+1 x)/(1 + sinx) ≥ 1/(1 + sinx) > 1/2

mentre perx = π/2 e

F2k = 1 > 1/2 ;

di conseguenza l’equazioneF2k = α, conα ≤ 1/2, non ha soluzioni su [0, π/2].

Per il punto (b), studiamo il comportamento delle funzioniF2k al crescere dik. Per ognix con 0≤ x < π/2la successione di numeriF2k(x) tende decrescendo a 1/(1 + sinx). La funzioneg(x) = 1/(1 + sinx) e unafunzione continua, decrescente con minimo 1/2 assunto nel puntox = π/2.

π/2 x

1/2

1

Figura 1

Per ogniα con 1/2 < α ≤ 1, esiste un puntox0 conx0 < π/2 per cui si hag(x0) = α.Se si considera il punto

x =12

(x0 +

π

2

)

si avra cheg(x) < g(x0) = α e siccomeF2k(x) tende ag(x), da un certok∗ in poi si avra F2k(x) − g(x) <α − g(x), ovveroF2k(x) < α.SiccomeF2k(0) = F2k(π/2) = 1, per ilteorema di esistenza dei valori intermedi vi sarannoalmeno due punti,x1 e x2, il primo nell’intervallo (0, x), il secondo nell’intervallo (¯x, π/2), per i qualiF2k(x1) = F2k(x2) = α.

Anno 1989 83

1989.2 SiaS una superficie sferica di centroO. Per ogniP ∈ S, siafP : S → S l’applicazione che adogni Q ∈ S associa il puntofP (Q) simmetrico diQ rispetto all’asseOP . Dimostrare che:(a) per ogniP ∈ S l’applicazionefP e composizione di due simmetrie rispetto a piani;(b) per ogniP , X, Y ∈ S la distanza traX eY e uguale alla distanza trafP (X) efP (Y ).(c) per ogniP , Q, X ∈ S si ha chefP (fQ(X)) = ffP (Q)(fP (X)).

RISOLUZIONE L’applicazionefP associa ad ogniQ il puntofP (Q) simmetrico rispetto all’asseOP ; quindil’asseOP e perpendicolare al segmentoQfP (Q) nel suo punto medio, e di conseguenza il puntofP (Q)giace sulla circonferenza intersezione diS col piano perpendicolare adOP passante perQ, e dunquefP (Q)appartiene adS.(a) Si considerino due pianiΠ′ e Π

′′ passanti perOP e fra loro perpendicolari. DatoQ suS, siaQ′ ilpunto simmetrico diQ rispetto aΠ

′ e Q′′ il simmetrico diQ′ rispetto aΠ′′. Se si taglia la figura con un

piano perpendicolare aOP e passante perQ, tale piano contiene siaQ′ siaQ′′ e seC e la sua intersezioneconOP si ha

Q Q ′

Q ′′

C

Figura 1

QCQ′ + Q′CQ′′ = (180 − 2CQ′Q) + (180 − 2CQ′Q′′) =

360 − 2 (CQ′Q) + CQ′Q′′) =

360 − 2 · 90 = 180 .

Pertanto i puntiQ, C, Q′′ sono allineati e si ha ovviamenteCQ = CQ′ = CQ′′; dunqueQ′′ coincide con ilsimmetrico diQ rispetto alla rettaOP , cioeQ′′ = fP (Q).

(b) E una conseguenza del punto precedente: in una simmetria rispetto a un pianoα le distanze siconservano perche il quadrilatero di verticiX, Y , Y ′, X ′ (fig. 2), essendo simmetrico rispetto aα, e untrapezio isoscele.

α

X

Y

Y ′

X ′

Figura 2

84 I problemi della Scuola Normale

(c) Iniziamo osservando che seP e Q sono allineati conO (Q coincide conP o col suo simmetricorispetto adO) la tesi e ovvia, perche in tal casofP (Q) = Q, fP = fQ e fP (fQ(X)) = fP

2(X) = X =fQ(fP (X)) = ffP (Q)(fP (X)) per ogniX. Per il caso in cuiP e Q non sono allineati, ricordiamo che perogniK ∈ S l’applicazionefK e una isometria; inoltre, in quanto composizione di simmetrie, pensata cometrasformazione di tutto lo spazio in se, e lineare:fK(x + y) = fK(x) + fK(y) e fK(αx) = αfK(x), per ognicoppia di vettori dello spaziox, y e per ogni numero realeα.E’ quindi sufficiente provare che la tesi vale per tutti i punti del pianoσ passante perO, P e Q, e per unaltro puntoT , non inσ, che possiamo prendere suS, e tale cheOT sia ortogonale aσ.

Proviamo chefP (fQ(X)) = ffP (Q)(fP (X)) vale per ogniX di σ. Indichiamo per semplicita di notazione conR il punto fP (Q). Sul pianoσ l’azione difP , fQ, fR e quella di una simmetria rispetto alle rette perOP ,OQ, OR; conseguentemente la composizionefP fQ agisce come una rotazione intorno adO di un angolopari al doppio di quello formato dalle semiretteOQ eOP ; il puntoR e il simmetrico diQ rispetto alla rettaOP , dunque l’angolo fraOP eOR e in valore assoluto e segno uguale a quello formato dalle semiretteOQeOP ; dunque le due trasformazionifP fQ e fR fP coincidono, e la tesi vale certamente per ogni puntoX appartenente aσ.

Indichiamo conT ′ il simmetrico diT rispetto adO. L’applicazionefQ mandaT nel simmetrico rispetto adOQ, e quindi inT ′, dato cheOT eOQ sono ortogonali, e lo stesso fafP conT ′; a primo membro abbiamodunquefP (fQ(T )) = T ; per il secondo membro, essendoP , Q efP (Q) complanari,OT e ortogonale ancheadOfP (Q) e quindiffP (Q)(fP (T )) = ffP (Q)(T ′) = T , e cio conclude la dimostrazione.

Anno 1989 85

1989.3 Trovare le soluzioni reali del sistema:

2y + x − x2 − y2 = 0

z − x + y − y (x + z) = 0

− 2y + z − y2 − z2 = 0

RISOLUZIONE Sottraendo termine a termine la prima e la terza equazione otteniamo

4y + x − z − x2 + z2 = 0, da cui y =14

(x − z) (x + z − 1).

Sostituiamo l’espressione pery nella seconda equazione, ottenendo che

(z − x) +(x − z) (x + z − 1)

4(1− x − z) = 0,

e quindi

(z − x)[1 +

(x + z − 1)2

4

]= 0,

che equivale a

z = x .

Di conseguenza, dall’espressione pery, sappiamo chey = 0; ricaviamo finalmentex sostituendo nella primaequazione, che diventax − x2 = 0; le soluzioni reali del sistema sono date da:

x = 0, y = 0, z = 0 ;

x = 1, y = 0, z = 1.

1989.4* Consideriamo la legge che ad ogni puntoP = (x, y) del piano cartesiano fa corrispondere ilpuntof (P ) dello stesso piano, definito da:

f (P ) =

P se OP ≤ 1( x

OP,

y

OP

)se OP ≥ 1

doveO = (0, 0) eOP indica la distanza daO a P .Provare che per ogni coppia di puntiP eQ la distanza fraf (P ) ef (Q) non supera la distanza fraP eQ.

RISOLUZIONE Notiamo anzitutto che l’applicazionef associa ad ogni puntoP del cerchioC di centrol’origine e raggio unitario il punto stesso, mentre ad ogni puntoP esterno associa il punto intersezione delbordo di detto cerchio con la semirettaOP .SeP e Q appartengono al cerchioC si haf (P ) = P , f (Q) = Q e quindi, ovviamente dist(f (P ), f (Q)) =dist(P, Q).Se uno dei due punti, ad esempioP , appartiene aC, e l’altro e esterno (fig. 1) l’angoloP f (Q)Q e ottuso equindi

dist(f (P ), f (Q)) = dist(P, f (Q)) < dist(P, Q)

in quanto in un triangolo ottusangolo il lato opposto all’angolo ottusoe maggiore degli altri lati.

86 I problemi della Scuola Normale

yy

x x

O O

P f( )= P

Qf(Q)

P

f( )P

QQ′f(Q)

Figure 1, 2

Se poi siaP cheQ sono esterni al cerchioC, si consideri la circonferenza che passa per il punto piu vicinoall’origine, diciamoP , e siaQ′ la sua intersezione con la semirettaOQ. I triangoli di verticiO, f (P ), f (Q)e O, P , Q′ sono simili e isosceli; essendo il segmento di estremiO, f (P ) minore del segmento di estremiO, P si avra

dist(f (P ), f (Q)) < dist(P, Q′) ,

l’angoloPQ′Q e ottuso (in quantoOPQ′ e isoscele) e quindi per la stessa considerazione precedente

dist(P, Q′) < dist(P, Q) .

In conclusione, sie provato che in ogni caso

dist(f (P ), f (Q)) ≤ dist(P, Q) .

1989.5 SiaS la superficie di un prisma a base ottagonale regolare inscritto in un cilindro circolareretto di raggioR e altezzah = 3R sin(π/8). SianoA eB due punti come in figura, conOA = R/

√2.

A O

B

P

N M

Determinare la lunghezza del minimo percorso suS tra A eB.

RISOLUZIONE Un percorso minimo che congiungeA con B corrisponde ad un cammino rettilineo checonnetteA eB nello sviluppo piano del prisma, cammino interamente contenuto nelle facce dello sviluppo.E chiaro che vi sono solo due possibilita da prendere in considerazione, corrispondenti allo sviluppo chevede la faccia superiore del prisma connessa al latoNM (nel qual caso il percorso taglia solo lo spigoloNM )

Anno 1989 87

Q

P A

H O

N M

K B

Figura 1

e allo sviluppo che ha la faccia superiore connessa al latoPN (e allora il percorso taglia sia lo spigoloPNsia lo spigolo laterale uscente daN ).

O

AQ

P L N MH

B

Figura 2

Confrontiamo la lunghezza di tali percorsi. Postoα = π/8, si ha che la lunghezza del lato dell’ottagonoe2R sinα e la lunghezza dell’apotemaeR cosα.Osserviamo che essendoON = R, OA = R/

√2 eAON = π/4 il triangoloONA (fig. 2) e rettangolo inA e

isoscele. Ne segue cheANL = α e AL = R sinα/√

2 = LH. Pertanto la retta che passa per i puntiA e Hforma un angolo diπ/4 col lato orizzontalePN e quindi incontrera la vertical uscente daM in un puntoXtale cheXM = MH = 3R sinα, ovvero nel puntoB. Pertanto il segmentoAB e parallelo al latoQP dellabase (fig. 2) e dunque il suo prolungamento incontraQO in un puntoA′ che dista daO una quantita pari adOA (fig. 3). Il segmentoA′B′ ha quindi la stessa lunghezza del segmentoAB della Figura 1.

B ′ B

K

P

AQ

A ′

O

Figura 3

In conclusione ci siamo ridotti a confrontare le lunghezze dei segmentiAB eA′B′ della Figura 3.

88 I problemi della Scuola Normale

Si mandi la perpendicolareA′K adAB; si ha

KB =B′B√

2=√

2R sinα = AA′ ,

ma l’ipotenusaA′B′ del triangolo rettangoloA′KB′ e maggiore del catetoA′K e pertanto

A′B′ > A′K = AK + AA′ = AK + KB = AB .

Possiamo cosı concludere che il percorsoAB della Figura 2e minore del percorsoAB della Figura 1.

1989.6 Siaf (x) una funzione a valori reali definita sulla semiretta realex ≥ 0. Supponiamo che:(a) f (x) sia derivabile con derivataf ′ continua;(b) f (0) = 0;(c) per ognix ≥ 1 risulti

0 < f (x) ≤ xf ′(x).

Provare che l’equazionef (x) = k ha almeno una soluzionex ≥ 0, per ognik ≥ 0 .

RISOLUZIONE Proveremo che la funzionef (x) tende a +∞ al tendere dix a +∞. Essendof (0) = 0, dalteorema dei valori intermedi seguira che essa assume almeno una volta ogni valorek ≥ 0.La funzioneg(x) = f (x)/x ha derivata

g′(x) =f ′(x) x − f (x)

x2

non negativa su[1, +∞); ne segue cheg(x) e non decrescente su[1, +∞) e dunque si avra

g(x) ≥ g(1) = f (1) > 0 .

Pertanto per ognix > 1 si ha

f (x) ≥ f (1)x

e quindi

limx→+∞

f (x) = +∞ .

1990.1* Considerare nello spazio euclideo nove punti distinti a coordinate intere. Dimostrare che neesistono due tali che il segmento che li congiunge contiene almeno un punto interno (cioe distinto dagliestremi) a coordinate intere.

RISOLUZIONE Come prima osservazione, notiamo che con meno di nove punti esistono controesempi;basta prendere gli otto punti distinti dello spazio che hanno come possibili coordinate 0 oppure 1; nessunsegmento che abbia come estremi due di questi punti ha punti interni a coordinate intere.SeP1 = (x1, y1, z1) e P2 = (x2, y2, z2) hanno coordinate intere, il segmentoP1P2 contiene certamente unpunto interno a coordinate intere, il suo punto medio, sex1 +x2, y1 +y2 ez1 +z2 sono tutti e tre pari, e questoaccade sex1 ex2 sono entrambi pari o entrambi dispari, e analogamente per lealtre coordinate. Le possibilicombinazioni di parita o disparita per le tre componenti sono solo 23 = 8 e dunque, per una qualsiasi sceltadi 9 punti a coordinate intere, ne esisteranno almeno due,Q e R tali che le relative componentix sianonella stessa classe di parita e cosı per le componentiy e z. QuindiM , punto medio diQ e R, sara internoal segmentoQR, e avra coordinate intere.

Anno 1990 89

1990.2 Sia P un poligono semplice (cioe tale che da ogni vertice escono esattamente due lati) nonnecessariamente convesso, con almeno4 lati. Supponiamo cheP abbia al piu un vertice concavo.E’ vero che esistono due vertici non consecutivi con la proprieta che il segmento che li congiungee contenutoin P? In caso affermativo dimostrarlo, altrimenti trovare un controesempio.

RISOLUZIONE Se il poligonoe convesso ogni diagonalee interamente contenuta in esso; se none convessosiaP il vertice del suo solo angolo concavo e sianoA e B i vertici adiacenti ad esso; due dei semipianiindividuati dalle rette perA, P e perB, P tagliano il poligono in due figure convesse e una almeno diesse contiene un quarto verticeC; dunque il segmentoPC congiunge due vertici non consecutivi edeinteramente contenuto nel poligono.

A

B

P

C

Figure 1, 2

Si noti che se il poligonoe intrecciato la proprieta none piu vera, come mostra la Figura 2.

90 I problemi della Scuola Normale

1990.3 Dato il sistema

x1 + x2 + . . . + x100 = 5050

x22 − x2

1 = 3

· · · ·x2

k − x2k−1 = 2k − 1

· · · · · ·x2

100− x299 = 199

trovare tutte le soluzionix1, x2, . . . , x100, conxk ≥ 0, k = 1, 2, . . . , 100.

RISOLUZIONE Sommando le relazionix22 − x2

1 = 3 ,x23 − x2

2 = 5, . . . , x2k − x2

k−1 = 2k − 1 possiamo ricavarexk in funzione dik e della solax1, infatti

x2k − x2

1 = (x2k − x2

k−1) + (x2k−1 − x2

k−2) + . . . + x22 − x2

1 =

(2k − 1) + (2k − 3) + . . . + 3

e quindi

x2k − x2

1 = (k − 1)(2k − 1) + 3

2= k2 − 1

da cuixk =√

k2 − 1 +x21 ; si vede facilmente chexk e una funzione strettamente crescente dix1. Inoltre,

sex1 = 1 si avraxk = k e quindix1 +x2 + . . . +x100 = 100· 101/2 = 5050, per cuie verificata anche la primaequazione.Non vi sono altre soluzioni perche sex1 < 1 la somma delle cento incognite risulta strettamente minore di5050, e sex1 > 1 risulta strettamente maggiore.

Anno 1990 91

1990.4 Sia dato il polinomioF (x) = xn + an−1xn−1 + . . . + a0 con coefficientiai interi. Supponiamo

che esistano quattro interi distintia, b, c, d tali cheF (a) = F (b) = F (c) = F (d) = 7.Dimostrare che non esiste nessun numero interok tale cheF (k) = 12.

RISOLUZIONE Dato che il polinomioF (x) assume il valore 7 ina, b, c, d il polinomio G(x) = F (x) − 7 siannulla in tali punti e dunque per ilteorema di Ruffini si ha

G(x) = (x − a) (x − b) (x− c) (x − d) G1(x)

conG1(x) polinomio a coefficienti interi. Provare che perk interoF (k) non puo valere 12 equivale a provarecheG(k) non assume il valore 5. MaG(k) = (k − a) (k − b) (k − c) (k − d) G1(k) e, dato chea, b, c, d sonotutti distinti, almeno due fra i numerik − a, k − b, k − c, k − d sono diversi da±1 e quindiG(k) non puomai valere 5, chee un numero primo.

1990.5 Trovare il piu piccolo numeroα > 1 tale che risulti:

α + sinx

α + siny≤ ey−x per ognix, y conx ≤ y

RISOLUZIONE Daα > 1 segue cheα+siny e sempre positivo; separandox ey otteniamo la diseguaglianzaequivalente

(α + sinx) ex ≤ (α + siny) ey per ognix, y conx ≤ y

che a sua volta equivale al fatto che la funzionefα(t) = (α + sint) et e crescente.La funzionefα(t) e derivabile; risultera crescente se e solo se

f ′α(t) = (cost + α + sint) et ≥ 0 .

Deve dunque essereα ≥ −(cost + sint) per ognit, e quindi il piu piccoloα per cui vale la diseguaglianzasara dato dal massimo dei valori assunti da

−(cost + sint) = −√

2 sin(t + π/4) ,

che si ha perx = −(3/4)π + 2k π e vale√

2.

92 I problemi della Scuola Normale

1990.6* Si consideri un rettangoloR di misure6 × 4 metri. Nel punto di mezzoO di un lato eincernierato un braccio articolato, della lunghezza totale di due metri, formato da due segmentiOA eAB(vedi figura). Il braccio puo muoversi solo all’interno diR. Piu precisamente il segmentoOA puo ruotareintorno al punto fissoO e, per ogni posizione assunta, il segmentoAB puo ruotare intorno al puntoA;naturalmente, durante il movimento, il braccio deve restare inR.

OA

B

R

E’ possibile scegliere le lunghezze dei segmenti in modo cheogni punto diR a distanza minore o uguale adue metri daO sia raggiunto daB?

RISOLUZIONE Indichiamo conr la lunghezza del braccioOA e con r′ quella del braccioAB. Si haOA + AB = 2 m.Per la diseguaglianza triangolare si ha

|OA − AB| ≤ OB ≤ OA + AB

cioe

|r − r′| ≤ OB ≤ r + r′ .

E quindi chiaro che ser 6= r′ i punti interni del semicerchio di centroO e raggio|r− r′| interno al rettangoloR non sono raggiungibili dal braccio (fig. 1 e 2).

O

A B

R

O

A

B

R

Figure 1, 2

Quindi la sola possibilita e che siar = r′ = 1 m. Proviamo che in questo caso il problema si puoeffettivamente risolvere.Preso un qualunque puntoB del semicerchioC di raggio 2 m e centro inO, si costruiscano i due triangoliisosceliOAB, OA′B di baseAB e aventi latiOA = AB = OA′ = A′B = 1 m. I puntiA eA′ sono simmetricirispetto alla rettaOB, cioe ciascuno dei due appartiene ad una delle due semicirconferenze di centro inO eraggio 1 m suddivise dalla retta perO e B, il che implica che uno dei puntiA, A′ e interno al semicerchioC e quindi l’intero triangolo formato da tale punto e dai puntiO e B e interno aC, pertanto il problemaerisolto.

Anno 1990 93

O

AB

A ′

R

Figura 3

94 I problemi della Scuola Normale

1991.1 Provare che per ogni numero interon ≥ 2 si ha

n√

n! <n + 1

2

e che(n + 1)/2 none mai un multiplo intero din√

n!.

RISOLUZIONE Applichiamo ladiseguaglianza fra le medie geometrica e aritmetica,√

ab <(a + b)/2 , valida per ogni coppiaa, b di numeri positivi diversi fra loro, ai prodotti 1· n, 2 · (n − 1), . . . ,(n − 1) · 2, n · 1 ; moltiplicando len diseguaglianze otteniamo

n! =√

1 · n√

2 · (n − 1) . . .√

(n − 1) · 2√

n · 1 <

1 +n

22 + (n − 1)

2· · · (n − 1) + 2

2n + 1

2=

(n + 1

2

)n

da cui, estrendo la radicen-esima, otteniamo la diseguaglianza richiesta.

Per il secondo punto dobbiamo provare che per nessunk intero vale l’uguaglianza

kn√

n! =n + 1

2o, equivalentemente, che none mai verificata

(2k)n · n! = (n + 1)n .

Ma l’ultima uguaglianza non puo essere soddisfatta per nessunk perchen en+1 sono sempre primi tra loro,e quindi ogni numero primo che dividen divide il primo membro dell’uguaglianza ma non puo dividere ilsecondo membro.

Anno 1991 95

1991.2 Fra tutti i quadrilateri convessi inscritti in un quadrato,in modo che ogni lato del quadratocontenga almeno un vertice del quadrilatero, si determinino quelli aventi minimo e massimo perimetro.

RISOLUZIONE Determiniamo per prima cosa i quadrilateriMNOP inscritti di perimetro massimo.

M B

P

A

D CO

N

Figura 1

Per la diseguaglianza triangolare abbiamo che

MN ≤ MB + BN , NO ≤ NC + CO ,

OP ≤ OD + DP , PM ≤ PA + AM ,

da cui, sommando, otteniamo

MN + NO + OP + PM ≤ AB + BC + CD + DA .

Dunque, ogni quadrilatero inscritto inABCD ha perimetro non superiore a quello diABCD, e l’uguaglianzasi ottiene scegliendo i vertici diMNOP coincidenti con quelli del quadrato.

A M 1 B A ′

D ′D O C O ′

P

2

3

4

N

N ′

Figura 2

Cerchiamo ora di determinare quali siano i quadrilateri chehanno il perimetro minimo fra quelli inscritti.Denotiamo con 1, 2, 3, 4 i latiAB, BC, CD, DA del quadrato e costruiamo l’immagine speculare diABCDrispetto al segmento 2; seN ′ e l’intersezione col lato 2 della retta perM e perO′, immagine diO, dalladiseguaglianza triangolare si haMN ′ + N ′O′ = MO′ ≤ MN + NO′ = MN + NO.Pertanto il percorso di lunghezza minima daM aO si avra quandoN coincide conN ′.Alla stessa stregua riflettiamo il quadrato attorno al lato 3e poi al lato 4.

96 I problemi della Scuola Normale

MA MA A ′

A ′′ M ′′′

Figure 3, 4

Il percorso di lunghezza minima che connette il puntoM con la sua immagineM ′′′ dopo le tre riflessionie quello rettilineo. Siccome ovviamenteAM = A′′M ′′′ la retta perM eM ′′′ forma angoli di 45 con i latidel quadrato e quindi tale percorso ha lunghezza 2

√2AB. Sul quadrato originario cio corrisponde a un

percorso che daM torna aM dopo aver toccato i lati 2, 3, 4 lungo i lati di un rettangolo con lati parallelialle diagonali del quadrato. La lunghezza di tale percorso non dipende dalla posizione del puntoM suABe quindi il problema ammette infinite soluzioni (fig. 6).

M

Figure 5, 6

1991.3* Trovare il piu piccolo numero interoN0 ≥ 1 con la proprieta cheN0 + 1 e 2N0 + 1 sianoentrambi quadrati perfetti.Mostrare poi che ogni intero N con questa proprieta e multiplo diN0.

RISOLUZIONE E’ piu comodo descrivere l’insieme degli interiN per cuiN + 1 e 2N + 1 sono quadratiperfetti come l’insieme degli interip2 − 1 tali che 2p2 − 1 e un quadrato perfetto. Per verifica diretta, il piupiccolo interop per cui 2p2 − 1 e un quadrato perfettoe 5, e il numeroN0 richiestoe 52 − 1 = 24.Per il secondo punto, dobbiamo provare che se 2p2 − 1 e un quadrato perfetto allorap2 − 1 e divisibile per24 = 23 · 3.Ma il numerop2 − 1 = (p + 1) (p − 1) e pari se e solo sep e dispari; in tal caso essoe prodotto di due pariconsecutivi e quindie divisibile per 8. Inoltre, uno e uno solo dei numerip − 1, p e p + 1 e divisibile per 3,e quindip2 − 1 e divisibile per 3 se e solo sep none divisibile per 3.Dobbiamo quindi far vedere che se 2p2 − 1 e un quadrato perfetto, allorap e dispari, e none divisibile per3.Supponiamo chep sia pari,p = 2k. Dividendo per 8 il numero 2p2 − 1 = 8k2 − 1 = 8 (k2 − 1) + 7 il restoe7, ma dall’identita

(4r + s)2 = (16r2 + 8r s + s2) = 8 (2r2 + r) + s2

vediamo che dividendo per 8 un quadrato perfetto i resti possibili sono solamente 0, 1 o 4. Ne deduciamoche, se 2p2 − 1 e un quadrato perfetto, allorap e dispari.

Anno 1991 97

Analogamente, supponiamo per assurdo chep sia multiplo di 3,p = 3k.sostituendo otteniamo 2p2 − 1 = 18k2 − 1 = 9 (2k2 − 1) + 8 e quindi dividendo 2p2 − 1 per 9 si ha resto 8,mentre i resti ottenibili da un quadrato perfetto, per l’identita (9r + s)2 = 9 (9r2 + 18r s) + s2, sono tutti esoli quelli che si hanno dai quadrati dei numeri da 0 a 8 e cioe, nell’ordine:

0, 1, 4, 0, 7, 7, 0, 4, 1 ,

e dunque se 2p2 − 1 e un quadrato, allorap none multiplo di 3.

In conclusione se 2p2 − 1 e un quadrato perfetto il numeroN = p2 − 1 e divisibile perN0 = 24.

1991.4 Su un treno, inizialmente senza passeggeri e formato dan carrozze, salgonok viaggiatoridisponendosi in modo casuale e indipendente l’uno dall’altro. Qual e la probabilita che solo tre carrozzesiano occupate da almeno un viaggiatore?

RISOLUZIONE I k passeggeri si distribuiscono sullen carrozze, che possiamo supporre contraddistinte dalleetichettec1, c2, . . . , cn, in un numero di modi distinti dato dank. Vogliamo calcolare il numero dei casiin cui solo le carrozzeci1 , ci2 e ci3 risultano occupate da almeno un passeggero. Qualunque scelta dellecarrozzei1, i2 e i3 con 1≤ i1 < i2 < i3 ≤ n porta allo stesso numero di disposizioni accettabili che siottiene peri1 = 1, i2 = 2 ei3 = 3 e quindi, indicando tale numero conS(k, 3), i casi favorevoli saranno datidaS(k, 3) moltiplicato per il numero di modi distinti di scegliere tre dellen carrozze, cioeCn,3 =

(n3

).

Calcoliamo quindi il numeroS(k, 3) di modi distinti in cuik passeggeri occupano 3 carrozze, senza lasciarnenessuna vuota. Ogni viaggiatore ha 3 scelte, il che porta ad un numero totale di 3k disposizioni; fra questepero vi sono anche quelle in cui nessun passeggero occupa la vettura c1 (pari a 2k possibilita), oppure lavetturac2 oppure ancora la vetturac3. Dobbiamo dunque escludere questi casi, che darebbero un totale di3 · 2k; tali casi pero non sono disgiunti: i tre casi in cui tutti i passeggeri salgono sulla stessa carrozza sonostati conteggiati due volte. Il numero di modi distinti in cui i k viaggiatori possono occupare le prime trecarrozzee quindi dato daS(k, 3) = 3k − 3 · 2k + 3.L’evento “solo 3 dellen carrozze hanno almeno un passeggero” ha quindi probabilita

P =

(n3

)S(k, 3)nk

=n (n − 1) (n − 2) (3k − 3 · 2k + 3)

6nk.

98 I problemi della Scuola Normale

1991.5 Costruire un polinomio (a coefficienti reali)

P (x, y) = a x2 + b x y + c y2

verificante le proprieta:(i) P (x, y) = 0 soltanto perx = y = 0;(ii) sex e y sono numeri interi allora ancheP (x, y) e intero.Determinare poi il massimo della quantita

∆ = b2 − 4a c

al variare diP nell’insieme dei polinomi soddisfacenti le proprieta precedenti.

RISOLUZIONE SeP (x, y) = 0 e y 6= 0, allorax/y e una soluzione reale dia t2 + b t + c = 0, e dunque ildiscriminante di tale equazioneb2 − 4a c e non negativo; analogamente, seP (x, y) = 0 ex 6= 0, alloray/x euna soluzione reale dic t2 + b t + a = 0, e dunque ancorab2 − 4a c ≥ 0.Un polinomioP verifica quindi l’ipotesi (i) se e solo seb2 − 4ac < 0.Per la seconda condizione, seP (x, y) e intero perx, y interi, avremo cheP (1, 0) = a, P (0, 1) = c eP (1, 1) = a + b + c sono tutti interi, e quindia, b, c debbono essere interi.La funzione∆ = b2 − 4a c assume solo valori interi negativi; il polinomioP (x, y) = x2 + x y + y2 verificale ipotesi (i) e (ii) e fornisce−3 come valore di∆. Vediamo se le equazioni∆ = −1 o ∆ = −2 ammettonosoluzione.∆ = b2−4a c = −1 equivale ab2 + 1 = 4a c, che none mai verificata percheb2 + 1 none mai divisibile per 4.∆ = b2−4a c = −2 equivale ab2 + 2 = 4a c, che none mai verificata percheb2 + 2 none mai divisibile per 4.Quindi il massimo valore della funzione∆ e−3, e si ottiene, ad esempio, perP (x, y) = x2 + x y + y2.

1991.6* Ci si propone di congiungere con una strada due localita A eB che distano 4 km, fra le qualisi trova una zonaZ costituita da terreno pietroso e avente la forma di un cerchio con centro nel punto mediodi AB e raggio di 1 km.(a) Sapendo che, a parita di lunghezza, il costo di costruzione della strada nella zona pietrosae λ volte

(λ numero reale maggiore di 1) quello relativo alla zona circostante, determinare due puntiP , Q sulbordo diZ in modo che il percorsoAPQB (vedi figura) sia il piu economico possibile.

A O B

PQ

Z

(b) Discutere poi il caso piu generale in cui si considerano percorsi formati, oltre cheda tratti rettilinei,anche da eventuali tratti curvilinei contenuti nel bordo diZ (dove il costo unitario di costruzione si puoconsiderare lo stesso che nella zona esterna aZ).

RISOLUZIONE

(a) Ci proponiamo anzitutto di dimostrare che il percorso cercatoe simmetrico rispetto alla perpendicolarer al segmentoAB passante per il centroO.Supponiamo che il percorso ottimale siaAPQB e siaM l’intersezione del segmentoPQ con la rettar. SiaC1 il costo di costruzione del trattoAPM eC2 quello trattoMQB. Si dovra avereC1 ≤ C2 oppureC1 > C2.Nel primo caso, siaP ′ il simmetrico di P rispetto ar; il percorsoMP ′B avra costoC1 e, essendoPP ′ < PM + MP ′, il percorsoAPP ′B avra costo inferiore a quello diAPMP ′B chee 2C1 ≤ C1 + C2.Allo stesso modo si ragiona seC2 < C1, considerando il percorsoAQ′QB, oveQ′ e il simmetrico diQrispetto ar.

Anno 1991 99

A O BZ

P

Q

r

M

P ′

Figura 1

In entrambi i casi si riesce quindi ad ottenere un costo di costruzione inferiore prendendo un percorsosimmetrico rispetto ar.Supponiamo dunquePQ parallelo adAB, indichiamo conH e H ′ i piedi delle perpendicolari daP e Q aAB e conx la lunghezza del segmentoOH. Le tangenti daA e daB al cerchio formano un angolo 30 conAB (AO ha lunghezza doppia del raggio).

A O BH H ′

P Q

x

Figura 2

La geometria del problema impone dunque la condizione

12≤ x ≤ 1 .

Si haAP =√

(2− x)2 + 1− x2 =√

5− 4x e dunque il costo di costruzione sara proporzionale a

F (x) = λx +√

5− 4x .

Cerchiamo i punti di minimo di tale funzione su [1/2, 1]. Si ha

F ′(x) = λ − 2√5− 4x

e dunqueF ′(x0) = 0 perx0 = 5/4− 1/λ2. Il puntox0 appartiene a (1/2, 1) solo se 2/√

3 < λ < 2, e in talcasoF (x) ha inx0 un massimo, dato che a sinistra dix0 si haF ′(x) > 0 mentre a destra dix0 e F ′(x) < 0.Pertanto il minimo diF (x) e da ricercarsi agli estremi dell’intervallo [1/2, 1].Si haF (1/2) = λ/2 +

√3, F (1) = λ + 1 e quindi seλ + 1 ≤ λ/2 +

√3, ovvero seλ ≤ 2 (

√3− 1), il percorso

ottimale sara quello rettilineoAB, seλ > 2 (√

3 − 1) il percorso piu economico sara costituito dai duesegmentiAT , BT ′ tangenti al cerchio e dal trattoTT ′.

100 I problemi della Scuola Normale

A B

P Q

Figure 4, 5

(b) Volendo considerare anche tratti curvilinei sul bordo di Z la sola possibilita da prendere in esameequella del percorso costituito dai segmentiAT , BT ′ e dall’arco

_TT ′. Si vede facilmente infatti che altri

percorsi avrebbero lunghezza, e quindi costo, maggiore.

A B

Figura 6

Il costo di costruzione del percorsoAT ,_TT ′ eT ′B e proporzionale a

2 · 2√

32

3= 2

√3 +

π

3

e dunque detto percorso avra costo inferiore a quelloATT ′B se

λ > 2√

3 +23

π .

Anno 1992 101

1992.1* E’ assegnata una legge che a ogni coppia di interix, y associa un interox y in modo che

x (y + z) = y x + z x

per tutti gli interi x, y, z. Si dimostri che

x y = x y (1 1).

RISOLUZIONE Osserviamo intanto che

x x = x (x + 0) = x x + 0 x ,

e quindi

0 x = 0 .

Di conseguenza,

x y = x (y + 0) = y x + 0 x = y x ;

dunque l’operazione e commutativa, e in particolare si ha che

x (y + z) = y x + z x = x y + x z

(vale la proprieta distributiva rispetto alla somma).Dimostriamo usando ilprincipio di induzione che sey ≥ 1 si hax y = y (x 1). Pery = 1 la tesie immediata; dalla proprieta distributiva segue che se

x y = y (x 1)

vale per un certoy, allora

x (y + 1) = x y + x 1 = y (x 1) +x 1 = (y + 1) (x 1) .

Sey = 0 si ha chex y = 0 = 0· (1 1).Sey < 0, osserviamo che

x y + x (−y) = x (y − y) = x 0 = 0 ,

e dunque

x y = −(x (−y)) = −((−y) (x 1)) = y (x 1) .

Possiamo quindi concludere:

x y = y (x 1) = y (1 x) = y (x (1 1)) = x y (1 1) .

102 I problemi della Scuola Normale

1992.2 Nel piano due quadratiABCD eA′B′C′D′ sono disposti come in figura.

A

BC

D A ′

B ′

C ′

D ′

Si dimostri che la retta passante perA e perpendicolare aDD′ incontra il segmentoBB′ nel punto medio.

RISOLUZIONE Si mandi perB′ la parallela adAB e siaQ il punto in cui essa interseca la perpendicolareAP aDD′.

A

BC

D

P

D ′

C ′

A ′

B ′

M

Q

Figura 1

Si considerino i triangoliADD′ eB′AQ. Si haQAB′ = DD′A perche entrambi gli angoli sono complemen-tari aPAD′ (D′AB′ e retto eAPD′ e un triangolo rettangolo). Dato che per costruzioneB′Q e paralleloadAB, l’angoloAQB′ e uguale aBAQ, e questo a sua voltae uguale aD′DA perche sono entrambi angolicomplementari aPAD.I triangoli ADD′ eB′AQ hanno quindi tutti gli angoli uguali e, avendo uguale anche un lato (AD′ = AB′),sono tra loro uguali. PertantoB′Q = AD = AB; ne segue che il quadrilateroABQB′ e un parallelogrammae dunque la diagonaleAQ biseca l’altra diagonaleBB′.

Anno 1992 103

1992.3 Verificare che la somma delle quarte potenze di due numeri reali di assegnato prodottop > 0a) decresce se decresce il valore assoluto della differenza dei due numeri;b) raggiunge il valore minimo quando i due numeri sono uguali.

RISOLUZIONE Supponiamo di aver due coppie,x1 e y1, x2 e y2, di numeri tali chex1 y1 = x2 y2 = p, e taliche sia

x41 + y4

1 ≥ x42 + y4

2 .

Sommando a primo e secondo membro

2p2 = 2x21 y2

1 = 2x22 y2

2 ,

otteniamo una diseguaglianza equivalente che possiamo scrivere come

(x21 + y2

1)2 ≥ (x22 + y2

2)2

che, ancora, equivale a

x21 + y2

1 ≥ x22 + y2

2 .

Sottraendo ora da primo e secondo membro il termine

2p = 2x1 y1 = 2x2 y2 ,

si ottiene la diseguaglianza

(x1 − y1)2 ≥ (x2 − y2)2 ,

che a sua voltae equivalente a

|x1 − y1| ≥ |x2 − y2| .Questo prova il punto (a).

Il punto (b) segue immediatamente dal fatto chex4+y4 e minima quando loe|x−y|, cioe quandox = y =√

p ;il minimo di x4 + y4 sara quindi dato da 2p2.

104 I problemi della Scuola Normale

1992.4 Mostrare che, per ogni intero positivo fissatok, esiste almeno un interon tale che

100≤ nk + n ≤ 101 +k nk−1.

RISOLUZIONE La funzionef (n) = nk + n e una funzione crescente din, qualunque sia l’intero positivok;inoltre essa tende all’infinito al crescere din. Siccomef (1) = 2, vi sara un minimo numero interon0 > 1per il quale 100≤ f (n0). Dato chen0 e il piu piccolo intero con tale proprieta, si avraf (n0−1) < 100, cioe

(n0 − 1)k + (n0 − 1) < 100. [1]

Per ogni numeroa > 1 si ha che

ak − k ak−1 ≤ (a − 1)k . [2]

Usufruendo di [1] e di [2] si ricava

nk0 − k nk−1

0 + n0 − 1 ≤ (n0 − 1)k + n0 − 1 < 100,

vale a dire

nk0 + n0 < 101 +k nk−1

0

e quindin0 e l’intero cercato.Per dimostrare la [2] si puo procedere per induzione suk. Ovviamente la proprieta e vera perk = 1; se sisuppone che la disuguaglianza sia soddisfatta perk − 1, cioe che valga

ak−1 − (k − 1)ak−2 ≤ (a − 1)k−1 ,

moltiplicando entrambi i membri pera − 1 si ottiene

ak − k ak−1 + (k − 1)ak−2 ≤ (a − 1)k ,

da cui

ak − k ak−1 ≤ (a − 1)k ;

dunque, in base al principio di induzione, la diseguaglianza e verificata per ogni intero positivok e per ogninumeroa > 1.

Anno 1992 105

1992.5* Sia F (x) = a3 x3 + a2 x2 + a1 x + a0 un polinomio di terzo grado con coefficienti interi. Sidimostri chei) sep/q, conp, q interi primi tra loro e q 6= 0, e una radice del polinomio, per ogni interom il numero

F (m) e divisibile perp − m q;ii) se esistono due interix1 e x2 tali cheF (x1) = 1, F (x2) = −1, e che|x1 − x2| > 2, allora F (x) non ha

radici razionali.

RISOLUZIONE Supponiamo che siaF (p/q) = 0; si avra che

a3 p3 + a2 p2 q + a1 p q2 + a0 q3 = 0

da cui segue cheq e un divisore dia3, cioe a3 = Aq conA intero, e chep e un divisore dia0, cioe a0 = C pconC intero.Siccomep/q e radice del polinomio, per ilteorema di Ruffini si avra F (x) = (x − p/q) P (x) conP (x) polinomio di secondo grado a coefficienti razionali, e quindi

F (x) = (q x − p) (a x2 + b x + c) [1]

cona, b, c razionali.Ma ovviamentea q = a3 = Aq, e quindia e intero; allo stesso modo−c p = a0 = C p e dunquec e intero.Siccome

a2 = q b − p a ,

a1 = q c − p b

sono numeri interi, ne segue che siaq b, siap b sono interi.Dato chep e q sono primi fra loro se ne deduce che ancheb e un numero intero.La [1] consente dunque di concludere che per ognim interoF (m) e divisibile perq m − p.

Per il punto (ii), ragioniamo per assurdo, supponendo chep/q sia una radice razionale diF (x); per il punto(i), F (m) e divisibile perp − m q. In particolare, per gli interix1 ex2 risulta:

1 = F (x1) e divisibile per p − x1 q

−1 = F (x2) e divisibile per p − x2 q

e quindip − x1 q = ±1, p − x2 q = ±1, da cui, tenendo conto del fatto chex1 6= x2 e q 6= 0, seguono duepossibilita

p − x1 q = −1

p − x2 q = 1

oppure

p − x1 q = 1

p − x2 q = −1

e sottraendo membro a membro otteniamo (x2 − x1) q = −2 oppure (x2 − x1) q = 2 e comunque|x2 − x1| =(2/q) ≤ 2, contro l’ipotesi. Dunque, nelle ipotesi del punto (ii),F (x) non ha radici razionali.

106 I problemi della Scuola Normale

1992.6*1) Dimostrare che, presi comunque tre vertici di un cubo, il triangolo da essi individuatoe rettangolo

oppure equilatero.2) Calcolare la probabilita che tre distinti vertici del cubo, scelti a caso, individuino un triangolo rettangolo

(la probabilita e il rapporto tra il numero di casi favorevoli e il numero di casi possibili).3) Si escludano tre vertici del cubo, e si considerino “ammissibili” i restanti cinque. Si indichi conP

la probabilita che tre vertici ammissibili del cubo, scelti a caso, individuino un triangolo rettangolo.Stabilire quanti valori puo assumereP al variare dei tre vertici esclusi all’inizio.

RISOLUZIONE

Per il primo punto osserviamo che, presi tre vertici del cubo, si hanno due casi:a) due vertici appartengono ad uno stesso spigolo; in tal caso il terzo vertice o appartiene a una delle facce

che confluiscono in tale spigolo, e allora il triangoloe la “meta” di tale faccia, o appartiene allo spigoloopposto, e in tal caso il triangoloe “meta” del rettangolo formato dai due spigoli e da due diagonalidelle facce.

Figura 1

In entrambi i casi il triangoloe rettangolo.b) Se nessuna coppia di vertici appartiene ad uno stesso spigolo la sola possibilita e che il triangolo sia

formato da tre diagonali delle facce, e dunque il triangoloe equilatero.

Figura 2

Fissato un vertice del triangolo, diciamoA, vi sono 7 possibilita per il verticeB e, fissato ancheB, altre 6possibilita per il terzo verticeC. Dunque vi sono 7·6 = 42 scelte possibili; i triangoli distinti con un verticein A sono di fatto la meta, in quanto per ogni scelta diB e di C vi e la possibilita di scambiare tali vertici.Dei 21 triangoli possibili, quelli formati da diagonali di facce sono solo 3. Infatti daA devono partire duelati, e possiamo sceglierli in 3 modi distinti decidendo quale delle 3 diagonali di facce escludere.Pertanto la probabilita che presi 3 vertici di un cubo essi formino un triangolo rettangoloe data da

21− 321

=67

.

La Figura 3 mostra i casi possibili.

Anno 1992 107

Figura 3

Per il terzo punto osserviamo che, esclusi i vertici non ammissibili, vi sono 5 scelte possibili per il primovertice, 4 per il secondo e 3 per il terzo, ma ovviamente una permutazione dei vertici da luogo allo stessotriangolo.Pertanto in ogni caso vi sono solo 5· 4 · 3/3! = 60/6 = 10 triangoli possibili.Se i tre vertici “proibiti” stanno su una stessa facciae possibile formare con i 5 vertici rimasti un solotriangolo con i lati che siano diagonali di facce. Pertanto la probabilita che un triangolo con verticiammissibili sia rettangoloe

10− 110

=910

.

Figura 4

La stessa situazione si ha se i tre vertici non ammissibili, pur non giacendo su una stessa faccia, sono taliche due di essi sono vertici di uno stesso spigolo.

Figura 5

108 I problemi della Scuola Normale

Se invece i tre vertici non ammissibili sono i secondi estremi degli spigoli che escono da un vertice,epossibile formare quattro triangoli, con i vertici ammissibili, che hanno i lati che sono diagonali di facce, epertando la probabilita cercatae

10− 410

=35

.

Figura 6

1993.1* Nel piano cartesiano riferito a coordinate ortogonali, un quadrato striscia in guisa che duesuoi vertici consecutivi appartengano rispettivamente all’asse delle ascisse non negative e a quello delleordinate non negative.Descrivere analiticamente la traiettoria di un puntoP interno al quadrato e rigidamente ancorato ad esso.Caratterizzare le posizioni diP nel quadrato per le quali la traiettoria appartiene ad una circonferenza,oppuree un segmento di retta.

AO x

D

y

PB

C

RISOLUZIONE Supporremo che il quadrato abbia lato unitario. SianoA eD i vertici del quadrato che sonoa contatto rispettivamente con gli assix ey e siaP un punto del quadrato che nella posizione iniziale (fig. 1)ha coordinate (a, b).

D

Cy

P

B

H A

xab

y

t

A

B

C

DP

a b t

x

Figure 1, 2

Anno 1993 109

Detto t l’angolo formato dal latoAD con l’assex e H il piede della perpendicolare calata daP suAD siavra che l’angolo formato daPH con la verticale daP e uguale at (fig. 2) e quindi l’ascissa diP e datadaa cost + b sint; analogamente si vede che l’ordinata diP e data da (1− a) sint + b cost. Abbiamo cosıottenuto le equazioni parametriche del luogo descritto dalpuntoP :

x = a cost + b sint

y = (1− a) sint + b cost .[1]

Consideriamo la [1] come un sistema lineare in cost, sint. Se risultaa (1− a) − b2 6= 0, tale sistema puoessere risolto in cost, sint ottenendo

cost =x (1− a) − y b

a (1− a) − b2, sint =

y a − x b

a (1− a) − b2.

Quadrando e sommando, la relazione cos2 t + sin2 t = 1 diviene

(x (1− a) − y b)2 + (y a − x b)2 =(a (1− a) − b2

)2,

ovvero

x2((1− a)2 + b2

)− 2x y b+y2

(a2 + b2

)=

(a(1− a) − b2

)2 [2]

Questae l’equazione cartesiana soddisfatta dai punti del luogo descritto daP (per valori positivi dix, y);essa rappresenta una circonferenza se e solo se il coefficiente del terminex y e nullo e i coefficienti dix2 ey2 sono uguali. Pertanto deve essere

b = 0 , (1− a)2 = a2 ,

ovveroa = 1/2, b = 0. Dunque l’unico punto del quadrato che si muove lungo un arco di circonferenzaeil punto medio del latoAD, e l’arco da esso descritto appartiene alla circonferenza di centro (0, 0) e raggio1/2.

O A x

D

y

P

Figura 3

Se invecea (1− a)− b2 = 0, allora le relazioni [1] comportano chex ey sono proporzionali e precisamentex

y=

a

b

e pertanto il puntoP si muove lungo un segmento di retta che passa per l’origine. Pensando il quadratonella posizione iniziale la condizionea (1− a) − b2 = 0 equivale a imporre che il puntoP appartenga allasemicirconferenza di equazionea2 + b2 = a, ossia (a − 1/2)2 + b2 = 1/4.

110 I problemi della Scuola Normale

A xO

D

P

y

Figura 4

D’altra parte in ogni posizione la circonferenza che ha centro nel punto medio diAD e raggio 1/2 passaperO e dunque l’angolo che il segmentoPO forma con l’asse dellex e l’angolo che sottende l’arcoPA edunquee costante, il che significa che, viceversa, ogni puntoP della semicirconferenza sopra descritta simuove lungo una retta. Si noti che in generale il luogo del punto P descritto dalle [2]e un arco di ellisse dicentro nell’origine (fig. 5).

O A x

D

y

P

Figura 5

Anno 1993 111

1993.2 In un piano cartesiano un oggetto puntiforme parte dal punto(0, 2n) (conn intero positivo)e scende fino all’asse delle ascisse compiendo2n passi, con la seguente regola: se prima di compiere unpasso si trova nel punto di coordinate intere(k, l), puo recarsi o in(k−1, l−1) o in (k +1, l−1) con ugualeprobabilita.Le mosse eseguite nei diversi passi sono indipendenti.Si indichi conpn(k) la probabilita che dopo2n passi l’oggetto si trovi nel punto(k, 0).i) Calcolarepn(k).ii) Mostrare che2pn(2) ≥ 1/(2n + 1).

RISOLUZIONE Vi sono 22n percorsi possibili; occorre calcolare il numero dei percorsi che terminano nelpunto di ascissa 2k conk = −n, . . . ,−1, 0, 1, . . . , n (i punti di ascissa dispari non sono raggiungibili percheil numero dei passie pari, e si parte dall’ascissa 0). Si termina in (2k, 0) se e solo se si sono fattin +k passi

verso destra en − k passi verso sinistra, il chee possibile in(

2n

n + k

)modi. Dunque

pn(2k) =1

22n

(2n

n + k

), k = −n, . . . ,−1, 0, 1, . . . , n .

La diseguaglianza cercata si puo dimostrare per induzione sun. Essae certamente soddisfatta pern = 1,perche p1(2) = 1/4 ≥ 1/(2 · 3). Supponendola vera pern − 1, cioe

pn−1(2) =1

22n−2

(2n − 2

n

)≥ 1

21

2n − 1

proviamola pern:

pn(2) =1

22n

(2n

n + 1

)=

14

(2n)(2n − 1)(n + 1)(n − 1)

pn−1(2) ≥

14

(2n)(2n − 1)(n + 1)(n − 1)

12

12n − 1

=

14

(2n)(2n + 1)(n + 1)(n − 1)

12

12n + 1

.

Siccome si vede facilmente che per ognin

2n (2n + 1) ≥ 4 (n + 1) (n − 1)

la diseguaglianza cercatae dimostrata.

112 I problemi della Scuola Normale

1993.3* Dati tre numeri interip > 2, q > 2, r > 2 si consideri un parallelepipedo di legno taleche i tre spigoli uscenti da un vertice abbiano lunghezzap, q, r. Dopo aver dipinto la superficie esternadel parallelepipedo, questo viene tagliato, mediante sezioni parallele alle facce, in cubetti aventi spigolidi lunghezza 1. Ovviamente alcuni dei cubetti sono parzialmente colorati, mentre altri non sono coloratiaffatto.Si dimostri che esiste solo un numero finito di terne(p, q, r) per ciascuna delle quali il numero dei cubettiparzialmente coloratie uguale al numero di quelli che non sono colorati affatto.

RISOLUZIONE I cubetti non colorati formano un parallelepipedo con spigoli di misurap− 2, q − 2 er − 2.Pertanto se il numero dei cubetti coloratie uguale a quello dei cubetti non colorati si avra

p q r = 2 (p − 2) (q − 2) (r − 2) , [1]

che possiamo riscrivere come

12

=p − 2

p

q − 2q

r − 2r

. [2]

Supponiamo che siap ≤ q ≤ r; si avra allora(

p − 2p

)3

≤ 12

<p − 2

p,

da cui

12

<p − 2

p≤ 3

√12

e pertanto 4< p < 10; dunque vie solo un numero finito di possibilita per il piu piccolo interop. Conpfissato l’equazione [2] diventa

p

2 (p − 2)=

q − 2q

r − 2r

,

e con la stessa argomentazione si trova che(

q − 2q

)2

≤ p

2 (p − 2)<

q − 2q

,

da cui

p

2 (p − 2)<

q − 2q

≤√

p

2 (p − 2)< 1 .

Per ognip ammissibile vie quindi un numero finito diq possibili e, fissatip e q, vi e (al piu) un valorepossibile perr, il che conclude la dimostrazione.

Anno 1993 113

1993.4 Sia data una circonferenzaΓ. Un arco circolareγ congiunge due punti distinti diΓ ede internoal cerchioC racchiuso daΓ. Dimostrare che, se le due regioni in cuiγ divide C hanno aree uguali, lalunghezza diγ supera il diametro diΓ.

RISOLUZIONE Indichiamo conA eB gli estremi dell’arcoγ e conM il suo punto medio. Il triangoloAMBdeve contenere al suo interno il centroO di Γ; se cosı non fosse il diametro parallelo adAB dividerebbeil cerchio in due parti di egual area, una delle qualie interamente contenuta in una delle regioni in cuiγdivide il cerchio; tali regioni non potrebbero percio avere area uguale.

O O

M

M

A B

A B

Figura 1

Cio osservato bastera notare che un arco di circonferenza ha lunghezza maggiore della corda che essosottende e dunque

_AB =

_AM +

_MB > AM + MB .

A BH

M

O

Figura 2

D’altra parteAM > AO perche i triangoli rettangoliAMH e AOH hanno uno stesso cateto e si haOH < MH. AnalogamenteBM > BO e dunque

_AB > AO + BO = diametro.

114 I problemi della Scuola Normale

1993.5 Sian un intero positivo pari. Mostrare che si possono trovare (in modo non necessariamenteunico)n numeri realia1 > a2 > . . . > an > 0 tali che

a1 + a2 + . . . + an = 2n

(a1 − a2) + (a3 − a4) + . . . + (an−1 − an) = n .

Mostrare poi che questo none possibile (pern > 2) se si impone la condizione ulteriore

a1 − an ≤ n .

RISOLUZIONE Cominciamo osservando che, sen = 2, vi e una sola soluzione data daa1 = 3, a2 = 1.Nel caso che sian > 2 mostriamo che se esiste una soluzione allora possiamo costruirne infinite altre.Consideriamo ad esempio i primi quattro numeria1 > a2 > a3 > a4 e notiamo chee possibile trovare unnumeroh abbastanza piccolo in modo che, postoa′

1 = a1+h ea′3 = a3−h, si abbia ancoraa′

1 > a2 > a′3 > a4;

i numeria′1, a2, a

′3, a4 . . . , an verificano per costruzione anche le due uguaglianze.

Per dimostrare l’esistenza cerchiamo di costruire glin numeri della sequenza, cercando nel repertorio delleprogressioni piu familiari.Proviamo a vedere per prima cosa se le due condizioni possonoessere verificate prendendoa1, a2, . . . , an

in progressione aritmetica,ak = a1 − (k − 1)q, conq > 0. Sostituendo nella seconda relazione si ha:

n = (a1 − a2) + (a3 − a4) + . . . + (an−1 − an) = q · n/2 ,

e quindi deve essereq = 2; sostituendo nella prima relazione otteniamo

2n =n∑

1

[a1 − 2 (k − 1)] = n a1 − 2n∑

1

(k − 1) = n a1 − n (n − 1) ;

deve esserea1 = n + 1, e i numeriak = (n + 1)− 2 (k − 1), con 1≤ k ≤ n verificano le condizioni richieste,tranne quella di essere tutti positivi. Tentativo fallito.Proviamo conak in progressione geometrica,ak = a1 · qk−1, conq compreso fra 0 e 1; sea1 > 0, leak sonopositive e decrescenti.Notiamo poi che anche i numeria1 − a2, a3 − a4, a5 − a6, . . . , an−1 − an sono i primin/2 termini diuna progressione geometrica di ragioneq2 e primo termine (1− q) a1. Imponiamo le due condizioni perdeterminarea1 e q.

a1 + a2 + . . . + an = = a1 (1 + q + . . . + qn−1) =

a11− qn

1− q= 2n

(a1 − a2) + (a3 − a4) + . . . + (an−1 − an) =

a1 (1− q) (1 + q2 + . . . + (q2)(n/2)−1) =

a1 (1− q)1− qn

1− q2=

a11− qn

1 + q= n ,

da cui1− qn

1 + q=

n

a1=

12

1− qn

1− q.

Deve essere quindi (1 +q)/2 = 1− q, da cuiq = 1/3, e sostituendo, ad esempio nella prima equazione,otteniamo

a1 = 2n1− q

1− qn=

43

n

1− (1/3)n.

Anno 1993 115

Per il punto (b) mostriamo che la condizionea1 − an ≤ n e incompatibile con le altre due. Infatti per ladecrescenza delleah si ha:

(a1 − a2) + (a3 − a4) + . . . + (an−1 − an) =

(a1 − an) + (a3 − a2) + (a5 − a4) + . . . + (an−1 − an−2) <

a1 − an ≤ n

contro l’ipotesi

(a1 − a2) + (a3 − a4) + . . . + (an−1 − an) = n .

116 I problemi della Scuola Normale

1993.6 Per ogni intero positivon si indichi conθ(n) il numero reale, compreso fra 0 e2π, tale che

θ(n) = 3n (mod2π)

cioe tale cheθ(n) − 3n sia un multiplo intero di2π.i) Mostrare che0 ≤ θ(n) ≤ π/2 per infiniti valori di n (si puo usare il fatto che3.14 < π < 3.15).ii) Mostrare cheθ(n) 6= θ(m) sen 6= m.

RISOLUZIONE Da 3.14 < π < 3.15 otteniamo che 50· 2π < 315 e che 315< 50· 2π + 1 < 50· 2π + π/2 edunque che

0 < 3 · 105− 50 · 2π <π

2;

per definizione diθ si ha quindi che

0 < θ(105)<π

2.

Mostriamo ora che, dato un interon0 per il quale 0< θ(n0) < π/2, se ne puo trovare uno piu grande chegode della stessa proprieta.Infatti, si scelga un interok grande a sufficienza in modo che sia

k< θ(n0) <

π

2− 2π

k[1]

e si suddivida l’intervallo [0, 2π] in k intervalli di ampiezza 2π/k (si noti chek e necessariamente maggioredi 8). I k + 1 numeri

θ(n0), θ(n0 + 1), θ(n0 + 2), . . . , θ(n0 + k)

appartengono tutti a [0, 2π] e per il principio dei cassetti due di essi devono appartenere a unostesso intervallo. Cio significa che esistonom, n distinti tali che

|θ(m) − θ(n)| <2π

k[2]

e per ragioni di simmetria si puo supporrem > n.Sep, q, r sono gli interi per cui

θ(n0) + 2π p = 3n0

θ(m) + 2π q = 3m

θ(n) + 2π r = 3n

se ne deduce che

θ(n0) + θ(m) − θ(n) = 3 (n0 + m − n) − 2π (p + q − r) .

A causa delle diseguaglianze [1] e [2] il primo membro di questa uguaglianzae compreso fra 0 eπ/2, edunque essoe uguale aθ(n0 + m− n). Si e cosı trovato un interon1 = n0 + m− n strettamente maggiore din0 tale che 0< θ(n1) < π/2; proseguendo in questo modo se ne trovano infiniti.

La seconda parte segue dalla irrazionalita di π: se fosseθ(n) = θ(m) conn 6= m si avrebbe

3n − 2π j = 3m − 2π k

conj, k interi distinti, da cui

π =32

m − n

k − j

il che e assurdo.

Anno 1994 117

1994.1 SiaX un insieme din elementi, doven e un numero pari e siak un intero positivo. Diciamo cheuna funzionef daX in X ha molteplicita k se per ognia ∈ X l’insiemex | f (x) = f (a) hak elementi.Sono di piu le funzioni di molteplicita 1 o quelle di molteplicita 2?

RISOLUZIONE SiaX un insieme din = 2m elementi. Le funzionif daX in X di molteplicita 1 sono lefunzioni biunivoche diX in se, e sono quindi tante quante le permutazioni din oggetti, cioe n!.Per contare le funzionif daX in X di molteplicita 2, possiamo pensare di raggrupparle in base all’insiemeimmagine: f1 e f2 stanno nello stesso sottoinsieme sef1(X) e f2(X) sono lo stesso sottoinsieme conmelementi diX; quindi il numero delle funzioni di molteplicita 2 e il prodotto del numero dei sottoinsiemiconm elementi di un insieme con 2m elementi (cheeC(2m, m), numero delle combinazioni di 2m oggettiadm adm), per il numero di funzioni di molteplicita 2 da un insiemeX di 2m oggetti in un insieme dimoggetti,Y = y1, . . . , ym .Contiamo queste funzioni. Per ognii = 0, 1, . . . , m−1 dobbiamo scegliere i due punti diX che hanno comeimmagine il puntoyi; questoe possibile farlo in (2m−2 i) (2m−2 i−1)/2 modi distinti, e di conseguenzail numero delle funzioni di molteplicita 2 daX in Y e

2m (2m − 1)2

(2m − 2)(2m − 3)2

. . .(2 · 1)

2=

(2m)!2m

.

Vi sono dunque

C(2m, m) · (2m)!2m

=((2m)!)2

2m(m!)2

funzioni di molteplicita 2 daX in X; il rapporto fra tale numero e il numero di quelle di molteplicita 1e

((2m)!)2

2m(m!)2

1(2m)!

=1

2m

2m(2m − 1) . . . (m + 1)m (m − 1) . . .1

=

2m(2m − 1) . . . (m + 1)2m (2m − 2) . . .2

> 1 ;

Vi sono quindi (molte) piu funzioni di molteplicita 2 che funzioni di molteplicita 1.

118 I problemi della Scuola Normale

1994.2* Vi sono 4 citta collegate a due a due da 6 strade che non si intersecano (cioe ogni coppiadi citta e collegata da una sola strada). Tutte le strade sono aperte al traffico con la stessa probabilitap = 1/2.Determinare la probabilita che in un determinato istante partendo da una qualsiasi citta si possa arrivaread ogni altra citta.

RISOLUZIONE Per fissare le idee, tracciamo il grafo corrispondente ai collegamenti fra le quattro citta,numerando le strade.

1 3

2

4

5

6

Figura 1

Indichiamo con una sestupla di numeri lo stato delle strade:uno 0 in posizionei-esima indichera che lai-esima stradae chiusa al traffico, un 1 indichera invece che essae percorribile. Cosı (110011) indicherache sono aperte la prima, la seconda, la quinta e la sesta strada, chiuse la terza e la quarta.Vi sono 26 = 64 possibilita per lo stato delle strade, il numero delle sestuple distinte di 0 o 1, e, supponendoche lo stato di una strada sia indipendente da quello delle altre, ciascuna di tali possibilita e equiprobabile.Occorre dunque calcolare quante sono le sestuple che consentono da ogni citta di raggiungere tutte le altre.Ovviamente, con la sestupla di tutti 0 il traffico risultera bloccato; neppure si riuscira a raggiungere tutte lecitta con una sola strada aperta (6 possibilita), ne con due strade aperte (se esse sono consecutive risulterannocollegate tre citta, ma sara isolata la quarta, e in caso contrario sara possibile viaggiare tra due coppie dicitta, ma non da una citta di una coppia ad una dell‘altra). Le sestuple con due soli 1 sono

(62

)= 15.

Se vi sono tre strade aperte, occorre distinguere due casi: se tali strade formano un circuito chiuso, vi sarauna citta che rimane non collegata, e questo puo avvenire in 4 modi diversi (le possibili scelte di 3 citta su4).

Figura 2

In tutti gli altri casi, che sono(6

3

)− 4 = 16, sara sempre possibile raggiungere da ogni citta le altre tre;

infatti, o le tre strade sono consecutive e quindi, essendo il percorso aperto, connettono tutte e quattro lecitta, o formano un percorso ramificato, nel qual caso escono tutte da una stessa citta e dunque connettono,tramite questa, le altre tre. In figura sono rappresentati i 16 casi possibili:

Anno 1994 119

Figura 3

Con 4 “1” (quattro strade aperte, e(6

4

)= 15 possibilita) si puo sempre andare da una qualsiasi citta a

qualunque altra, perche l’aggiunta di una strada aperta ai “casi peggiori” con tre strade (fig. 2) connettenecessariamente anche la quarta citta. A maggior ragione cio si potra fare con cinque strade aperte (6possibilita) o con tutte le strade aperte (1 possibilita).In conclusione i casi favorevoli sono 16 + 15 + 6 + 1 e quindi la probabilita cercatae 38/64 = 19/32.

1994.3 Mostrare che 41 non puo essere espresso come differenza di una potenza di 2 e di una potenzadi 3, cioe che non puo sussistere nessuna delle due uguaglianze seguenti:

41 = 2n − 3m , 41 = 3n − 2m

conn, m interi positivi.

RISOLUZIONE Iniziamo provando che none mai verificata 41 = 3n−2m ; se per assurdo vi fossero due interin, m per cui l’uguaglianza vale, prendendo i resti modulo 3 del primo e del secondo membro dovremmoavere che

41 mod 3 = 2 =−2m mod 3

e−2m mod 3 vale 2 perm pari e 1 perm dispari; quindim deve essere pari,m = 2k, e dunque esistonon,k tali che 41 = 3n − 22k. Passando ai resti modulo 4 abbiamo

41 mod 4 = 1 = 3n mod 4

e 3n mod 4 vale 1 pern pari e 3 pern dispari; quindin deve essere pari,n = 2h, ed esistonoh, k taliche 41 = 32h − 22k = (3h + 2k) (3h − 2k) ; dato che 41e primo, dovrebbe aversi che 3h − 2k = 1, e quindi41 = (2k + 1) + 2k = 2k+1 + 1 e questoe impossibile perche 40 none una potenza di 2.

Proviamo ora che none mai verificata 41 = 2n − 3m ; se per assurdo vi fossero due interin, m per cuil’uguaglianza vale, prendendo i resti modulo 3 a primo e a secondo membro dovremmo avere che

41 mod 3 = 2 = 2n mod 3 ;

l’espressione 2n mod 3 vale 1 pern pari e 2 pern dispari e quindin deve essere dispari,n = 2h+1; esistonopertantoh, m tali che 41 = 22h+1 − 3m. Passando ai resti modulo 4 abbiamo

41 mod 4 = 1 =−3m mod 4,

ma−3m mod 4 vale 3 perm pari e 1 perm dispari, e quindim deve essere dispari,m = 2k + 1, ed esistonoh, k tali che 41 = 22h+1 − 32k+1 = 2 · 4h − 3 · 9k; questo pero e impossibile, perche passando ai resti modulo8 avremmo che 41 mod 8 = 1, mentre

120 I problemi della Scuola Normale

(2 · 4h − 3 · 9k) mod 8 =−3 (8 + 1)k mod 8 = 5.

1994.4 SiaP un punto interno ad un triangolo equilatero. Per ogni retta passante perP sianoX eY idue punti di intersezione tra la retta e i lati del triangolo.Determinare, per ogni puntoP , la retta o le retteche rendono minimo il prodotto

PX · PY [1]

RISOLUZIONE Cominciamo col risolvere un sottoproblema: dato un angoloMON con un’ampiezza di 60

e un puntoP in tale angolo, determinare le rette perP che rendono minimo il prodotto [1].

O K Y

P

XH

M

ϑ

Figura 1

Indichiamo conH eK i piedi delle altezze condotte daP sui lati dell’angolo. Se indichiamo conϑ e conτ

gli angoli PXH eP Y K abbiamo

PX · PY =PH

sinϑ· PK

sinτ

e quindi l’espressione [1] sara minima quandoϑ e τ rendono massimo il prodotto

sinϑ · sinτ =12

(cos(ϑ − τ) − cos(ϑ + τ)) =

12

(cos(ϑ − τ) − cos(120)) ,

cioe quandoϑ = τ = 60; il triangoloOXY deve quindi essere equilatero e il valore minimo dell’espressione[1] e dato daPH · PK / sin2 60 = (4/3)PH · PK.

Torniamo ora al problema di partenza, consistente nel determinare, fissatoP nel triangolo equilateroABC,quali rette perP forniscano il valore minimo del prodotto [1].

Anno 1994 121

A C

B

P

KB

YA

KC

XA

A C

B

P

KB

KC

KA

Figure 2, 3

Per il risultato precedente applicato all’angoloA si ha che il valore minimoe vA = (4/3)PKC · PKB esi ottiene con la rettaXAYA per P parallela aBC; lavorando sull’angoloB si ha che il valore minimoevB = (4/3)PKC · PKA, ottenuto con la rettaXBYB parallela adAC; relativamente alle intersezioni con ilati dell’angoloC si ha che il valore minimoevC = (4/3)PKA · PKB, ottenuto con la rettaXCYC parallelaadAB e il valore minimo ottenibilee dunque il piu piccolo fravA, vB evC .Ricordando che la bisettriceAHA e il luogo dei puntiP per cui si haPKB = PKC, e analogamente per lealtre due bisettrici, possiamo cosı concludere:• seP e interno al quadrilateroAHCOHB , intersezione dei triangoliABHB eACHC , la soluzionee data

davA, ede realizzata da una unica retta,XAYA;• seP e interno al quadrilateroBHAOHC il minimo evB , realizzato daXBYB;• seP e interno al quadrilateroCHBOHA il minimo e vC , realizzato daXCYC .E chiaro che seP = O i valori vA, vB e vC coincidono, e sono ottenuti dalle retteXAYA, XBYB e XCYC ;se inveceP e interno al segmentoOHA, i valori vB e vC coincidono, e sono ottenuti dalle due retteXBYB

eXCYC , e analogamente seP e interno aOHB oppure aOHC .

1994.5* Consideriamo un triangolo e dividiamo i suoi lati inn parti uguali medianten − 1 puntisu ciascun lato. Congiungiamo ogni vertice con i punti cosı ottenuti sul lato opposto. Si dimostri che sen e primo maggiore di 2 allora non esistono punti appartenentisimultaneamente a tre dei segmenti cosıcostruiti.

RISOLUZIONE Supponiamo che il puntoP appartenga a tre delle rette considerate,AR, BS, CT e che sia

SA =h

nAC , TB =

k

nAB , RC =

j

nBC .

Tracciamo, come in Figura 1, la parallela aAC passante perB, e sianoM eN le intersezioni con tale rettadelle rette perA eR e perC eT rispettivamente.

N B M

A S C

TR

Figura 1

122 I problemi della Scuola Normale

Per similitudine si avraSA

SC=

BM

BN,

RC

RB=

CA

BM,

TB

TA=

BN

CA.

Moltiplicando fra loro tali relazioni si ha

SA

SC

RC

RB

TB

TA= 1

e quindi

h

n − h

k

n − k

j

n − j= 1,

ovvero

n3 − (h + k + j) n2 + (h k + h j + k j) n = 2h k j .

Ne segue che 2h k j, con 0< h, k, j < n, e divisibile pern, ma questo none possibile sen e primo diversoda 2.

Anno 1994 123

SECONDA SOLUZIONE

Ad ogni puntoP di un triangoloP1P2P3 corrisponde in modo univoco la terna di numeri non negativi(a1, a2, a3) cona1 + a2 + a3 = 1, tali che

P = a1P1 + a2P2 + a3P3

dove, seα e un numero reale eA, B sono i due puntiA = (x1, y1) e B = (x2, y2), la scritturaα A indicail punto di coordinate cartesiane (αx1, α y1) e conA + B si intende il punto (x1 + x2, y1 + y2). La terna(a1, a2, a3) fornisce le coordinate baricentriche del puntoP .

P1 P3

P1

P

Figura 2

Determiniamo le coordinate baricentriche del punto ottenuto proiettandoP daP1 sul lato oppostoP2P3: lasemiretta che esce daP1 e passa perP ha equazione parametrica

[(1 − s) + sa1]P1 + sa2P2 + sa3P3

ed intersecaP2P3 quando (1− s) + sa1 = 0, e dunque pers = 1/(1− a1); la proiezione ha quindi coordinatebaricentriche (0, a2/(1− a1), a3/(1− a1)).Analogamente, la proiezione daP2 sul latoP1P3 ha coordinate baricentriche (a1/(1− a2), 0, a3/(1− a2)) ela proiezione daP3 sul latoP1P2 ha coordinate (a1/(1− a3), a2/(1− a3), 0).Dobbiamo dimostrare che sen e un numero primo maggiore di due, per nessun punto (a1, a2, a3) le proiezionisui lati hanno coordinate baricentriche con componenti della formai/n, coni intero, 0< i < n.Supponiamo chen, h1, h2, h3 siano numeri interi tali che

a1 =h3

n(1− a3) , a2 =

h1

n(1− a1) , a3 =

h2

n(1− a2) .

Siccomea1 + a2 + a3 = 1 possiamo riscrivere le uguaglianze precedenti come

a1 =h3

n(a1 + a2) , a2 =

h1

n(a2 + a3) , a3 =

h2

n(a1 + a3) .

Quindi a1, a2, a3 verificano la condizionea1 + a2 + a3 = 1 e risolvono il sistema

(n − h3) a1 − h3 a2 = 0

(n − h1) a2 − h1 a3 = 0

−h2 a1 + (n − h2) a3 = 0 .

Perche tale sistema ammetta soluzioni diverse daa1 = a2 = a3 = 0, il determinante della matrice associatadeve annullarsi, pertanto

det

[n − h3 −h3 0

0 n − h1 −h1

−h2 0 n − h2

]=

(n − h1) (n− h2) (n − h3) − h1 h2 h3 = 0

124 I problemi della Scuola Normale

Deve quindi essere

n3 − (h1 + h2 + h3) n2 + (h1h2 + h1h3 + h2h3)n = 2h1h2h3 .

Sen e primo e diverso da 2, per qualunque scelta dih1, h2, h3 fra 1 en − 1, il secondo membro nonedivisibile pern, mentre il primo loe, e dunque la tesie dimostrata.

1994.6 Sianoa1, a2, . . . , an numeri reali e sianob1, b2, . . . , bn definiti da

bi = max1≤j≤n

(i · j − aj) per ogni i = 1, 2, . . . , n.

Allo stesso modo si costruisconoc1, c2, . . . , cn a partire dab1, b2, . . . , bn e poid1, d2, . . . , dn a partire dac1, c2, . . . , cn; si dimostri che

ci ≤ ai per ognii = 1, 2, . . . , n,di = bi per ognii = 1, 2, . . . , n.

RISOLUZIONE Scriviamo esplicitamente la dipendenza deic1, c2, . . . , cn dai numeria1, a2, . . . , an .

ch = max1≤i≤n

(h · i − bi) =

max1≤i≤n

(h · i − max1≤j≤n

(i · j − aj)) =

max1≤i≤n

min1≤j≤n

(i · (h − j) + aj) .

Si vede allora che, qualunque siai,

min1≤j≤n

(i · (h − j) + aj)

e il minimo di un insieme di numeri fra i quali compare ancheah e dunque

ch = max1≤i≤n

min1≤j≤n

(i · (h − j) + aj) ≤ max1≤i≤n

ah = ah ,

il che prova la prima parte della tesi.

Per la seconda parte, notiamo che daai ≥ ci segue che, per ognii e j risultai · j − aj ≤ i · j − cj ; ma allorail massimo dei numeri a primo membro al variare dij e minore o uguale del massimo al variare dij deinumeri a secondo membro, e quindi:

bi = max1≤j≤n

(i · j − aj) ≤ max1≤j≤n

(i · j − cj) = di

e dunque, per ognii, risulta

bi ≤ di ;

ma idi sono costruiti a partire daici come ibi sono costruiti partendo dagliai e dunque per quanto dimostratonella prima parte si ha anche che

di ≤ bi ,

e quindi deve essere

di = bi

per ognii = 1, 2, . . . , n.

Anno 1995 125

1995.1 SianoC1, C2 due circonferenze di centri e raggi rispettiviO1, O2 e r1, r2. Dato un puntoPesterno alle due circonferenze si considerino le tangenti per P alle due circonferenze e sianoM1, N1 eM2,N2 i rispettivi punti di contatto.Si determini il luogo dei punti tali chePM1

2 + PM22 = 1.

Si determinino i puntiP per cuiPM12 + PM2

2 e minima.

RISOLUZIONE

M1 P

M2

C1

C2

N1

N2

r1

r2

O1 Q O2

Figura 1

Osserviamo che 1 =PM12 + PM2

2 = PO12 − r2

1 + PO22 − r2

2; il luogo e quindi descritto anche dallacondizionePO1

2 + PO22 = 1 +r2

1 + r22.

SiaQ il punto medio del segmentoO1O2; dal teorema della mediana applicato al triangoloO1PO2

si ha

2PQ2 = PO12 + PO2

2 − 12

O1O22 [1]

e quindi dovra essere

PQ2 =1 +r2

1 + r22

2− 1

4O1O2

2 .

SeO1O2 ≤√

2 (1 +r21 + r2

2) il luogo cercato giace sulla circonferenza di centroQ e raggior dato da√

1 + r21 + r2

2

2− 1

4O1O2

2 ,

e sara rappresentato dai punti di tale circonferenza che sono esterni sia aC1 sia aC2 (eventualmente,dall’intera circonferenza).

r1

O1 Q O2

P

r2 O1 O2

Q

P

Figure 2, 3

126 I problemi della Scuola Normale

La quantita PM12 + PM2

2 e minima se e solo see minimaPO12 + PO2

2 e dalla [1] segue subito che cio siha perP ≡ Q.

1995.2* Dati quattro punti distinti nel piano dimostrare chee sempre possibile sceglierne tre chedeterminino un angolo inferiore o uguale a45. In generale datin punti dimostrare che se ne possonoscegliere 3 che determinino un angolo inferiore o uguale a180/n.

RISOLUZIONE Formuleremo l’ipotesi che nessuna terna di punti sia allineata; in tal caso infatti si ha unasoluzione banale.E sempre possibile, datin punti nel piano (n ≥ 3), trovare un angolo convesso che ha vertice in uno di essi,lati che passano per altri due punti, e che contiene i restanti n−3 punti: basta infatti prendere l’involucroconvesso degli n punti, scegliere un verticeP del poligono che delimita l’involucro e le due semiretteche escono daP e passano per i due vertici adiacentiA eB.

P

B

A

P

B

A

C

Figure 1, 2

Consideriamo dapprima il cason = 4 (fig. 2). Se l’angolo inP e ottuso allora nel triangoloAPB il pi upiccolo fra gli angoli inA o in B deve essere minore di 45, perche la somma degli angoli inA, in B e inP vale 180. Se viceversa tale angoloe retto o acuto, allora la retta congiungenteP con il quarto verticeCdivide l’angolo in due parti, una almeno delle quali deve essere minore o uguale a 45.Nel caso generale si procede in modo simile. Scelto un angoloAPB che contiene i restanti punti all’internosi hanno due casi: o tale angolo supera (1−2/n)180, e allora nel triangoloAPB uno degli angoli inA o inB deve essere minore di 180/n, oppure, in caso contrario, le rette che congiungonoP con i restantin − 3punti dividono l’angolo inP in n− 2 angoli che, se fossero tutti maggiori di 180/n darebbero per l’angoloin P una ampiezza piu grande di (1− 2/n)180, contro l’ipotesi.Si noti che il risultato none migliorabile: nel caso del poligono regolare din lati gli angoli formati da duediagonali consecutive uscenti dallo stesso vertice valgono esattamente 180/n.

1995.3 Dimostrare che sea, b, c sono tre interi consecutivi alloraa3 + b3 + c3 e multiplo di 9.

RISOLUZIONE Supponiamo che siaa < b < c; uno dei tre numerie multiplo di 3, e gli altri due sonorispettivamente della forma 3h + 1 e 3k + 2. Il cubo del multiplo di 3e certamente divisibile per 9; bastaprovare che (3h + 1)3 + (3k + 2)3 e anch’esso multiplo di 9. Infatti

(3h + 1)3 + (3k + 2)3 =

27h3 + 27h2 + 9h + 1 + 27k3 + 54k2 + 36k + 8 =

9 (3h3 + 3h2 + h + 3k3 + 6k2 + 4k + 1) .

Anno 1995 127

1995.4 Siaf : R → R una funzione non negativa, con concavita rivolta verso il basso, derivabile e talechef ′(0) > 0 ef (x) = f (2− x) per ognix ∈ R.Dimostrare che

∫ 2

0f (x) dx ≤ 2f (1)− [f (1)− f (0)]2

f ′(0)

Provare che se la condizionef (x) = f (2− x) none verificata, tale diseguaglianza puo non valere.

RISOLUZIONE L’ipotesi f (x) = f (2−x) implica che la funzione abbia derivata nulla nel puntox = 1: infattiseh e un incremento positivo si haf (1 + h) = f (2− (1− h)) = f (1− h) e dunque i rapporti incrementalidestro e sinistro nel punto 1

f (1 +h) − f (1)h

,f (1− h) − f (1)

−h

hanno segno opposto.Poiche la funzionee derivabile perx = 1, e dunque derivata destra e sinistra coincidono, tale derivata devenecessariamente essere 0.Il grafico di ogni funzione concava e derivabile giace al di sotto di ogni sua retta tangente. Quindi il graficodi f (x) nell’intervallo [0, 1] e contenuto nel trapezio delimitato dalla retta tangente in(0, f (0)), dalla rettay = f (1), tangente in (1, f (1)), e dalle rettey = f (0) ex = 1.

1 2

f(0)

f(1)

x

y

Figura 1

La retta tangente inx = 0 ha equazioney = f (0) + f ′(0)x e incontra la rettay = f (1) nel punto di ascissa(f (1)− f (0))/f ′(0). L’area di detto trapezioe quindi

12

[1 +

(1− f (1)− f (0)

f ′(0)

)](f (1)− f (0)) .

L’integrale dif (x) su [0, 1] sara dunque minore o uguale a tale area sommata all’area del rettangolo di base1 e altezzaf (0), cioe

∫ 1

0f (x) dx ≤ f (1)− 1

2[f (1)− f (0)]2

f ′(0).

La tesi segue poi osservando che∫ 2

1f (x) dx =

∫ 2

1f (2− x) dx =

∫ 1

0f (x) dx .

Se la condizionef (x) = f (2 − x) non e soddisfatta la diseguaglianzae in generale falsa, come proval’esempiof (x) = x (3− x) per cui si ha:

∫ 2

0f (x) dx =

103

> 2f (1)− [f (1)− f (0)]2

f ′(0)= 2 · 2− 4

3=

83

.

128 I problemi della Scuola Normale

1995.5 Dato A =(x, y) ∈ R

2 : |x|2a + |y|2a ≤ 1

ovea e un numero reale positivo, si consideri alvariare di k ≥ 0 la classeC degli insiemi

Bk =(x, y) ∈ R

2 : |x| + |y| ≤ k

contenuti inA. Determinarek in funzione dia in modo cheBk abbia area massima tra gli insiemi diC .

RISOLUZIONE Gli insiemiBk sono una famiglia di quadrati di centro (0, 0) e vertici sugli assi, e sek1 > k2

l’insieme corrispondenteBk1contieneBk2

; quindi, per ogni fissatoa, dobbiamo determinare il piu grandek per cuiBk sia contenuto inA.

a=2 a=1 a=1/4

Figura 1

Pera = 1/2 gli insiemiA eB1 coincidono e quindi il massimo valore possibile perk e 1.In ogni caso, seA contieneBk deve contenere anche i suoi vertici (±k, 0), (0,±k), da cui segue chek2a ≤ 1.Cio significa chek non puo superare 1, qualunque sia il valore dia.Sea > 1/2 l’insiemeA contieneB1: infatti se (x, y) appartiene aB1 si ha|x|+|y| ≤ 1, e dalla disuguaglianza|x|2a + |y|2a ≤ |x| + |y| ≤ 1 segue che (x, y) appartiene anche adA; quindi k = 1 e un valore ammissibile ede anche il massimo.Sea ≤ 1/2 le diseguaglianze si invertono e per trovarekmax dobbiamo determinare il piu piccolo valoreassunto dax + y quandox2a + y2a = 1 ex, y > 0. Parametrizziamo questo insieme ponendoxa = cosθ eya = sinθ con 0< θ < π/2, e cerchiamo il minimo di

x + y = (cosθ)1/a + (sinθ)1/a = f (θ)

con 0< θ < π/2. Determiniamo i valori per cuif ′(θ) si annulla

f ′(θ) = (cosθ)1/a−1 · (− sinθ) + (sinθ)1/a−1 · cosθ = 0

da cui

sinθ

cosθ=

(sinθ

cosθ

)1/a−1

;

per 0< θ < π/2 l’unica soluzioneeθ = π/4, chee un punto di minimo perchef (0) = f (π/2) = 1 ef ′(θ) < 0perθ vicino a 0. Quindi

kmax = minf (0), f (π/4) = min1, 2 (1/√

2)1/a = 21−1/(2a) .

Anno 1996 129

1995.6 Sianoa, b numeri reali non negativi tali cheb2 + b6 ≤ a2 − a6. Dimostrare che allora risulta:1) a ≤ 1;2) b < 2/3.

RISOLUZIONE

1) Se fossea > 1 si avrebbea2 − a6 = a2 (1 − a4) < 0 e quindi nessun numerob puo verificare ladiseguaglianza proposta.

2) Si haa2 − a6 = a2 (1− a2) (1 +a2) ≤ (1/4) · 2. Infatti il prodotto dei due numeria2 e 1− a2, che hannosomma indipendente daa, e massimo quando i due numeri sono uguali, cioe pera2 = 1/2, mentre ilfattore 1 +a2 non supera 2, per la prima parte.Se fosseb ≥ 2/3 si dovrebbe avere

(23

)2

+(

23

)6

≤ b2 + b6 ≤ a2 − a6 ≤ 12

,

il che e assurdo, come si verifica facilmente.

1996.1* Dato un quadratoABCD di lato unitario, determinare la massima costanteα e la minimacostanteβ per cui si ha

α ≤ PA + PB + PC + PD ≤ β

per ogni puntoP contenuto nel quadrato.

RISOLUZIONE Per il minimo osserviamo cheα ≥ min(PA + PC) + min(PB + PD) = AC + BD per ladiseguaglianza triangolare; d’altra parteAC + BD e un valore assunto daPA + PB + PC + PD,seP e l’intersezione delle diagonali del quadrato.

BA

CD

P

Figura 1

Quindi α = AC + BD = 2√

2.

Per studiare il valore massimo, consideriamo un sistema di assi cartesiani di origine inA, con l’assexlungo il lato AB e l’assey lungo il lato AD; i punti B e D avranno coordinate rispettivamente (1, 0) e(0, 1). Cerchiamo il massimo dell’espressionePA + PB per un puntoP a distanzah dal latoAB. Dato chepunti simmetrici rispetto alla rettax = 1/2 forniscono lo stesso valore per l’espressionePA +PB, poniamoP = (1/2 +x, h), con−1/2 ≤ x ≤ 1/2.

130 I problemi della Scuola Normale

A 1/2 B

R S

CD

P

Figura 2

Studiamo quindi

f (x) = PA + PB =

√(12

+ x

)2

+ h2 +

√(12− x

)2

+ h2

La disuguaglianza

f ′(x) =12 + x√(

12 + x

)2+ h2

−12 − x√(

12 − x

)2+ h2

≥ 0

equivale a

h2

[(12

+ x

)2

−(

12− x

)2]

= 2h2 x ≥ 0 ;

quindi il punto medio diRS (fig. 2) e punto di minimo.Il massimo dif (x) = PA + PB ad h fissatoe raggiunto inS e in R; qualunque delle due scelte rendemassima anchePC + PD e dunque tutta la somma. SeP appartiene, ad esempio, al latoBC, si haPA + PB + PC + PD = PA + PD + BC e l’espressionePA + PD e minima nel punto medio diBC ede massima negli estremi, ad esempio inC, dove valeBC + DC +

√2 = 2 +

√2, per quanto visto prima.

Dunque

β = 2 +√

2 .

SECONDA SOLUZIONE

SiaP un punto del quadratoABCD; mantenendo costante la sommaPA + PB troviamo quali posizioni diP rendono massimaPC + PD.Il luogo dei puntiP per i qualiPA+PB e una assegnata costante (maggiore diAB) e una ellisseE di fuochiA eB; sianoR eS le intersezioni di tale ellisse con i latiAD eBC.

Anno 1996 131

A B

CD

P SR

E

Figura 3

I punti R e S sono esterni all’ellisse di fuochiC e D passante perP ede quindi chiaro che fra tutti i puntidi E il massimo diPC + PD si ha quandoP coincide conR o conS.Prendiamo in considerazione, per fissare le idee, il puntoS interno aBC e vediamo come rendere minimala quantita SA + SD (SB + SC e sempre 1 per ogni posizione diS).

A B

D C

S

Figura 4

Allo scopo consideriamo l’ellisse di fuochiA e D passante perS; i punti B e C sono esterni a tale ellissequalunque siaS sul segmentoBC, pertanto tali punti realizzano il massimo della somma

SA + SB + SC + SD

che vale (ad esempio perS ≡ C) BC + DC + AC = 2 +√

2. Dunque

β = 2 +√

2 .

132 I problemi della Scuola Normale

1996.2 Il prezzo di mercatoP di una certa merce dipende dalla quantita totaleQ venduta secondo laleggeP = a − b Q, dovea e b sono due assegnati valori positivi.Sul mercato operano solo due produttori, in concorrenza fraloro.A regime, cioe quando nessuno dei due ha interesse a cambiare la quantita di merce da lui venduta, i dueproduttori vendono rispettivamente le quantita X eY di merce. Supponendo che la produzione avvenga acosto zero, determinareX eY .

RISOLUZIONE Si intende che in questo “modello” il mercatoe in grado di assorbire qualunque quantita diprodotto.Osserviamo intanto che nessuno dei due produttori ha interesse ad immettere sul mercato una quantita dimerce maggiore o uguale aa/b, perche cio azzera il prezzo; quindiX < a/b, Y < a/b.Il ricavo del primo produttoree dato da

(a − b (X + Y )) X = (a− b Y ) X − b X2 .

SeY e fissato, con 0< Y < a/b, tale ricavo ha per grafico una parabola che passa per 0 quandoX = 0 equandoX = (a− b Y )/b, e ha massimo perX = (a− b Y )/(2b); per valori diX inferiori il primo produttoreha interesse ad aumentare la produzione, per valori superiori a diminuirla.La condizione simmetrica,Y = (a − bX)/(2a) e la condizione di equilibrio per il secondo produttore, edevono valere entrambe:

2b X + b Y = a

b X + 2b Y = a .

Sottraendo si hab (X − Y ) = 0, e quindiX = Y , da cui 3b X = a; i valori di X e Y all’equilibrio sonodunque

X = Y =a

3b.

Puo essere interessante confrontare i risultati dello stessomodello in condizione di monopolio.Il ricavo dell’unico produttore, fornito dall’espressione (a−b X) X, e massimo perX = a/(2b); tale volumedi produzionee inferiore a quello di equilibrio per la situazione con due produttori, dato da 2· a/(3b).Il massimo ricavo possibile per il monopolistae

(a − b · a

2b

)· a

2b=

14

a2

b

ede maggiore della somma ricavata dai due produttori in concorrenza, data da(a − b · 2

a

3b

)· 2 a

3b=

29

a2

b.

Anno 1996 133

1996.3* Sia P un poliedro e sianoF il numero delle facce,S il numero degli spigoli eV il numerodei vertici di P. Sapendo che per il poliedroP, per la nota formula di Eulero, vale la relazioneF − S + V = 2,(a) provare cheP ha qualche faccia con meno di 6 lati;(b) dettok il numero delle facce con meno di 6 lati, determinare il minimo valore possibile perk.

RISOLUZIONE Sapendo che in un poliedro ogni spigolo appartiene esattamente a 2 facce, se supponiamoche ogni faccia abbia almeno 6 lati, arriviamo alla stima

S ≥ F · 62

= 3F , da cui F ≤ 13

S .

Stimiamo ora il numero di vertici in base a quello degli spigoli: da ogni vertice escono almeno 3 spigoli,ma ogni verticee contato due volte, una volta come primo estremo, l’altra come secondo estremo di unostesso spigolo e di conseguenza

S ≥ 32

V , da cui V ≤ 23

S .

Sostituendo nella uguaglianza di EuleroF − S + V = 2 si ottiene

2 = F − S + V ≤ 13

S − S +23

S = 0,

il che e chiaramente un assurdo; con cio e dimostrato il punto (a).Supponiamo ora chek facce abbiano meno di 6 lati; dobbiamo determinare il valoreminimo dik. La primastima diF si modifica nel modo seguente; perF − k facce sappiamo che i lati sono almeno 6, mentre lerimanentik facce hanno ognuna almeno 3 lati. Si ha quindi

S ≥ (F − k) · 6 +k · 32

= 3F − 3k

2,

da cui

F ≤ 13

S +12

k ,

e, sostituendo come prima,

2 = F − S + V ≤ 13

S +12

k − S +23

S =12

k ,

il che porta ak ≥ 4. Chek possa assumere il valore 4 si vede considerando il tetraedro(che contiene 0facce con almeno 6 lati); d’altra partee evidente che seF − k fosse 1 o 2 o piu grande, il numero di faccecon meno di 6 lati sarebbe certamente maggiore di 4, e dunque 4e la soluzione del problema di minimo.

134 I problemi della Scuola Normale

1996.4 Siaf (t) una funzione iniettiva definita sui numeri reali positivi. Dati x > 0 ey > 0, chiamiamof -Media dix e y l’unico numeroz tale che

f (z) =f (x) + f (y)

2.

Mostrare che la media geometrica√

x y e quella armonica2x y/(x + y) sono dellef -Medie.Fra le funzioni convessef , individuare quelle per le quali laf -Media risulta minore o uguale della mediaaritmetica.

RISOLUZIONE Per il primo punto, notiamo che:M (x, y) =

√x y = Mf (x, y) prendendo comef (t) la funzione logt: infatti

z =√

x y

se e solo se

logz = log√

x y = (logx + logy)/2 ;

N (x, y) = 2x y/(x + y) = Mf (x, y) perf (t) = 1/t; infatti

z = 2x y/(x + y) = 2/(1/x + 1/y) ,

che equivale a 1/z = (1/x + 1/y)/2 .

Sef e convessa ede iniettiva,f e o strettamente crescente o strettamente decrescente.Sef e strettamente crescente si ha:Mf (x, y) ≤ (x +y)/2 se e solo sef (Mf (x, y)) ≤ f (x +y)/2, equivalentea 1/2f (x)+1/2f (y) ≤ f (x+y)/2; d’altra parte laf e convessa, e quindif ((1− t)x+ty) ≤ (1− t)f (x)+tf (y)per ognit ∈ [0, 1] e in particolaref (1/2x+1/2y) ≤ 1/2f (x)+1/2f (y); quindi il grafico dif sta al di sopradi quello della retta secante perx e y, e coincide con questa nel punto medio e quindie unafunzione

affine.Se invecef e strettamente decrescente si ha:Mf (x, y) ≤ (x + y)/2 se e solo sef (Mf (x, y)) ≥ f (x + y)/2,equivalente a 1/2f (x) + 1/2f (y) ≥ f (x + y)/2 che segue direttamente dalla convessita di f .

Concludendo, le funzioni convesse per cui risultaMf (x, y) ≤ (x + y)/2 per ognix, y > 0 sono o convessestrettamente decrescenti, oppure sono funzioni affini.

Anno 1996 135

1996.5 Dato un triangolo nel piano euclideo si indichi conI il centro della circonferenza in essoinscritta e conΓ la circonferenza passante perI e per due qualunque dei vertici del triangolo.Provare che il centro diΓ si trova sulla circonferenza circoscritta al triangolo.

RISOLUZIONE Sia, ad esempio,Γ la circonferenza passante perI, B eC. Allora necessariamente il centrodi Γ deve giacere sull’asse diBC; si dovra provare che il centro diΓ e il puntoH, intersezione dell’asse delsegmentoBC con la circonferenza circoscritta. Si noti che, essendo

_BH =

_HC, la bisettrice dell’angolo in

A passa perH (oltre che perI).

H

A

B C

I

O

Figura 1

Dimostriamo cheHB = HI.Il centro I della circonferenza inscrittae punto di intersezione delle bisettrici; se indichiamo conα, β, γrispettivamente gli angoli del triangolo inA, B, C si avra che per il teorema dell’angolo esterno

BIH =α

2+

β

2.

D’altra parteHBC = HAC perche insistono sullo stesso arco e pertanto

HBI = HAC + CBI =α

2+

β

2= BIH .

Risulta cosı provato che il triangoloIBH e isoscele e quindiHB = HI. Allo stesso modo si prova cheHC = HI e cio conclude la dimostrazione.

136 I problemi della Scuola Normale

1996.6 A partire da un cerchioC1 tracciare successivamente: un triangolo equilateroP1 inscritto inC1, il cerchioC2 inscritto inP1, un quadratoP2 inscritto inC2, il cerchioC3 inscritto in P2, un pentagonoregolareP3 inscritto inC3, e cosı via, ottenendo in tal modo una successione

C1 ⊃ P1 ⊃ C2 ⊃ P2 ⊃ . . . Cn ⊃ Pn ⊃ Cn+1 ⊃ Pn+1 . . .

di cerchi e poligoni regolari concentrici, dovePn han + 2 lati.Mostrare che l’intersezione di tutti i cerchiCn e un cerchio di raggio positivo. Il candidato puo ricorrerealla diseguaglianza, valida per ogni interok ≥ 1:

1(k + 1)2

+1

(k + 2)2+

1(k + 3)2

+ . . . <1k

.

RISOLUZIONE Indichiamo conRn il raggio del cerchioCn, e calcoliamo il rapportoRn+1/Rn; Rn+1 el’altezza di un triangolo isosceleAOB di latoRn e angoloAOB = 2π/(n + 2).

C1

P1

C2

O

A B

Rn+1

Rn

Figura 1

Si ha quindi che

Rn+1

Rn= cos

π

n + 2

e moltiplicando queste diseguaglianze pern da 1 am − 1 otteniamo:

Rm

R1= cos

π

3· cos

π

4· cos

π

5· . . . · cos

π

m + 1

Per provare la tesie necessario e sufficiente dimostrare che i rapportiRm/R1 non possono avvicinarsiindefinitamente a 0 al crescere dim, o, il chee lo stesso, che i logaritmi di tali rapporti debbono esserelimitati dal basso. Passando ai logaritmi a primo e secondo membro

logRm

R1= log cos

π

3+ log cos

π

4+ log cos

π

5+ . . . + log cos

π

m + 1

D’altra parte, per ognin ≥ 1, posto per comodita di notazioneαn = π/(n + 2), si ha che:

log cosαn = −12

log1

cos2 αn= −1

2log(1 + tg2 αn) .

Ricordando che set e maggiore di−1 vale la diseguaglianza log(1+t) ≤ t, che per ognix si ha| sin(x)| ≤ |x|e che per ognin > 1 il valore di cosαn e maggiore di cosα1 = cos(π/3) = 1/2, abbiamo:

log cosαn ≥ −12

tg2 αn = −12

sin2 αn

cos2 αnr ≥ −2

( π

n + 2

)2.

Anno 1996 137

Si ha quindi

logRm

R1≥ −2π2

[132

+142

+152

+ . . . +1

(m + 1)2

].

Usando il suggerimento conk = 2 otteniamo

logRm

R1≥ −2π2 · 1

2= −π2 .

Possiamo concludere osservando che, qualunque siam, si ha

Rm ≥ R1e−π2

;

quindi tutti iCn contengono uno stesso cerchio e dunque la loro intersezionee un cerchio centrato nell’origine

di raggio maggiore o uguale aR1e−π2

.

138 I problemi della Scuola Normale

1997.1 Un cuboe appoggiato su un piano. Un bambino lo muoven volte, facendolo rotolare (senzastrisciare) ogni volta su uno dei lati della faccia su cuie appoggiato. Si suppone che la prima mossa siacasuale e che, ad ogni mossa successiva, il bambino scelga casualmente di far rotolare il cubo su uno deidue lati contigui al lato scelto in precedenza (vedi Figura:dopo aver rotolato sul latoa, il cubo rotolerasul latob oppure sul latoc).

a

b

c

a) Dimostrare che se il cuboe tornato nella posizione iniziale (non necessariamente appoggiato sullastessa faccia) alloran e divisibile per4.

b) Calcolare la probabilita p(n) che il cubo sia tornato nella posizione iniziale.c) Dimostrare che

516k

− 116k2

≤ p(4k) ≤ 13k

− 112k2

, k ≥ 1 .

RISOLUZIONE

a) Ad ogni mossa il cubo si sposta di un segmento pari al suo lato in direzione Nord, Sud, Ovest o Est.Affinche dopon mosse il cubo sia di nuovo nella posizione iniziale, il numero h degli spostamenti versoNord deve uguagliare quello degli spostamenti verso Sud; analogamente, se vi sonok spostamenti versoEst, altrettanti devono esservi verso Ovest.D’altra parte, per ipotesi, il bambino alterna una mossa Est–Ovest con una mossa Nord–Sud e dunque si hachek = h per cui, in definitiva,n = 4h.

b) Per il punto precedente, sen non e multiplo di 4, nessuna sequenza din mosse puo lasciare il cubonella posizione iniziale, e dunque in tal casop(n) = 0.Sen = 4h, le sequenze di mosse possibili si ottengono scegliendo in ogni modo possibile le 2h mosseNord–Sud e le 2h mosse Est–Ovest; tenendo conto del fatto che la prima mossa puo essere arbitrariamentein direzione Nord–Sud o Est–Ovest, il che porta ad un ulteriore fattore 2, il numero totale di percorsipossibilie dato da 2· 22k · 22k.Le sequenze di mosse favorevoli sono quelle per cui su 2h mosse Nord–Sud esattamenteh sono mosseverso Nord, e su 2h mosse Est–Ovest ve ne sonoh verso Est. Ognuna delle due scelte, fra loro indipendenti,

puo essere effettuata in(2h

h

)modo distinti; vi sono quindi 2

(2hh

)2casi favorevoli, e si ha quindi

p(4k) =[

122k

(2k

k

)]2

.

c) Perk = 1 le diseguaglianze sono verificate poiche tutti i membri valgono 1/4. Useremo un ragiona-mento per induzione suk:

p(4 (k + 1)) = p(4k)[

2k + 12 (k + 1)

]2

.

Per la disuguaglianza a destra si ha per induzione:

p(4 (k + 1)) ≤ 13k

(1− 1

4k

) [2k + 1

2 (k + 1)

]2

.

Si verifica facilmente che

Anno 1997 139

13k

(1− 1

4k

) [2k + 1

2 (k + 1)

]2

≤ 13 (k + 1)

(1− 1

4 (k + 1)

).

Per la disuguaglianza a sinistra si ha per induzione:

p(4 (k + 1)) ≥ 516k

(1− 1

5k

) [2k + 1

2 (k + 1)

]2

.

Ma perk ≥ 1 e facile verificare che vale la disuguaglianza

516k

(1− 1

5k

) [2k + 1

2 (k + 1)

]2

≥ 516 (k + 1)

(1− 1

5 (k + 1)

)

il che conclude la dimostrazione.

1997.2 Si dice che un puntoP esterno ad una circonferenzaC “vede” la circonferenza sotto un angoloα se l’angolo (contenenteC) compreso fra le tangenti aC condotte daP e uguale aα.a) Data una circonferenzaC e un angoloA di ampiezzaα > 0, costruire il luogo dei punti del piano che

vedonoC sotto l’angoloα.b) Date due circonferenzeC e C′ esterne l’una all’altra, di centriO, O′ e raggi r, r′ rispettivamente,

costruire il luogoL dei punti del piano che vedono le due circonferenze sotto lo stesso angolo.c) Dire (in termini dei dati) in che intervallo varia l’angolo di visuale al variare diP in L e quali sono i

punti inL dove tale angoloe minimo e massimo.

RISOLUZIONE

a) Preso un diametroMN della circonferenzaC, costruiamo l’angoloMOM ′ di ampiezza pari a quelladi A e tracciamo le tangenti aC perM ′ e perN . DettaP l’intersezione di dette tangenti,e evidente che,per differenza di angoli uguali, l’angolo inP ha ampiezzaα; il luogo richiestoe la circonferenza di centroO passante perP .

OM N

M ′

P

C

α

Figura 1

b) SiaP un punto esterno alle circonferenzeC e C′. Tracciamo, come in Figura 2, le tangenti daP alledue circonferenze e i segmentiPO e PO′. Il punto P appartiene aL se e solo seOPT = O′P T ′, e in talcaso i triangoliOPT eO′PT ′ sono simili, e quindiOP : O′P = OT : O′T ′. Dunque il luogoL e dato daipunti P per cuiOP : O′P = r/r′.Ser = r′ il luogo L e l’asse del segmentoOO′; ser er′ sono diversi, il luogoe dato dallacirconferenza

di Apollonio relativa al triangoloOPO′.

140 I problemi della Scuola Normale

CC ′

O O ′

T

T ′

P

r

r ′

Figura 2

Tale luogo contiene chiaramente i puntiP1 e P2 da cui si possono mandare tangenti comuni alle duecirconferenze, e questo consente di costruireL tracciando la circonferenza avente centro inM , punto mediodi P1P2 e raggioMP1.

P1

CC ′

O P2 O ′

P

L

Figura 3

c) La distanzaPO′ e massima quandoP coincide conP1, e decresce fino ad assumere il suo valore minimoperP = P2; per quanto osservato nel primo punto, quindi, l’angolo di visuale assume il suo massimo perP = P2 e il minimo perP = P1. Nel caso che le due circonferenze abbiano uguale raggio, illuogo e datodall’asse del segmentoOO′. Il massimo dell’angolo visuale si ha quandoP coincide con il punto medio diOO′, ma nessun punto dell’assee punto di minimo per l’angolo di visuale.

Anno 1997 141

1997.3 Si determini, al variare dei parametriα e β interi pari e positivi, il numero di soluzioni realidell’equazione

(α + β) x sin(π x) = x2 + α β .

RISOLUZIONE Poiche α + β e maggiore di 0 possiamo dividere per questo numero e scrivere l’equazionenella forma

x sin(π x) =x2 + α β

α + β

Il numero delle soluzionie uguale al numero dei punti comuni ai grafici delle funzioni

y = x sin(π x) ,

y =x2 + α β

α + β.

Poiche le due funzioni sono pari basta studiarle perx ≥ 0. Si noti che la prima non dipende dai parametriα eβ e il suo graficoe il seguente:

y

x1 2 3 4

Figura 1

La seconda invece rappresenta una famiglia di parabole con l’asse coincidente con l’assey e con la concavitarivolta verso l’alto; l’ordinata del verticee data day0 = α β/(α + β).La generica parabola di tale famiglia incontra la bisettrice del primo quadrante nei punti la cui ascissaverifica l’equazione

x =x2 + α β

α + β

che si puo scrivere nella formax2 − (α + β) x + α β = 0; le soluzioni sono quindi date da

x1 = α, x2 = β .

Si noti che tutte le parabole incontrano la bisettrice del primo quadrante in punti con ascissa intera positiva eche in tali punti la prima funzione vale sempre 0. Sugli intervalli (2 n−1, 2n) la prima funzionee negativa equindi sicuramente il suo grafico non incontra la parabola; gli incontri possono avvenire solo sugli intervallidel tipo (2n, 2n + 1) contenuti traα eβ, e su ciascuno di questi si sono esattamente due punti di incontro.Chiaramente non si hanno intersezioni seα = β. Supponiamo per fissare le idee che siaα < β; l’intervallo(α , β) conterra (β − α)/2 intervalli del tipo (2n, 2n + 1) e dunque l’equazione avra β − α soluzioni conascissa positiva e 2 (β − α) soluzioni su tutta la retta.

142 I problemi della Scuola Normale

2 4 6

y

x

α = 2β = 6

Figura 2

1997.4* Sia dato un insieme finitoΩ di punti distinti del piano tra loro collegati da un certo numero dipercorsi elementari congiungenti coppie di vertici distinti come esemplificato nel disegno seguente:

Dati due puntiA e B di Ω un cammino che parte daA e termina inB e una successione di verticiv0, v1, v2, . . . , vn di punti di Ω tali che v0 = A, vn = B, e tale chevi e vi+1 sono congiunti da un percorsoelementare; in questo caso si dice chen e la lunghezza del cammino.I punti, i percorsi elementari e i cammini soddisfano le seguenti proprieta:i) i percorsi elementari non si incontrano fuori dai punti diΩ;ii) dati due qualsiasi puntiA, B ∈ Ω, esiste almeno un cammino che parte daA e termina inB;iii) c’e un particolare puntoX ∈ Ω per il quale esiste un cammino che parte e termina inX avente

lunghezza dispari.Si dimostri allora che esisteN intero positivo tale che, scelti due qualsiasi puntiA e B di Ω, esiste uncammino di lunghezzaN che parte daA e termina inB.

RISOLUZIONE Osserviamo per prima cosa che seΩ e ridotto al solo puntoX la tesie vera, conN = 1.Notiamo che se vi sono due cammini di lunghezzal1 e l2 che iniziano e terminano inX, ce n’e anche uno dilunghezzal1 + l2 che si ottiene percorrendo il primo cammino e di seguito il secondo. In particolare esisteun cammino di lunghezza 2 che inizia inX, passa per un altro punto e torna inX. Dunque perX passaalmeno un cammino chiuso di lunghezza 2 e, per ipotesi, un cammino di lunghezza disparim; per un notorisultato sulleequazioni diofantee,dal fatto chem e 2 sono primi fra loro, segue che esistono dueinteri α eβ tali che

m α + 2β = 1.

Anno 1997 143

Necessariamenteα e β hanno segno discorde. Supponiamo che siaα ≥ 0 eβ ≤ 0 (nel caso contrario siragiona in modo del tutto analogo). Scriviamo

m α = 1− 2β .

Vi e dunque un cammino di lunghezzas = −2β ed un altro di lunghezzas + 1 = 1− 2β = m α che partonodaX e tornano inX. FissiamoN0 = s2 e dimostriamo che per ognin ≥ N0 esiste un cammino di lunghezzan che inizia e termina inX; in effetti, dividendon per s si ottienen = a s + b, dovea ≥ s e 0≤ b < s, equindi

n = a s + b = (a − b) s + b (s + 1) .

Questo dimostra l’esistenza del cammino di lunghezzan. Sappiamo per ipotesi che preso un qualunquealtro puntoY ∈ Ω, esiste un camminocY daY adX di lunghezzalY . Siano ora dati due puntiA, B ∈ Ω.Percorrendo il camminocA, poi un cammino di lunghezzan che parte e termina inX e poi il camminocB

in senso inverso si ottiene un cammino che parte inA e termina inB di lunghezzalA + lB + n.

ABX

cA

cB

Figura 1

Se indichiamo conL la massima lunghezza dei camminilA al variare del puntoA in Ω e definiamo

N = 2L + N0

abbiamo chiaramente che una qualunque coppia di puntie collegabile con un cammino di quella lunghezza.

144 I problemi della Scuola Normale

1997.5 In ognuna delle tre figure seguentie disegnato un poligonoP . Si immagini che, in ognuno deitre casi, il poligonoP sia costituito di materiale elastico e flessibile. Si chiededi disegnare o descriveresinteticamente la figuraS che, in ognuno dei tre casi, si ottiene facendo combaciare lefrecce dei lati chehanno la stessa lettera, e di illustrare come questi lati appaiono inS.

P P P

a

a

b b c a

d ba

d bc a

a

b c

c b

RISOLUZIONE Iniziamo con il primo poligono. Possiamo arrotolarlo a forma di cilindro, facendo coinciderei segmenti marcati cona, e curvarlo poi orizzontalmente portando a combaciare le basi del cilindro, chesono le trasformate dei segmentib. E facile vedere che la direzione delle frecce viene rispettata.

b b

a

Figura 1

La figura che si ottienee quella di una ciambella, untoro in linguaggio matematico.

a

b

Figura 2

Procedendo in ordine di numero di lati, passiamo al terzo poligono. Dato che il materiale di cuie costituitoe elastico, il risultato a cui si arriva effettuando gli incollamenti partendo da questo poligonoe lo stesso diquello che si ottiene partendo dai poligoni in Figura 3.

a

a

b

c

c

b

a

a

b

c

c

b

Figura 3

Anno 1997 145

Il primo di tali poligoni si puo ottenere da quello di partenza contraendolo o dilatandolo(in maniera nonuniforme) nella direzione orizzontale; il secondo si ottiene dal primo trascinando verso l’alto il secondolato verticale.Si vede facilmente che l’ultima figura puo essere arrotolata attorno ad un cilindro in modo che i segmenti asi sovrappongano, rispettando la direzione delle frecce. Il cilindro cosı ottenuto (fig. 4)

c cb ba

Figura 4

puo essere ora curvato in modo da portare a coincidere gli archib e c, ottenendo ancora un toro (fig. 5).

ac

b

Figura 5

Per l’ottagono centrale iniziamo osservando che il problema presenta una simmetria. Come si vede inFigura 6, introducendo un nuovo segmentoz possiamo dividere l’ottagono in due poligoni tali che suognuno gli incollamenti avvengono indipendentemente dall’altro. La figura ottenuta puo essere deformatain maniera equivalente come mostrato in Figura 7.

da

b

c a

d bc

z z

d a

c b

d a

c b

z z

Figure 6, 7

Facendo ora combaciare i latid e i lati a otteniamo due cilindri, che deformiamo ulteriormente ritraendo gliarchiz fino a far coincidere gli estremi degli archic e b(fig. 8).

146 I problemi della Scuola Normale

c c

d z

b b

az

c c

dz

b b

az

Figura 8

Pieghiamo ora ad anello i due cilindri in modo da incollare gli archi c e b; si ottengono due ciambelle a cuisono state tolte due calottine che hanno per bordo gli archiz. Incollando i due tori lungo tali archi si ottieneuna ciambella con due buchi (fig. 9).

b

a

c

d

Figura 9

Anno 1997 147

1997.6* Si determinino tutti gli interi positivin che sono divisibili per tutti gli interi positivi minori ouguali a

√n.

(Suggerimento: considerare il minimo comune multiplo dei numeri minori o uguali a√

n).

RISOLUZIONE Siam il minimo comune multiplo di tutti gli interi minori o ugualia√

n e sia

m = pν1

1 pν2

2 . . . pνs

s

la sua scomposizione in fattori primi.Sen e divisibile per tutti gli interi non superiori a

√n si ha necessariamente che

√n < pi

νi+1 per i = 1, 2, . . . , s

e quindi(√

n)

s < p1ν1+1 · · · ps

νs+1 = m · (p1 · · · ps) ≤ m2 ;

d’altra partem e minimo comune multiplo di divisori din e dunquee ancora un divisore din; in particolaresi ham ≤ n.Si ha quindi

(√

n)s < m2 ≤ n2

e dunques < 4; ne segue chep4 = 7 >√

n e quindin < 49.A questo punto una semplice analisi dei vari casi mostra che gli unici valori possibili per n sono:1, 2, 3, 4, 6, 8, 12, 24.

SECONDA SOLUZIONE

Da una analisi immediata vediamo che 1, 2, 3 godono della proprieta richiesta; possiamo quindi studiare ilcason ≥ 4.Siak il pi u grande intero per cuik2 ≤ n; avremo quindi

k2 ≤ n < (k + 1)2 .

Tantok quantok − 1 sono, per ipotesi, divisori din e sono primi fra loro, per cui il loro prodottoe undivisore din, e strettamente minore din perche k (k − 1) < k2 ≤ n e dunquek (k − 1) ≤ n/2.Possiamo dedurne che

k (k − 1) ≤ n

2≤ (k + 1)2

2

da cui 2k (k − 1) ≤ (k + 1)2, o ancorak2 − 4k − 1 ≤ 0.Ne segue che deve esserek ≤ 2 +

√5 < 5, e quindin < 36.

Come nella soluzione precedente, l’ultimo passo consiste nel verificare che i valori ammissibili sono datida 1, 2, 3, 4, 6, 8, 12, 24.

148 I problemi della Scuola Normale

Glossario

In questa appendice si richiamano succintamente alcune nozioni e proprieta utili nella soluzione deiproblemi riportati in questo volume o di altri problemi simili.

Come ricordato nella introduzione, i richiami sono schematici ed essenziali; sie preferito rinunciaread una maggiore generalita e ad un maggior rigore a favore di una presentazione il piu possibile agile esemplice.

BARICENTRO, di un triangolo. E il punto in cui concorrono le mediane del triangolo. Il baricentrodivide ogni mediana in due parti, di cui quella che ha un estremo nel verticee doppia dell’altra. Si dicebaricentro di un sistema di puntiP1, P2, . . . , Pn il punto le cui coordinate sono la media aritmetica dellecoordinate dei puntiPk.

BINOMIO DI NEWTON Dati due numeria, b e un intero positivon si ha

(a + b)n = an+(

n

1

)an−1 b +

(n

2

)an−2 b2 + · · ·+

(n

n − 1

)a bn−1 + bn ,

ove i coefficienti(

n

k

)sono icoefficienti binomiali(vediCombinazioni).

CIRCONFERENZA DI APOLLONIO Dati due puntiA, B e un numero positivok, e il luogo dei puntiP peri quali

AP = k BP ;

tale luogoe una circonferenza che ha come diametro il segmentoMN , oveM e N sono i due punti dellaretta perA eB che hanno distanze daA e daB nel rapporto assegnatok (uno di tali puntie interno adAB,l’altro esterno).

COMBINAZIONI, di n elementi ak a k (conk ≤ n). Sono i sottoinsiemi (non ordinati) dik elementiscelti in un insieme din elementi. Il loro numeroe dato dalcoefficiente binomiale

(n

k

)=

n!k! (n− k)!

.

150 I problemi della Scuola Normale

CONGRUENZE ARITMETICHE Si dice che l’interoa e congruo all’interob modulom, e si scrivea ≡ b(modm), sea e b hanno ugual resto nella divisione perm (cioea − b e multiplo dim).Valgono le seguenti proprieta:sea ≡ b (modm) e c ≡ d (modm), alloraa + c ≡ b + d (modm) ea c ≡ b d (modm).

COORDINATE BARICENTRICHE, di un punto in un triangolo. Dato un triangolo non degenere diverticiA ≡ (x1, y1), B ≡ (x2, y2), C ≡ (x3, y3), le coordinate baricentriche di un puntoP ≡ (x, y3) appartenente altriangolo sono la terna di numeri,c1, c2, c3, conci ≥ 0, c1 + c2 + c3 = 1 per cui si ha

x = c1 x1 + c2 x2 + c3 x3

y = c1 y1 + c2 y2 + c3 y3 .

BC

A

L

MN

P

Condotte le rette perP dai vertici del triangolo fino ad incontrare i lati opposti,e facile vedere che lecoordinate baricentriche diP sono date da

c1 =AM

AC=

AN

AB

c2 =BL

BC=

BN

BA

c3 =CM

CA=

CL

CB.

In maniera del tutto analoga si definiscono le coordinate baricentriche per un punto in un tetraedro.

DERIVATA La derivata di una funzionef (x) in un puntox0 e il limite (se esiste) perh → 0 delrapporto incrementale

f (x0 + h) − f (x0)h

.

Tale numeroe anche la tangente trigonometrica dell’angolo formato conl’assex dalla retta tangente algrafico dif nel punto (x0, f (x0)).

DISTANZA, tra due rette sghembe. Date due rette sghembe, esiste uno e unsolo segmento che ha unestremo su ciascuna di esse ede perpendicolare ad entrambe; essoe il segmento piu breve fra quelli checongiungono un punto dell’una con un punto dell’altra; la sua lunghezzae la distanza fra le due rette.

DISUGUAGLIANZA FRA LE MEDIE Dati n numeria1, a2, . . . , an, la loro media aritmeticae

Glossario 151

A =a1 + a2 + · · · + an

n,

mentre la loro media geometricae

G = n

√a1 a2 · · · an

n.

RisultaA ≤ G e l’uguaglianza vale solamente quandoa1 = a2 = · · · = an.

DISUGUAGLIANZA TRIANGOLARE 1. Ogni lato di un triangoloe minore della somma e maggiore delladifferenza degli altri due; tale condizionee necessaria e sufficiente affinche tre segmenti possano essere latidi un triangolo.2. Dati due numeria, b si ha

||a| − |b|| ≤ |a + b| ≤ |a| + |b| .

EQUAZIONE DIOFANTEA E una equazione polinomiale, a coefficienti interi, di cui sicercano soluzioniintere. Ad esempio l’equazione diofanteaa2 + b2 = c2 ha per soluzioni le terne pitagoriche.E un importante risultato sulle equazioni diofantee del primo ordine che l’equazione inx, y

m x + n y = p

ammette soluzioni se e solo se il massimo comun divisore dei coefficientim e n divide il termine notop ein particolare ha soluzione per ognip sem en sono primi fra loro.Vari problemi riguardanti le equazioni diofantee hanno trovato soluzione solo in tempi recenti. Nel 1970Matyasevich ha dimostrato che non esiste un algoritmo in grado di decidere se una data equazione diofanteaammetta soluzioni, rispondendo cosı al decimo problema di Hilbert.Nel 1995 Wiles ha dimostrato che sen e un intero maggiore di 2 l’equazione diofanteaan + bn = cn non hasoluzioni (ultimo teorema di Fermat).

FATTORIALE, di un numero naturalen. E il prodotto dei numeri naturali minori o uguali an, e siindica conn!:

n! = n · (n − 1) · (n − 2) · · · 3 · 2 · 1 .

Si pone per convenzione 0! = 1. Il numero di permutazioni din oggetti e dato dan!, come si verificafacilmente usando ilprincipio di induzione.

FORMULA DI EULERO Se un poliedro convesso haV vertici, S spigoli eF facce, allora

V − S + F = 2.

Ad esempio per il cubo si haV = 8,S = 12,F = 6, per il tetraedroV = 4, S = 6,F = 4.Una versione della formula di Eulero riguarda i grafi planaricon archi che non si intersecano: in un talegrafo, sen e il numero dei nodi,a quello degli archi, allora il grafo divide il piano inr regioni e si ha

r = a − n + 2.

152 I problemi della Scuola Normale

FORMULE PARAMETRICHE

cosα =1− t2

1 + t2, sinα =

2t

1 + t2, ovet = tg

α

2.

FUNZIONE CONTINUA Informalmente si dice continua una funzionef (x) tale che il valore da essaassunto cambia di poco per piccoli cambiamenti della variabile x. Piu precisamente, una funzionef (x) sidice continua in un puntoa se per ogni numeroε > 0 si puo trovare un numeroδ tale che

se |x − a| < δ allora |f (x) − f (a)| < ε .

Una funzionee continua in un intervallo see continua in ogni punto di tale intervallo.

FUNZIONE CONVESSA Una funzionef (x) definita su un intervalloI della retta si diceconvessase suogni intervallo [x, x′] contenuto inI il grafico della funzione giace al di sotto della retta secante nei punti(x, f (x)) e (x′, f (x′)) o, in simboli,

f ((1− t) x + t x′) ≤ (1− t) f (x) + t f (x′) per ognit ∈ [0, 1].

Una funzione convessa su (a, b) e continua in ogni punto dell’intervallo; una funzionef (x) derivabile duevolte in (a, b) e convessa se e soltanto sef ′′(x) ≥ 0 in ogni puntox di (a, b).

FUNZIONE AFFINE E una funzione espressa da un polinomio di primo grado. Una funzione di unavariabilee affine se e solo se ha per grafico una retta; in due variabili ilgraficoe un piano. Una trasformazionein cui le coordinate del punto di arrivo sono funzioni affini delle coordinate del punto di partenzae dettaaffine.

GRAFO Un grafoe un insieme finito di oggetti, dettinodidel grafo, e diarchi che connettono coppiedi nodi. Un grafoG si dicecompletose ogni coppia di nodie collegata da un arco, si diceconnessose presicomunque due nodin, n′ e possibile passare dan a n′ seguendo uncamminoformato da archi diG. Uncammino chiuso, che parte ed arriva in uno stesso nodo, visitando gli altri nodi una sola volta,e dettociclo.Un grafo connesso privo di ciclie dettoalbero.SeG e un grafo, il suo complementareG′ e definito come il grafo che ha gli stessi nodi diG ed ha l’insiemedegli archi complementare a quello degli archi diG: due nodi sono connessi inG′ se e solo se non losono nel grafoG. Un primo risultato sui grafie il seguente: se un grafo none connesso allora il suocomplementare loe (ma il viceversae chiaramente falso in generale).

INSIEME CONVESSO Un insieme di punti della retta, del piano o dello spazio si diceconvessose, datidue qualunque suoi punti, il segmento che li uniscee interamente contenuto nell’insieme.

INSIEME NUMERABILE Un insiemeA e numerabile se si puo mettere in corrispondenza biunivocacon l’insieme dei numeri naturali. L’insieme dei numeri razionali e numerabile, l’insieme dei polinomi acoefficienti interie numerabile mentre none numerabile l’insieme dei numeri reali.

Glossario 153

INVOLUCRO CONVESSO o inviluppo convesso di un insiemeA e il piu piccolo insieme convesso checontieneA; essoe dato dall’intersezione di tutti gli insiemi convessi contenentiA.

OMOTETIA Dato un puntoO e un numero realek, l’omotetia di polo (o centro)O e rapportok e unatrasformazione in cui ad ogni puntoA corrisponde un puntoA′ sulla semiretta perA uscente daO sek epositivo, della semiretta opposta sek e negativo, in modo che

OA′ = k OA .

Ogni omotetia manda rette in rette, conserva gli angoli e i rapporti fra i segmenti ede dunque una similitudine.

PRINCIPIO DEI CASSETTI, (o dei nidi di piccione): sen oggetti vengono posti in (n−1) cassetti almenoun cassetto contiene piu di un oggetto. Se infatti in ogni cassetto vi fosse al piu un oggetto, il numero deglioggetti non potrebbe superare quello dei cassetti.

PRINCIPIO DI IDENTITA DEI POLINOMI Se due polinomiP (x) e Q(x) di grado minore o uguale anassumono valori uguali in almenon + 1 valori distinti dix, essi hanno uguali i coefficienti dei termini similie quindi coincidono; in particolare si ha cheP (x) = Q(x) per ognix.

PRINCIPIO DI INDUZIONE Ad ogni intero positivon sia associata una proprieta P (n), che puo esserevera o falsa. Se(a) P (1) e vera;(b) per ognik, P (k) implicaP (k + 1),alloraP (n) e vera per ogni intero positivon.Il principio di induzione puo essere assunto come assioma nella presentazione dei numeri naturali, o, avolte, e derivato assumendo come assioma altre proprieta equivalenti, come ad esempio che ogni insiemedi numeri naturali ammette un primo elemento.Ecco alcuni esempi tipici di risultati che si possono dimostrare facilmente con il principio di induzione:- per ogni intero positivon

n∑

k=1

k2 =16

n (n + 1) (2n + 1) ;

- per ogni intero positivon, (cosϑ + i sinϑ)n = cosn ϑ + i sinn ϑ;- per ogni intero positivon, n3 − n e divisibile per 6.Ovviamente il principio di induzione puo essere usato per dimostrare che la proprieta P (n) e verificata daogni numero naturale maggiore o uguale a un fissato interon0 se la (a)e sostituita da(a’) la proprietaP (n0) e vera.

PROBABILITA Se un esperimento han possibili risultati e un certo eventoE si realizza ink di essi, sidice che la probabilita dell’eventoE e

P (E) =k

n.

In questa definizione si presuppone di sapere che, deglin risultati dell’esperimento, nessuno sia privilegiatorispetto agli altri, cioe che essi sianoequiprobabili; la definizionee quindi tautologica. In molti casi

154 I problemi della Scuola Normale

concreti, tuttavia, basandosi sul comportamento statistico dei risultatie ragionevole assumere l’ipotesi chei singoli risultati siano equiprobabili. Cosı ad esempio lanciando un dado non truccato possiamo dire chel’evento “esce 1” ha probabilita 1/6, l’evento “esce un numero pari” ha probabilita 3/6, ecc.

PRODOTTO SCALARE, di due vettori nel piano o nello spazio. Il prodotto scalare dei vettori u e v,indicatou · v, e definito come prodotto delle lunghezze dei vettori per il coseno dell’angolo compreso; ilsuo valore assoluto coincide con l’area del parallelogramma che hau e v per lati, e il segnoe positivo sela proiezione diu suv ha lo stesso verso div, e negativo altrimenti. Seu e v hanno componenti (u1, u2) e(v1, v2) rispetto ad un sistema di riferimento ortogonale, il loro prodotto scalaree dato da

u · v = u1 v1 + u2 + v2 ,

ed una analoga espressione ha il prodotto scalare di due vettori dello spazio.

PROGRESSIONE ARITMETICA Una successione di numeria1, a2, . . . , an, . . . e detta progressionearitmetica se la differenza tra due termini consecutivie una costanter, dettaragionedella progressione.Si puo dunque definire per ricorrenza una progressione aritmetica tramite le formule

a1 = a , an = an−1 + r pern > 1 .

L’espressione generale del terminen-esimoe data da

an = a + (n − 1)r .

La sommasN dei primiN termini di una progressione aritmetica puo essere calcolata con la formula

sN =a1 + aN

2N =

2a + (N − 1)r2

N .

PROGRESSIONE GEOMETRICA E detta progressione geometrica una successione di numeria1, a2, . . . ,an, . . .per la quale il rapporto tra due termini consecutivie una costanter; tale costantee dettaragionedella progressione.Una progressione geometrica si puo definire per ricorrenza tramite le formule

a1 = a , an = an−1 r pern > 1 .

L’espressione generale del terminen-esimoe data da

an = a rn−1 .

La sommasN dei primiN termini di una progressione geometrica con ragioner 6= 1 e fornita da

sN = arN − 1r − 1

.

Se|r| < 1 si possono anche “sommare” gli infiniti termini della progressione ottenendo

s = a (1 + r + r2 + . . . + rn + . . .) = a1

1− r.

Ad esempio(

1 +12

+14

+ . . . +12n

+ . . .

)=

11− 1/2

= 2.

Glossario 155

RAPPRESENTAZIONE NUMERICA IN BASEb, dove b e un intero positivo maggiore di 1. Ogni interopositivoa si puo scrivere (dividendolo ripetutamente per la base e prendendo i resti di tali divisioni) nellaforma

a = a0 + a1 b + a2 b2 + · · · + an bn ,

ove 0≤ ai < b per ognii. Possiamo scrivere piu compattamente tale rappresentazione come

a = (anan−1 . . . a1a0)b

Seb e 10, tale scrittura coincide con la usuale rappresentazione decimale.

156 I problemi della Scuola Normale

REGOLA DEI SEGNI DI CARTESIO Sia

P (x) = a0 xn + a1 xn−1 + · · · + an−1 x + an

un polinomio a coefficienti reali. Si scrivano in ordine i segni dei coefficienti non nulli (+ o−). Il numerodelle radici positive del polinomio non supera il numero deicambiamenti di segno. Ad esempio, il numerodi radici positive del polinomio 8x9 − 7x6 − 3x5 + 2x3 + 4 non supera 2.

SIMILITUDINE Trasformazione che conserva i rapporti tra i segmenti. Similitudini particolari sono leomotetie, le traslazioni, le rotazioni, le simmetrie e le loro composizioni.

TEOREMA DEL PARALLELOGRAMMA La somma dei quadrati costruiti sui lati di un parallelogrammaeequivalente alla somma dei quadrati costruiti sulle diagonali. E una facile conseguenza del teorema dellamediana.

TEOREMA DELLA BISETTRICE 1. La bisettrice interna di un triangolo divide il lato opposto in partiproporzionali ai due lati che la comprendono.2. Il quadrato di una bisettrice di un triangoloe equivalente al rettangolo dei lati che la comprendono menoil rettangolo dei segmenti che essa determina sul terzo lato.

TEOREMA DELLA MEDIANA Il doppio del quadrato di una mediana di un triangoloe equivalente allasomma dei quadrati dei due lati che la comprendono meno la meta del quadrato del terzo lato.

TEOREMA DELLA SECANTE E DELLA TANGENTE Condotte da un puntoP esterno ad una circonferenzaC

una semirettaPA tangente aC in A e una retta che intersecaC nei puntiB eC si haPB : PA = PA : PC.

TEOREMA DI ESISTENZA DEI VALORI INTERMEDI Se una funzione continuaf (x) e definita sull’intervallo[a, b] e t e un numero compreso fraf (a) e f (b), c’e almeno un puntoc in [a, b] per cui si haf (c) = t. Inparticolare una funzione continua definita su un intervallolimitato e chiuso assume almeno una volta ognivalore compreso fra il suo minimo e il suo massimo.

TEOREMA DI PITAGORA E il piu noto teorema di geometria piana: in un triangolo rettangolo il quadratocostruito sull’ipotenusae equivalente alla somma dei quadrati costruiti sui cateti.In altri termini, sec e lamisura dell’ipotenusa ea, b le misure dei cateti si haa2 + b2 = c2.Una elegante dimostrazione si puo ottenere suddividendo un quadrato di latoa + b in due modi differenti,come mostra la figura, e uguagliando le aree.

Glossario 157

a b

a

b

ca

bc

TEOREMA DI RUFFINI Un polinomio P (x) = a0 xn + a1 xn−1 + · · · + an−1 x + an e divisibile per ilbinomio (x − α) se e solo seα e radice diP (x).SeP (x) ha radiciα1, α2, . . . , αk rispettivamente di molteplicitah1,h2, . . . , hk, (h1 + h2 + · · ·+ hk = n) alloraP (x) si puo scomporre in fattori lineari

P (x) = a0 (x − α1)h1 (x − α2)

h2 · · · (x − αk)hk .

TEOREMA DI TALETE Tre rette parallele staccano su due trasversali coppie di segmenti in proporzione.Vale anche il viceversa: se i segmenti staccati sono fra loroin proporzione le tre rette sono parallele. Inparticolare una omotetia del piano o dello spazio trasformauna retta in una retta ad essa parallela.

TERNA PITAGORICA E un insieme di tre interi positivia, b, c tali chea2 + b2 = c2.Sea, b, c e una terna pitagorica , allora anchek a, k b, k c e una terna pitagorica per ognik intero positivo.La formula

a = 2m n , b = m2 − n2 , c = m2 + n2

al variare dim, n fra gli interi positiivi conm > n fornisce infinite terne pitagoriche.Sono terne pitagoriche, ad esempio,3, 4, 5, 5, 12, 13, 8, 15, 17.